Consulting Case Operations Strategy [PDF]

  • 0 0 0
  • Gefällt Ihnen dieses papier und der download? Sie können Ihre eigene PDF-Datei in wenigen Minuten kostenlos online veröffentlichen! Anmelden
Datei wird geladen, bitte warten...
Zitiervorschau

Consulting Case Operati ons strategy

Contents Spanish Juices Company to Reduce High Spoilage Rate..................................................................1 Chicago Booth to Increase On-campus Internship Offers...................................................................3 Discount Store Chain Kmart to Cut Costs of Distribution....................................................................5 PepsiCo Set to Run Nationwide Promotion Campaign.......................................................................7 Netflix to Test New Streamed Video Product in 5 Pilot Cities...........................................................10 How to Prevent Another Hurricane Katrina?.......................................................................................11 Ashland Chemical Company to Develop New Business Model.......................................................13 Bristol-Myers Prioritizes Drug Candidates to Cut R&D Cycle Time.................................................17 Aspen Mountain Ski Field to Build a Second Ski Lift.........................................................................18 Washington Post to Incease Migration from Print to Online.............................................................21 Takeda Pharma to Delay Launch of New Drug Delivery Product....................................................22 NYC Rolls Out Nation’s Largest Free Public Wi-Fi Network.............................................................24 Whirlpool to Re-focus on End Consumer and Product Quality.........................................................26 Direct Relief to Increase Donation Made by General Public.............................................................29 Boehringer Ingelheim to Optimize Current R&D Portfolio.................................................................34 Software Firm VMware to Set Up Engineering Unit in India.............................................................36 Warner Bros. to Do Simultaneous Global Release for Batman 3....................................................39 Newmont Mining Corporation to Exploit Gold Mine in Peru..............................................................42 Cisco Closes Minnesota Plant to Remove Excess Capacity............................................................46 Cummins To Reduce Warranty Claims for Diesel Engines..............................................................48 Allure Magazine to Close Printing Facility in Idaho............................................................................51 Trader Joe’s Chain to Centralize Company’s Sourcing.....................................................................53 Airbus to Build New A380 Assembly Facility in France.....................................................................55 Revol Wireless to Focus Investment in In-bound Call Centers........................................................58 Mayor of Baltimore Aims to Grow City Population by 5%.................................................................61 R&R Ice Cream to Reduce Sales Seasonality....................................................................................64 Red Bull’s Marketing Campaign Not a Huge Success.......................................................................68 Adobe Systems to Not Introduce “Suite” Software Product..............................................................69 Hedge Fund Firm Citadel to Hire More Quant Analysts....................................................................74

Riva Group to Exit Hot Roll Commodity Steel Production.................................................................78 City of Atlanta to Develop Strategic Plan to Revive Economy..........................................................81 Canadian Oil Sands Concerned about Cost & Time Overruns........................................................83 MDP to Re-design Supply Chain for Metal Processing Factory.......................................................88 AGCO Optimizes Operations Among Different Business Units........................................................90 Perot Systems to Provide Free Cell Phones to Project Teams........................................................92 Robert Joss to Step Down As Dean of Stanford GSB.......................................................................94 Marriott Upgrades Service to Compete with Four Seasons..............................................................96 What Operating Metrics to Look at in a Consulting Firm?.................................................................97 Kaiser Permanente Develops Strategy to Manage Suppliers...........................................................98 McDonald’s to Offer More Health Food Options in the U.S..............................................................99 Poland Springs Create Private Label Bottled Water for Walmart...................................................101 Whole Foods Grocery Store to Cut Tomato Price by Half..............................................................102 The Hartford to Develop 5-year Strategic Plan for Operations.......................................................105 Rockwell Collins to Diagnose Problems in Supply Chain................................................................106 Emerged from Bankruptcy, Tronox Looks Ahead to the Future.....................................................108 Marathon Petroleum to Offer Co-branded Credit Card....................................................................111 Scotch Whisky Johnnie Walker Develops Marketing Strategy.......................................................112 P&G Considers Consolidating Purchasing Efforts...........................................................................114 HSBC Bank Strives to Improve Customer Service Quality.............................................................115 Johnson Controls to Boost Its Return on Net Asset by 5%.............................................................116 Ford Kills Competing Models in Fear of Cannibalization.................................................................119 GM Research Lab to Prioritize Products Development...................................................................120 What is Biggest Challenge for a New University President?..........................................................125 Symantec Turns Down PC Maker’s Product Bundling Offer...........................................................126 Unilever USA Retains PwC to Improve Profitability.........................................................................129 PwC to Develop Business & IT Strategy for Telekinesis.................................................................133 Century 21 Real Estate to Increase Earnings by 200%..................................................................137 Emerged from Bankruptcy, Chemtura Seeks Strategic Alternatives.............................................140 Romance Publisher Harlequin Rejects Bookstore’s Deal...............................................................143 Ryder to Grow Business & Offer Maintenance Only Product.........................................................144 Microsoft Pursues Strategy for Penetrating Large Organizations..................................................147 US Airways to Re-paint Its Fleet of 350 Aircrafts.............................................................................149

What Would Southwest Do If Oil Price Dropped to Zero?..............................................................150 Bank of East Asia Improves Cost-to-Revenue Ratio.......................................................................152 DeVere Group Assesses Health Club Industry Trend.....................................................................154 Should Cab Driver Stay in O’Hare or Go Back to City?...................................................................157 Burton Snowboards Develops Profit Growth Strategy.....................................................................159 Lockheed Martin Reduces Costs of Cyclical Swings.......................................................................161 How to Analyze Spanish Conquest of Inca Empire?.......................................................................164 Marriott Hotels to Develop E-commerce Strategy............................................................................165 US Steel Concerned about Vulnerability to Market Cycles.............................................................167 Airbus Replaces CAD-CAM with Virtual Reality Technology..........................................................168 Corn Products International Refurbishes Ohio Plant.......................................................................170 How to Save Bank One’s Retail Lockbox Services?........................................................................171 Meijer Considers Establishing Centralized Warehouse...................................................................173 Alcoa Improves Manufacturing Process and Reduces Cost...........................................................174 AIG Assess Corporate Compensation Structure..............................................................................176 Naked Juice Considers Selling Chilled Juices Business.................................................................176 P&G Pet Food Approached by Wal-Mart...........................................................................................178 Kimberly-Clark to Increase Profitability in Paper Stationery...........................................................179

Spanish Juices Company to Reduce High Spoilage Rate Case Type: operations strategy, value chain optimization. Consulting Firm: Arthur D. Little first round full time job interview. Industry Coverage: consumer goods; food & beverages. Case Interview Question #00770: Our client Exotic Juices Company is a consumer packaged goods company located in Madrid, Spain. The company’s main business is to produce fruit pulp and extracts for manufacturing different flavored beverages. The business has been owned and managed by

a  year.

Spanish family for several decades. It has revenues of 100 million Euros a

The client Exotic Juices Company sells its products through two major distribution channels: to restaurants and to retail stores such supermarkets. It has been facing a recurring problem of spoilage and has displayed higher spoilage rates as compared to its competitors. An Arthur D. Little case team has been engaged to help them solve this problem. What can be done to reduce their high spoilage rate? Additional Information: (to be provided to candidate upon request)  Distribution channels: wholesale suppliers to restaurants, supermarket private labels.  Shelf life of 180 days with customers requiring 60 days.  High recipe switching costs.  Competitors’ spoilage rates are only one-fourth of our client’s.  Spoilage occurring at the client’s distribution centers and not at the customers’ facilities.  The client’s capacity utilization is 80%. Possible Answer: Question #1: Lay out the end-to-end value chain Suggested Solution: Demand forecast –> Sourcing –> Manufacturing –> Sales & Distribution Question #2: Estimate the demand of pulp and extracts for our client Additional Information: (Only Provide if Asked) 1 gram of fruit extract is required per 200 ml of beverage (industry standard) Total market size of the restaurant channel: 1.5 billion liters Market share in the restaurant channel: 30% Total market size of the supermarket channel: 750 million liters Market share in the supermarket channel: 40% Suggested Solution: Total demand for the client’s products: (30% * 1.5 billion liters + 40% * 750 million liters) * 1 gram/200 ml = 3.75 million kilograms Additional Information to be provided next to interviewee: The client’s forecasting tools and methods are out dated and generate estimates that are not in tune with the current market demand. The tool generates production requirements of 4.9 million kilograms annually. The interviewee should be asked for reaction and possible solutions: The client over produces by 1.15 million kilograms which would contribute to the problem of excessive spoilage. The client also faces a severe problem of over-capacity. Candidate should suggest means by which capacity can be reduced (by selling off facilities) or effectively utilized (renting to other manufacturers, for instance). Question #3: Where else in the value chain could there be possibility of wastage or spoilage? Suggested Solution:

Ideally the candidate should examine each step in the value chain:

Sourcing The client sources its raw materials from 3 main suppliers. In recent years Additional relations with one of them has Information to soured and consequently be provided if quality of raw materials has asked declined considerably.

Insight

Spoilage could occur due to low quality of raw materials used. This in the long run can affect the client’s credibility in the market. Re-evaluate available options and consider changing suppliers.

Manufacturing

Sales & Distribution

2 distribution centers: one in Barcelona and Client has recently one in Sevilla. Some refurbished its production products have very low equipment, installing the inventory turn overs latest available exceeding the 180 day technology. shelf life.

Manufacturing does not directly contribute to spoilage but due to mistakes in demand forecast there is a problem of over production.

Rationalise product SKUs. Eliminate those with very low turn-over rates.

Conclusion: Final recommendation should include a summary of all the problems analyzed with insights drawn from each clearly stated. Strong candidates will also include actions to be taken.

Chicago Booth to Increase On-campus Internship Offers Case Type: operations strategy, value chain; math problem. Consulting Firm: IBM Global Business Services (GBS) first round summer internship job interview. Industry Coverage: Education & Training Services. Case Interview Question #00755: The Booth School of Business is a graduate business school located in Chicago, Illinois, at the University of Chicago. Formerly known as the University of Chicago Graduate School of Business, Booth is the second-oldest business school in the U.S., the first such school to offer

an  Executive MBA program, and the first to initiate a Ph.D. program in business. The school was renamed in 2008 following a $300 million endowment gift to the school by alumnus David G. Booth. US News, in the 2013 Best Business Schools ranking, ranked Chicago Booth 4th in the U.S. The Booth School of Business is looking to promote its MBA program’s reputation and ranking position, by improving its on-campus summer internship employment stats. Currently, only 60% of Booth’s first

year full-time MBA students secure an internship through on-campus recruiting. The Dean of Chicago Booth has hired our consulting firm to provide insight and recommendations on how to improve the oncampus offers. What would you recommend him to do? Additional Information: (to be provided upon request) Assume that Booth School of Business has 500 full-time MBA students in each class. Push the interviewee to understand the “value chain” of on-campus recruiting (see below) Wait for the interviewee to ask clarifying questions about specific objectives of this case: Primary objective: increase on-campus internship offers to 75%. Secondary objective: the school administration is very cost sensitive and is only willing to spend up to $500K on this project. Possible Answer: 1. Value Chain Analysis  

First, ask the interviewee to brainstorm the value chain for Booth to assist its first-year MBA students to get internship offers. Data for current on-campus recruiting:  # of companies that recruit on-campus = 100  average # of positions offered per company = 2  # of interview slots per position = 15  Interview success rate = 10% (assumption = each MBA student receives only one offer) Then, ask the interviewee to brainstorm on possible ways to increase the number of total offers made (he/she should go over the “value chain”) The Booth school is looking into two possible strategies:  Increase the number of companies that recruit on campus  Improve the interview success rate 2. Quantitative Analysis a. Increase the number of companies To attract more companies to recruit on-campus, Booth’s Office of Career Services (OCS) needs to hire additional firm relations managers. Each manager can handle 5 companies and requires an annual salary of $75K. Additional costs (including travel, marketing expenses, etc.) per relations manager are estimated at $50K.

Target # of offers = 500 * 75% = 375 Current # of offers = 500 * 60% = 300 (375 – 300) / 300 = 25% Thus, Booth needs to increase the number of companies recruiting on-campus by 25% * 100 = 25 # of additional OCS firm relations managers needed = 25/5 = 5 Annual cost of firm relations managers = 5 * ($75K + $50K) = $625K b. Increase the interview success rate The interviewer should have the interviewee brainstorm on possible ways to increase interview success rate. According to a recent survey, the most important factor in interview success rate is the number of mock interviews. For every 0.5% increase in interview success rate, Booth will need to hire 15 second-year MBA students as part-time mock interview counselors. Each second-year MBAs works 40 hours per month, with an hourly wage of 20$. Recruiting season lasts for 5 months. Also, every 2% increase in interview success rate attracts 5 new companies that recruit on campus. Thus, adding 2% in interview success rate: (100 + 5) companies * 2 offers per company * 15 interview slots per offer * (10% + 2%) success rate = 378 offers Annual cost of second-year MBAs = 2%/0.5% * 15 MBAs * 40 hours per month * 5 months * 20$/hour = $240K Note: A good candidate will make sure that we have sufficient second-year MBA “Capacity”. 3. Conclusion a. Recommendation Recommend that Booth hires 60 additional second-year MBA students as part-time mock interview counselors. This will increase the total number of internship offers from 300 to 378 (meeting the goal of increasing on campus offers to 75%). b. Possible Risks (mitigation / next steps)  

Difficulty recruiting so many second-year MBA students (can even increase hourly wage up to $40 without exceeding target budget of $500K) Economic downturn may cause companies to reduce the number of internship positions / slots.

 

Limited # of study rooms at Booth to accommodate such an increase in the # of mock interviews (reach out to graduate school, law school, medical school, etc to get access to their rooms). With so many second-year MBAs spending so much time on mock interview counseling, their grades may be negatively impacted, affecting the total Booth brand image (employ grade non disclosure policy).

Discount Store Chain Kmart to Cut Costs of Distribution Case Type: reduce costs; operations strategy, optimization. Consulting Firm: Accenture first round summer internship job interview. Industry Coverage: retail; general merchandisers. Case Interview Question #00752: Your client Kmart (sometimes stylized as K-Mart) is an American chain of discount superstores similar to Wal-Mart. The company was founded in 1962 and currently is the third largest discount store chain in the world, just behind Walmart (NYSE: WMT) and Target (NYSE: TG).

Headquartered in Hoffman Estates, Illinois, United States, the client company operates a total of 1,221 Kmart stores in 49 states, Guam, Puerto Rico, and the U.S. Virgin Islands as of 2013, including 1,196 discount stores averaging 94,000 sq ft, and 25 Super Centers averaging 168,000 sq ft. Each store receives one delivery per day from a distribution center. The CEO of Kmart has hired your consulting firm to investigate if the costs related to distribution can be reduced. How would you approach this case? What recommendations would you give to the CEO of Kmart? Additional Information: (to be provided to candidate upon request)

The client Kmart owns multiple distribution centers (DCs) across the United States. (Show figure below) 

The 4 primary distribution centers are located in New York City, Chicago, Dallas, and San Francisco. The 20 secondary distribution centers are also spread out through the country. All of Kmart’s products are imported from Asia and arrive daily at the San Francisco distribution center. This case is mostly about qualitative analysis. Steer the candidate away from attempting any numerical calculations. Possible Answer: 1. Analysis (Wait for candidate to specifically ask about these aspects.) A. Current Distribution Routes Every day, trucks transport products from the San Francisco center to each of the primary and secondary distribution centers.  Trucks from each of the primary DCs also make daily deliveries to the nearby secondary DCs.  Each secondary DC makes one delivery per day to its assigned stores.  All routes to and from the DCs are the same each day regardless of demand (static routing).  Trucks are rented and are of uniform size. B. Capacity 

Not all distribution centers (primary or secondary) are at full capacity. The three primary distribution centers (excluding the West Coast DCs) are well under capacity.  Most trucks are NOT at full capacity.  Demand for products is not the same at all stores. 2. Sample Recommendations 

Deliver from the West Coast DC to the other primary DCs and secondary DCs on the West Coast. Do not deliver from West Coast DC to other secondary DCs.  Consolidate secondary DCs that are not at full capacity.  Determine optimal routes to and from the DCs.  Dynamic Routing – do not make daily deliveries to stores/DCs if there is low demand, or keep some extra inventory at the stores.  Consolidate trucks:  Use smaller trucks if it decreases the cost.  Buy trucks instead of renting. 3. Sample Risks and Next Steps 

A. Risks  Other primary DCs may not have enough capacity to hold the additional inventory.  By consolidating secondary DCs, capacity risks are magnified if demand increases drastically. B. Next Steps   

Determine capacities of different DCs to see which trucks to consolidate. Determine differences in demand of different stores. Investigate trucking contracts and if using different truck sizes or buying trucks would save money.

PepsiCo Set to Run Nationwide Promotion Campaign Case Type: operations strategy, marketing. Consulting Firm: ZS Associates first round full time job interview. Industry Coverage: food and beverage. Case Interview Question #00744: PepsiCo Inc. (NYSE: PEP) is an American multinational food and beverage corporation headquartered in Purchase, New York, United States, with interests in the manufacturing, marketing and distribution of grain-based snack foods, beverages, and other products.

PepsiCo was formed in 1965  with the merger of the Pepsi-Cola Company and Frito-Lay, Inc. PepsiCo has since expanded from its namesake product Pepsi to a broader range of food and beverage brands. PepsiCo has come to your consulting firm for advice on whether they should run a nationwide bottle-top promotion for a major sporting event in year 2012, e.g. the Super Bowl. What factors should PepsiCo consider in order to make this decision, and what recommendation should be given to PepsiCo? Possible Answer:

Part 1: Qualitative Analysis 1. Previous campaigns and scope of the proposed campaign a. Discuss any previous campaigns and any learning points i. Previous campaigns have generally been successful, but PepsiCo has found that they make most profit from high profile campaigns. ii. Previous campaigns have involved prizes ranging from soccer balls or basketballs (depending on the campaign) to all expenses paid trip to a sporting event. b. Lead time involved in changing the design of labels on bottles and bottle tops is minimal and is zero cost. c. Proposed campaign: iii. Discuss the type of sporting event. 2012 is a big year with the Olympics and annual sporting events such as the Super Bowl, NBA playoffs, etc. iv. Collect bottle tops and claim varying level of prizes vs. lucky bottle tops for grand prize vs. another idea or combination of all. v. Begin promotion three months before the sporting event. 2. Competitor campaigns d. Competitors, e.g., Coca Cola, are expected to run similar campaigns. Discuss historical drink preference of customer (Coke vs. Pepsi), and likelihood of switching between the two drinks. Also discuss keeping those incremental PepsiCo drinkers after the marketing campaign. e. Review the impact of other campaigns, such as McDonalds or Burger King, deemed minimal given the different product offerings, i.e. bottle of soda to quench thirst or give energy vs. extra value meal where the drink comes with. 3. Financials of running a campaign – incremental cash flow f. Monthly Revenue – see calculations below g. Costs – see calculations below. h. Time horizon i. Net cash flow 4. Consider the selection of sporting event and interest level of each event, Super Bowl vs. Olympics vs. NBA Playoffs and the difference in incremental revenue. Do this qualitatively and discuss where one would get data from if running out of time.

Part 2: Quantitative Analysis The quantitative analysis of this marketing strategy case should not focus on the total beverage market, but instead focus on the incremental PepsiCo sales that would be gained by the promotion over both the short-term and long-term (by customers switching from Coke to Pepsi). 1. Monthly revenue a. Price: Soda bottle – $1.50, and soda can – $1. Average out to $1.25 b. Quantity: Only target those who are currently indifferent between Coke and Pepsi, seeing as loyal Coke drinkers will not switch. Also consider Pepsi drinkers may switch from drinking another product to Pepsi more often – but not necessary in calculations (for time). Steps required: Size total soda market in USA   

US Population: 300 million Segment by age with life expectancy at 80 years, and assume population uniformly distributed among ages. Therefore 3.75 million in each age band (300m / 80 years) Make assumptions for number of soda drinkers in each category and build table similar to below.

Age group

# in each category

% soda drinkers in category

Frequency of soda drinking

Sodas per month

0–5 5 – 10 10 – 20 20 – 40 40 – 60 60 – 80 c. Make assumption for number of consumers that are indifferent between Coke and Pepsi to arrive at potential sales per month. 2. Monthly costs – take marketing costs as a percentage of revenues for simplification, i.e. 10%. 3. Cost of prizes – estimate costs of flights, hotels, tickets, spending money for 10 people. Can also estimate cost of smaller prizes too but likely to be immaterial. Part 3: Recommended Conclusion

Conclusion should be based on the net cash flow calculated above balances against any qualitative arguments made earlier.

Netflix to Test New Streamed Video Product in 5 Pilot Cities Case Type: new product, new technology; operations strategy. Consulting Firm: IBM Global Business Services (GBS) first round full timejob interview. Industry Coverage: telecommunications & network; information technology (IT). Case Interview Question #00736: Your client Netflix, Inc. (NASDAQ: NFLX) is the largest international integrated communications provider. The company offers on-demand Internet streaming media available to viewers in North and South America, the Caribbean, and parts of Europe (Denmark, Finland, Ireland,

the  Netherlands, Norway, Sweden, and the United Kingdom), and of flat rate DVD-by-mail in the United States, where mailed DVDs are sent via Permit Reply Mail. The company was established in 1997 and is headquartered in Los Gatos, California. It started its subscription-based digital distribution service in 1999, and by 2009 it was offering a collection of 100,000 titles on DVD and had surpassed 10 million subscribers. Recently, Netflix is looking at ways to retain existing customers and gain new customers by increasing their bandwidth for video content delivery. They have rolled the new technology out to 5 pilot cities in the U.S. They started this initiative to address three consumer trends:  Time shifting (e.g. Tivo)  Location Shifting (e.g. Slingbox)  Consumer streamed video content Your consulting firm has been retained by the CEO of Netflix to assess the effectiveness of the initiative. Specifically, you’re asked to address the following five questions:  What factors would you consider to assess the client’s strategy?  How the trends will affect that strategy?  What analysis would you do?  What data would you collect?  Other factors to consider? How would you approach this case? Possible Answers: Use of the new products/technology in the pilot cities. Usage rates will help determine the amount of bandwidth required at peak times and help the client plan effectively for future network expansion. It will also help the client estimate market potential.

Satisfaction ratings from pilot users. This will help them refine their product offerings and determine how effective the enhanced services will be in retaining existing customers and drawing in new ones. Competitors. Assess what the current competitors are in each of the trends (such as Tivo or Slingbox) and how the product stacks up against them. Satisfaction ratings and market research could be used to determine strengths / weaknesses and how to enhance services to compete effectively. Also, explore the potential for stealing customers from other similar service providers to try to quantify the potential additional market share and revenue captured from the network expansion.

How to Prevent Another Hurricane Katrina? Case Type: operations strategy. Consulting Firm: Booz Allen Hamilton (BAH) first round full time job interview. Industry Coverage: government & public sector. Case Interview Question #00735: In August of 2005, hurricane Katrina hit the southern coast of the United States, turning into the most economically damaging and fifth deadliest in history. At least 1,833 people died in the hurricane and subsequent floods. Total property damage was estimated at $108 billion

(2005 USD). The  most severe loss of life and property damage occurred in New Orleans, Louisiana, which flooded as the levee system catastrophically failed. Eventually 80% of the city became flooded and also large tracts of neighboring parishes and the floodwaters lingered for weeks. As part of an effort to prevent similar disasters in the future, the U.S. Federal Emergency Management Agency (FEMA) has approached our consulting firm in order to develop a strategic plan to help prevent such impact the next time a nature phenomenon like hurricane Katrina occurs. How might you approach this problem? Additional Information: (Provide the following information if requested by interviewee) The below information will help focus the interviewee on specific elements of the answer after they have had a chance to lay out their own thinking.  Economic assistance and funds are provided at the state and federal levels.  Federal Emergency Management Agency (FEMA) usually structures their plans in four dimensions: Preparation, Response, Recovery and Mitigation.  Our project should concentrate on Mitigation, meaning how to mitigate the impact of future disasters. Possible Answer: 1. Areas of Discussion Four major areas should be discussed:

 Identify the risks.  Assess, quantify and prioritize impacts to mitigate  Determine plan to mitigate risk  Implement: Training, Communication, Raise awareness 2. Recommended Conclusion This is not a typical business case where traditional “profitability” or “introducing new product” frameworks can be used. It is important not to try to force-fit the standard frameworks, but rather think about the problem in terms of the information you were given and the problem that needs to be solved. Even more than other cases, this case does not have one single right answer; however some approaches have been discussed below. The first phase of the case (before interviewee has been given focus on the mitigation piece) is an opportunity for the interviewee to exhibit structured thinking and sound process approach, there is no right answer. More important is whether the interviewee proposes a logical approach to the problem that covered a broad range of relevant considerations: Once the interviewee has been told to focus on Mitigation, generally their approach should follow a sequence along the lines of: Identify, Assess, Plan, Communicate / implement. Identify:  List out potential natural disasters (not only hurricanes) and risks associated with each.  Inventory of current state of infrastructure: levee, potential refugee centers (i.e. Astrodome) Assess: Out of the potential disasters, consider which one is most likely and should be prioritized for planning purposes.  Out of the risks associated with each disaster, prioritize those risks across all potential disasters that have maximum risk. Plan: 

 Raise necessary funds to invest in infrastructure, communication, etc.  Identify key suppliers and other resources that can be mobilized in the event of emergency.  Create a structured plan to address and communicate the risks and solutions identified. Communicate:    

Raise awareness within the general public; as well as institutions and partners: state, federal, NGO’s, private companies, media, etc. Use tools such as schools, media public announcements, celebrities, etc. in order to create awareness. Establish a conference to gather, align and integrate all parties. Potential of using Global Warming awareness as an issue to further leverage message.

A Second Approach: 1) Determine the objectives and priorities of the Client (FEMA) a. Reduce casualties b. Reduce damage to property 2) Understand the various types of disasters that must be considered a. Earthquate b. Hurricanes 3) Casualties a. Identify the most common causes of death and the number of people at risk b. Evaluate the possible mitigating actions (prevent risk, or mitigate once risk comes to pass) c. Implement/communicate plans 4) Property damage/other losses a. Identify causes b. Prioritize and plan c. Implement/ communicate 5) Other considerations a. Flexibility of response with changing conditions b. Reliability of suppliers c. Effectiveness of communication d. Chain of command

Ashland Chemical Company to Develop New Business Model Case Type: business turnaround; operations strategy. Consulting Firm: Roland Berger Strategy Consultants first round full time job interview. Industry Coverage: chemical industry. Case Interview Question #00730: Your client Ashland Inc. (NYSE: ASH) is an American Fortune 500 chemical company headquartered in Covington, Kentucky. Ashland is a major U.S. chemical manufacturer in the commodity chemicals business with single digit market share. It recently emerged

from bankruptcy  and has limited capital available. The chemicals business is cyclical with pricing cycles of 7 years. The client is worried about how it will survive year 2015 when it hits the bottom of the pricing cycle. The question is how this company can become sustainable, if at all possible. The CEO of Ashland has hired your consulting firm to develop a new business model, either through: (1) acquisition into a non-cyclical chemicals market, or (2) the manufacture of new products and services for customers, or (3) your own recommendations. What would you recommend? Additional Information: (Provide the following information if requested by interviewee) 1. Competitors The commodity chemical market is highly fragmented. A large number of competitors are either standalone or small in size, or are a small division of a larger conglomerate. Acquisition is difficult because of limited capital. 2. Customers Ashland’s customers are highly fragmented, each of them purchasing no more than 10% of the client’s inventory annually. 3. Raw Materials Your client’s raw material suppliers are increasingly turning to “greener” processes and products without increasing their total capacity. This trend is significantly reducing the supply of “non-green” raw materials which drives their cost up. 4. Infrastructure Your client currently has 5 plants. 3 are performing well, one has been having problems of quality consistency recently and one plant has been historically a bad performer in terms of capacity utilization. All plants are over 10 years old and production is spread evenly over all the plants. (none are at full capacity)

5. Government Increasing push from environmental groups has caused additional government taxes and regulations on shipping to be enacted recently. 6. Products The chemical manufacturer Ashland mainly produces two chemicals: X and Y. Chemical Y is a by-product of Chemical X with a weight ratio of X = 1.5Y (each 1.5 ton of X manufactured results in the byproduction of 1 ton of Y).

2012 Costs per 2012 Prices per ton 2013 Prices per ton ton

2013 Costs per ton

$150

$100

$95

Chemical Y $175

$100

Chemical X

$50

The client Ashland expects to sell 100% of the chemicals manufactured. All of its plants operate below capacity; they currently manufacture 100,000 tons of Chemical X annually to meet the market demand. The pricing changes are normal. Possible Answer: Profit margin should be calculated, in addition to the dollar margin derived from Chemical X versus Chemical Y to determine if there is any advantage to changing the product mix. In 2013, 100,000 tons of X are being manufactured at a profit of $5/ton, deriving a profit of $500,000 for Chemical X. The manufacturing of X results in the by-production of 100,000 / 1.5, i.e. around 66,667 tons of Chemical Y or 70,000 tons for rounding purposes. Since Chemical Y is a byproduct of X, one can assume a corresponding cost of $0/ton. The client should thereby derive a profit of $100 x 70,000 = $7,000,000 for Chemical Y, and a total profit of around $7,500,000 for both chemicals. Comparatively, the company generated a profit of $100 x 100,000 = $10,000,000 for Chemical X in 2005, plus an additional $175 x 70,000 = $12,250,000 for Chemical Y. This amounts to a total profit of 22,250,000 in 2012, and therefore a huge loss in profit. These results are summarized in the following Tables:

Year 2006 Volume (ton) Prince/ton Cost/ton Profit margin % Profit margin Chemical X

100,000

$100

$95

5%

$500,000

Chemical Y 66.667

$100

-

100%

$6,666,667

Total

$7,166,667

Year 2005 Volume (ton) Prince/ton Cost/ton Profit margin % Profit margin Chemical X

100,000

Chemical Y 66.667

$150

$50

67%

$10,000,000

$175

-

100%

$11,666,667

Total

$21,666,667

Recommended Conclusion This case is qualitative in nature and can take many directions. The interviewee should first explore the root causes of the sales and profit cycles including the following topics:  Why are costs increasing?  Why is there a reduction in supply of raw materials and can it be addressed?  Who are our client Ashland’s customers?  Can the client company pass them raw material cost increases?  What are the new government regulations’ impact?  How efficient is manufacturing? The interviewee should further review pros & cons of acquisition, joint venture, licensing, divesting a plant or two, and organic growth through new products/services (preferably with a pricing cycle opposite that of X and Y). The dimensions for evaluating each option should include impact on sales/profit, ROI, risk, feasibility, and timing. In addition to organic and inorganic growth, there seems to be opportunities to improve the current business model. For example, since none of the plants operates at capacity and Chemical Y can be sold at a higher price/margin than Chemical X in the upside part of the pricing cycle, the interviewee may ask these questions:  Can the plants increase their production of X and Y when the pricing cycle is up?  Are there opportunities for cost reduction by increasing the volume of raw materials purchased?  Can the more profitable chemical Y be produced in larger quantities in upside cycles? Exploring the trend of raw materials suppliers turning to greener products:  

If the suppliers of raw materials for X and Y are switching to “greener” processes across the industry, can the client pass on the price increase along the value chain? Can the client use this as an opportunity to differentiate its products as “green” as well and therefore charge a premium?

Possible recommendation may include: Launch new products & services to differentiate from the competition, build customer loyalty, gain new customers, and leverage synergies through existing plants.  Operating costs may increase with purchase of new materials/machinery for refining chemicals.  Organic growth must be considered in addition to improving the client’s current business model which does not seem to be working well.  Possible exploration into joint ventures or divesting the company. Any number of recommendations would work for this case as long as the interviewee properly explores all of the options. 

Bristol-Myers Prioritizes Drug Candidates to Cut R&D Cycle Time Case Type: operations strategy, optimization. Consulting Firm: IMS Health Consulting Group first round summer internship job interview. Industry Coverage: healthcare: pharmaceutical, biotech, life sciences. Case Interview Question #00727: Bristol-Myers Squibb (NYSE: BMY) is a U.S. pharmaceutical company headquartered in New York City, NY. The firm manufactures prescription pharmaceuticals in several therapeutic areas, including cancer, HIV/AIDS, cardiovascular disease, diabetes, hepatitis,

rheumatoid arthritis and psychiatric disorders. Your client for this case  is the Chief Scientific Officer of Bristol-Myers Squibb who is in charge of the company’s research and development. Currently, Bristol-Myers Squibb’s R&D process works like this: the company’s satellite research labs located all over the country send potential drug compounds to a centralized analysis center. Analysis center draws blood from animals, do all the testing work, analyzes the results, and then sends results back to satellite research lab – this is an iterative process for each drug compound (5 to 15 reviews per compound). 15 year patent begins after the 1st review/iteration if the compound passes initial tests during the 1st review/iteration. There tends to be a long queue of potential drug compounds waiting to be analyzed. Your client, the Chief Scientific Officer of Bristol-Myers Squibb, wants to reduce the R&D cycle time. What are ways to reduce the cycle time and what are the benefits to the company of doing so? Additional Information: (Provide the following information if requested by interviewee)  The company’s centralized analysis center recently hired more people (can also think about increasing capacity at the analysis center), but cycle time did not change.

The research scientists at the company’s satellite research labs heard that more people were hired at the analysis center, so they sent more drug compounds.  Number of compounds that passed the 1st review did not increase.  Scientists are starting sending low quality compounds (likelihood of going to market is very low) and queue remained long.  Scientists can usually predict the likelihood of each compound passing the 1st review.  There was no prioritization of the queue – it was first come, first serve. Possible Answer: The interviewee should come up with the idea of “prioritizing drug compounds”. 

Prioritize compounds: by using previous results or scientist’s confidence of passing review a compounds with highest likelihood of going to market should jump the queue. This discouraged scientists from sending low quality compounds. Note that the specific method of prioritization is less important than understanding the need to prioritize. Once the interviewee determines that prioritization is needed, the interviewer should provide the following information, and ask the interviewee to quantify the R&D cycle time decrease. Currently:  30% of compounds require 15 cycles  70% of compounds require 5 cycles After the prioritization:  Cycle time was 3.5 weeks per iteration, went down to 2 weeks. How many weeks on average will this save per compound? Possible Answer: (15 cycles * 30% + 5 cycles * 70%) * (3.5 – 2) weeks/cycle = 12 weeks Recommendation The interviewee should translate R&D cycle time decrease into business implications. By prioritizing drug compounds, it will save 12 weeks for an average compound. If a drug ever goes to market, 12 weeks of revenue is captured. The key issue is that revenue for a drug has a short life, based on the 15 year patent. Adding 12 weeks of revenue is a significant increase to the company’s revenue stream.

Aspen Mountain Ski Field to Build a Second Ski Lift Case Type: math problem; operations strategy, optimization. Consulting Firm: Kurt Salmon first round full time job interview. Industry Coverage: sports, leisure, recreation.

Case Interview Question #00717: Aspen Mountain is a ski area located in Pitkin County, Colorado, just outside and above the city of Aspen. It is situated on the north flank of Aspen Mountain and the higher Bell Mountain at an elevation of 11,212 ft (3418 m) just to the south of Aspen Mountain. It

was  founded in 1946 as the first ski area venture of the Aspen Skiing Company, and today it is one of four adjacent ski areas operated by the company as part of the Aspen/Snowmass complex. The Aspen Mountain ski field, with only one ski lift and one slope, has a limit of 200 skiers and is full every day. The ski lift has a capacity of 5 skiers per minute and it takes 10 minutes for the lift to carry a skier to the top of the mountain slope. Skiers take 5 minutes on average to ski down and get in the queue again. The manager of Aspen Mountain ski field has hired you to analyze how the time that skiers spend in the queue can be reduced. There are two options: (A) increase the speed of the ski lift or (B) build another equal lift. There are no budget constraints in the decision. Which of the two options would you recommend the manager pursues? Possible Answer: The recommended approach to this problem is as follows: 1. Find the time skiers spend in the line (25 minutes, see below) 2. Analyze the first option: increase the speed of the ski lift, e.g. 5 minutes. That would actually increase the queue time to 30 minutes (see below) 3. Analyze the second option: build another ski lift, that would decrease the queue time to 5 minutes (see below) 4. The mountain can hold the double of skiers if another lift is built (told only if asked) There are two methods one might take to calculate the queue time Method 1: Ski lift capacity * (Minutes in queue + Minutes on lift + Minutes skiing) = number of skiers / number of lifts 5 skiers per minute * (X + 10 + 5) mins = 200 / 1

# skiers

Before the change

Option A: increase lifting speed Option B: add a second lift

200 people

200 people

200 people

5 skiers/min

5 skiers/min

ski lift capacity 5 skiers/min

# lifts

1

1

2

mins lifting

10 mins

5 mins

10 mins

mins skiing

5 mins

5 mins

5 mins

mins queuing

25 mins

30 mins

5 mins

Method 2:

Option A: increase lifting speed

Option B: add a second lift

ski lift capacity 5 people/min

5 people/min

5 people/min

mins lifting

10 mins

5 mins

10 mins

mins skiing

5 mins

5 mins

5 mins

# skiers

200 people

200 people

200 people

Before the change

5 skiers/min * 10 mins = # skiers on lift 50 5 skiers/min * 5 mins = 25

5 skiers/min * 10 mins * 2 lifts = 100

5 skiers/min * 5 mins = # skiers skiing 25

5 skiers/min * 5 mins = 25

5 skiers/min * 5 mins * 2 lifts = 50

# skiers queuing

5 skiers/min * X mins = 125

5 skiers/min * X mins * 2 lifts = 5 skiers/min * X mins = 150 50

Total

200

200

200

queuing time

25 mins

30 mins

5 mins

The interviewee can use another hypothesis for lift time reduction in option A; but the conclusion must be the same. Conclusion: Building a new ski lift is a better option for queue time reduction. If there is time, the interviewer could ask the interviewee to brainstorm additional measures the manager might take to reduce queue time, e.g. build a cafeteria, lengthen the run, increase ticket’s price, etc.

Washington Post to Incease Migration from Print to Online Case Type: increase sales; operations strategy. Consulting Firm: Simon Kucher & Partners (SKP) first round summer internship job interview. Industry Coverage: Publishing, Mass Media & Communications; online business. Case Interview Question #00714: Your client The Washington Post (WP) is an American daily newspaper. It is the most widely circulated newspaper published in Washington, D.C., and was founded in 1877, making it the area’s oldest extant newspaper. Daily editions are printed for the District of

Columbia,  Maryland and Virginia. As a major metropolitan newspaper, Washington Post runs advertisements for employers seeking employees (job ads/help wanted ads). Recently, the newspaper has realized that to compete it must offer the ad space on its online version as well. Online, it doesn’t have traditional competitors – its competitors are sites like Linkedin.com, Dice.com, Monster.com and Craigslist.com. It is looking to you to tell it how to manage the migration from print to online. Specifically they would like to know: 1. How should they match the pace of the market as more employers go online? 2. Should they push employers to spend for online ads? 3. How can it maintain the revenue generated by printed advertising while still pushing for more online revenue? What would you recommend to Washington Post? Additional Information: (Provide the following information if requested by interviewee) Revenue from job advertising breaks down into 70 percent revenue from print ads and 30 percent revenue from online ads. Print ads are currently more profitable – $800 profit per print ad versus $400 profit per online job ad. Of jobseekers who use this newspaper (and the newspaper’s website) for job seeking, 70 percent currently use print, 30 percent use the website. Different companies are adopting online advertising at different paces: the early adopters of the website job ads are technology companies and professional service companies. The slower to adapt are government agencies, health care organizations. Possible Solution: Keep both print and online advertising as package options, especially as print is twice as profitable as online.

Target print advertising towards slower adaptors, while also developing a great site to serve clients that only want to go online (can be creative here – what would you do with the site to distinguish it from a site like monster.com?). Train sales force to plug both opportunities, but to understand the client’s needs and build a package that suit’s them best. Once the client is hooked to one option, it will be easier to sell the alternate option. Use the print vs. online packages as an opportunity to bundle advertising print and online ads.

Takeda Pharma to Delay Launch of New Drug Delivery Product Case Type: new product; operations strategy. Consulting Firm: IMS Consulting Group second round summer internship job job interview. Industry Coverage: healthcare: pharmaceutical, biotech, life sciences. Case Interview Question #00700: The client Takeda Pharmaceutical Company Ltd is the largest pharmaceutical company in Japan and Asia and one of the top 15 pharmaceutical companies in the world. The company has over 30,000 employees worldwide and achieved $16.2 billion USD in revenue

during the  2012 fiscal year. Takeda Pharmaceutical is focused on metabolic disorders, gastroenterology, neurology, inflammation, as well as oncology. Takeda Pharmaceutical has come to your consulting firm for advice on a new drug product launch that would treat a degenerative disease. A degenerative disease is a disease in which the function or structure of the affected tissues or organs will increasingly deteriorate over time, whether due to normal bodily wear or lifestyle choices such as exercise or eating habits. What factors should Takeda Pharmaceuticals consider in terms of timing the launch of a new drug product that just received FDA approval? Possible Answer: This is a qualitative strategy case with a nice twist at the end. The interviewee has to think outside the box to get to the solution. There is room for the interviewee to draw the pharmaceutical value chain during this case (added bonus). I. Suggested Approach/Framework The interviewee should think about the following areas within his or her framework.  

The Launch and Market Share Objectives of the client company The Competitive Landscape

 The Financial Implications 1. Objective of new product launch a. Gain market share, slipping market share due to undifferentiated products. b. Loyal patients have been moving to the competitor’s product; hence, we need to regain lost patients. The new launch is for a drug delivery device. c. The launch should also gain patients from the competition in order to build market share. d. Improve corporate image. 2. Competitive Landscape a. It is a $1B market, the client has 10% market share. b. A key competitor is going to launch their new device in a matter of weeks. There is unrest on further market share erosion and loss of current patients. c. There are four major competitors in the market; one with 60% share (the company with an upcoming launch) and the rest almost equally divided amongst the other three (which includes our client Takeda). 3. Financial Implications and Objectives a. The goal is to double revenue within one fiscal year. b. We are two months into the year and can potentially launch next month, but that will be a few weeks after the competition has launched anyway. c. The launch budget is $7 Million, 1/5 the competition’s budget for launch. d. The value each patient offers is $150,000 annually for adults and $60,000 annually with pediatrics. 4. Other Factors a. The positioning of the competitor’s product is along safety. Market research has indicated that this resonates most with pediatrics. b. We have 22 sales representatives in the field. c. The rest of the competition has no launch plans. II. Analysis 1. How many patients does Takeda need? Client company makes 10% of a $1B market which is $100M and needs to make another $10M within one year.  

This translates into about 67 adult patients ($100M / $150K = 667) within the fiscal year. The result is approximately 30 patients per sales representative within the year (667 / 22 = 30) – to even out the distribution you can average one patient in the weaker territories and 30 – 50 patients in the dense territories. (Please make a note that in defining what is weak/dense the upside potential of a territory should be identified, not just the hold of our client in that territory).

2. Focused targeting Physicians/Clinics – Target physicians who are more concerned with patient quality of life and ease of use (use publications and sales force input).  Distribution centers – Target distribution centers that are not in long-term contracts with the key competition and gain their loyalty.  Patient Type – Focus on young adults with a high volume upside and more inclination towards quality of life vs. safety (assuming all products meet safety standards by FDA guidelines). 3. Broadening reach to gain critical mass 

Large upside geographic centers – Research the location of clinics and consumers that have a large uptake, but a more fragmented dipping into all products – aim at gaining their loyalties.  Mid-size distribution centers – Since large distribution centers are in “grandfathered” contracts with the competition, target the mid-size centers that have been ignored and cater to their population. Try to lock annual high volume contracts.  Patient Advocacy groups III. Recommended Conclusion Timeline – the new product launch should indeed be delayed as the client Takeda is still in the first quarter and this gives them time to sharpen their strategy with us and leverage the following: 

Identify weaknesses from the competition’s launch. Allow the competition to preempt key customers and then target the unmet market. Wait until the competition has set their brand strategy and use the time to target a differentiated brand strategy.  Positioning and alignment with key customers especially while negotiating contracts with distributors and clinics (the competitor contracts would be public knowledge at this stage). There is benefit to being second-to-market, as long as a strategic plan is incorporated, inclusive of various scenarios. However, there should not be a delay over three months as this might compromise the launch uptake and time needed to gain critical mass.   

NYC Rolls Out Nation’s Largest Free Public Wi-Fi Network Case Type: new business; operations strategy. Consulting Firm: PwC Diamond Advisory first round summer internship job interview. Industry Coverage: government & public sector; information technology (IT). Case Interview Question #00694: Your client is the office of New York City Mayor Michael Bloomberg. Born February 14, 1942, Bloomberg served as the 108th Mayor of New York City, holding office for three consecutive terms (January 1, 2002 – December 31, 2013) beginning with his first election in 2001. He is

the founder and 88% owner  of Bloomberg L.P., the global financial data and media company notable for its Bloomberg Terminal. Mayor Bloomberg has asked you to come to NYC as a Diamond consultant and advise him on whether public Wi-Fi should be accessible to all of New York City free of charge. The Mayor has looked at several studies and has seen some research on the economic impact. What would you consider and investigate in other to determine whether Mayor Bloomberg should or should not take this decision? Possible Answer: This strategy case is focused on developing an approach to the question. No number calculations are necessary in this case. The case is broken into various levels of consideration: Level 1 – Would this implementation be useful to the NYC population? Analyze the number of people who current have and do not have access to the Internet. Of those who do not have Internet access, identify the reasons for lack of access (two main groups) Supply side: People want access, but currently do not have it due to a number of factors: service and geographic gaps, price, slow/subpar access speeds  Demand side: People do not want access because the do not have a computer or do not consider it valuable (think certain age demographics) Level 2 – How would this implementation affect constituents that currently provide Internet access forprofit? 

Identify affected players in the market (e.g. Comcast, AT&T, Time Warner Cable, FIOS, etc.) Identify repercussions that will be faced by these businesses by identifying the populations they serve. Level 3 – Estimate the revenue loss of affected players by sizing the population that currently subscribes to an Internet connection.  

 Households  Businesses  Academic Institutions: Schools and Universities The candidate should understand that some of these may benefit from free public Internet access. How do you justify a decision to provide free Internet access to the current providers? Level 4 – Identify advantages and caveats of the decision.

What positive impact will come from this decision? How do we know? Benchmark other cities that have implemented this model to determine the upside.  Why Wi-Fi? Is there better technology out there?  Are there other options worth investigating? E.g. DSL, cable, subsidizing Internet access to pockets of the population, etc.  How do we mitigate the risk that we invest significant capital for implementation, but meanwhile technology evolved and our investment is less valuable?  Are there alternative ways to provide Internet access: Kiosks, Internet cafes, increased public library Internet access  Additional Issues: security, privacy, speed Level 5 – Implementation 

  

How much will it cost? Who will bear the cost? Will the constituents agree to bear the cost?

Whirlpool to Re-focus on End Consumer and Product Quality Case Type: improve profitability; operations strategy Consulting Firm: Siemens Management Consulting (SMC) 2nd round full time job interview. Industry Coverage: consumer electronics; household goods. Case Interview Question #00691: Our client is Whirlpool Corporation (NYSE: WHR), an American multinational manufacturer and marketer of home appliances headquartered in Benton Charter Township, Michigan, United States. The company is listed in Fortune 500 and has annual revenue of

approximately  $20 billion, more than 70,000 employees, and more than 70 manufacturing and technology research centers around the world. The client Whirlpool is seeking to realign its business strategy in order to maximize profit potential. The firm has traditionally focused on internal processes to drive profits, but recent industry trends and competitive shifts are forcing key executives to reevaluate the company’s competitive strategy. The client has asked you to make recommendations to that end. What would you tell the CEO of Whirlpool? Additional Information: (Provide the following information to candidates if requested) 1. Company a. The company Whirlpool Corporation sells a broad range of consumer appliances (ovens, refrigerators, air conditioners,vacuum cleaners, etc.) and accessories through two major outlets: major retail outlets such as Best Buy, Wal-Mart, Target, Home Depot, IKEA, etc, and online. The breakdown is 80% of sales through major retail outlets and 20% online.

Key Insight: $16 billion of the client’s sales are through brick-and-mortar retailers which gives those retailers an enormous amount of bargaining power (High Customer Power if using Porters’ Five Forces). b. The firm has traditionally used cost-efficiencies to drive profitability, and as a result has not focused on organic growth initiatives (revenue enhancements). Therefore many functional areas have become siloed and myopic, attempting to maximize functional profits rather than company profits. The interviewee should ask about the major functional areas, and which are the profit centers. Cost Centers:  Marketing  Research and development (R&D)  Manufacturing  Call centers Profit Centers:  Product Sales  Warranty and Service Contract Sales  Appliance Parts (for repairs, upgrades, etc.) c. The client has seen decreasing product sales, in large part due to consumer dissatisfaction with product and service quality. Executives believe that the blind pursuit of functional profits has driven focus away from delivering highly-valued consumer products and service offerings to the end-consumer. Key Insight: The client has maximized profitability from the cost-side, and now needs to search for ways to grow revenues – siloed functional areas means integrated initiatives have not been utilized and that profitpursuit at the functional level may actually destroy downstream value, preventing optimal profit realization for the company as a whole. 2. Competition The three major players in the industry, including the client, account for approximately 80% of all sales (the interviewee should inquire about the competitive shifts mentioned in the opening of the case). The #1 and #3 players have recently merged and will now account for a combined 50% of all sales. Our client Whirlpool is the #2 player. Key Insight: The interviewee should recognize that the client’s market share is therefore 30% (and that the industry as a whole is therefore about $66 billion – because 30% of the total market equals $20 billion).

The merger of the two major competitors will give that new company significant bargaining power with the major retailer outlets in comparison to the client. It is therefore likely that the client’s retail outlet sales will continue to decline. (High Competitive Rivalry if Using Porter’s Five Forces) 3. Consumers The client Whirlpool draws a distinction between the following two groups:  Customers are the major retailers who purchase the client’s products for resale.  Consumers are individuals that purchase products through retailers or online. Key Insight: The end consumer is not the direct profit-maker for the client – as a result, the client has traditionally not focused on the needs of the end consumers and have seen significant drops in consumer satisfaction with regard to the entire product experience (from researching the purchase, to the buying process, to warranty management to repair service). Possible Answer: The main crux of this case is how to deal with increased competitive rivalry in a commodities business that is in large part at the whim of the powerful retailing outlets that purchase their products for resale to the end-consumer. 1. The client Whirlpool should re-focus resources to increase selling through the online space to mitigate the increased competitive and customer power. However, have the interviewee think through the risks associated with such a strategy. Potential Risks: Retailers may pull the brand from their shelves if the client places too much emphasis on direct-toconsumer sales by sidestepping the retail channel. Can the client afford to lose the retail play completely given a current 80% sales level? Many end-consumers may not be comfortable purchasing online or would prefer to comparison shop instore before deciding on a major purchase. Is there a natural ceiling to potential online sales? What additional costs will the client need to incur to increase direct sales? Does it have the internal capabilities necessary for such a play? (shipping, logistics, technology, human resources, etc.) 2. The client Whirlpool needs to focus on realigning internal processes, metrics and goals to maximize company profit rather than functional profit. This will require a renewed focus on the end-consumer and on product quality. In essence, the current competitive and customer situation dictates that the model of pushing product through retailers may not work for the client, as the newly merged competitor will be able to offer better deals to retailers and take over floor space. By focusing on the end-user the client can own

the “pull” for its products, and product demand will force the retailers to work cooperatively with the client and stave off competitive threats.

Direct Relief to Increase Donation Made by General Public Case Type: operations strategy, optimization; organizational behavior. Consulting Firm: Ernst & Young (EY) second round full time job interview. Industry Coverage: non-profit organization; healthcare: hospital & medical. Case Interview Question #00675: Our consulting firm is doing a pro-bono case for Direct Relief International, the largest non-profit medicine distribution network in the U.S. Founded in 1948, this organization focuses on improving the quality of life by bringing critically needed medicines and supplies

to local healthcare providers  worldwide. Based in Goleta, California, the client Direct Relief’s network consists of 50 Medicine Banks that distribute drugs to more than 30,000 charitable Agencies, including homeless shelters and community hospitals. Medicine Banks are warehouses that store drugs received from donations and distribute them to Agencies, for example, not-for-profit hospitals. Each year, Direct Relief’s network secures and distributes more than 2 million boxes of drugs, that in turn are provided to more than 700,000 Americans in need annually, like working poor, elderly, homeless, etc. Drugs are sourced from big pharmaceutical companies, and also from other corporations and the public in general, for instance, liquid asset and in-kind donations. The flow chart below shows the simplified network structure of our client Direct Relief.

Donors –> Pharma companies corporations medicine drives public in general

Agencies – Client’s network –> >

Medicine banks

Hospitals shelters others

End users

Homeless working poor others

In a talk with the client’s President and CEO, and the Senior Vice President (SVP) of Strategy, two main concerns were raised. Those will be the focus of our team’s activities: 1. Although the organization has significantly increased its output over the years to End Users (in terms of the number of boxes of drugs), reported need of drugs by Hospitals and Community Service Agencies continue to escalate. Why?

2. What could Direct Relief do to improve the situation? Note: This is a long introduction for a case. The interviewer or case giver should give the candidate enough time to take notes and understand the questions. Additional Information: (to be given if asked by candidate) 1. Our client Direct Relief Main Activities: Medicine Sourcing: Forge/maintain relationship with donors, acquire supplemental medicine using financial donations, centralize donations, distribute donations to members through its own fleet and transportation companies in-kind donations.  Public Education and Advocacy: Research and studies, lobbying, public relations Donation Trends: 

The number of boxes of drugs donated to Direct Relief has been increasing for the last 5 years with a CAGR (compound annual growth rate) of 7.2%  Liquid asset donations and In-Kind donations (services such as transportation) have also been increasing with a CAGR of 10.1% in the same period  Government programs dedicated to sourcing medicine to the needy have also increased their budget with a CAGR of 6.7% Demand Trends: 

 Total need for medicine to the needy has been growing at the rate of GDP for the last years. 2. Medicine Banks Medicine Banks are non-profit organizations that centralize and distribute drugs to Agencies. Those Medicine Banks are members of Direct Relief’s network, but Direct Relief does not control them because they are separate non-profit organizations.  Direct Relief’s network currently has one Medicine Bank in each of the 50 USA states.  Although Direct Relief is responsible for a significant portion of the total volume of drugs distributed by Medicine Banks, governmental programs and local donations also significant sources (65% and 25% respectively). 3. End-Users  

Recently (Year 2006) the client Direct Relief surveyed end-users’ demographics for the second time in its history (first survey was in 2001). Some of the results are the following (show Exhibits 1-3):

Exhibit 1. End-User Household Income

Exhibit 2. End-User Age

Exhibit 3. End-User Education

(Note: Do not volunteer this information unless the candidate asks about the change in demographics of End-Users) In interviews with Agencies, it was discovered that processes to evaluate End-User eligibility to free medicines were banished due to the additional operational complexities that were perceived by many as unnecessary.  It was also verified that some hospitals were using medicine sourced by Direct Relief’s network to fill regular inventory gaps. 4. Food Sourcing and Geographic Data 

Criteria for Allocation of Medicine to Medicine Banks The current criteria to allocate drugs to Medicine Banks (and consequently states) is based on the number of boxes of drugs distributed to end-users in the previous month by that state/Medicine Bank Exhibit 4. Top and Bottom 10 states by drugs sourced by the client (adjusted to eliminate differences in state population sizes) 

Top 10 States Bottom 10 States NY

NV

NC

WI

IL

UT

FL

PA

AK

CO

AZ

MO

KS

ND

MS

VT

SD

TX

CA

OH

Exhibit 5. Need of Medicine by State

Prompt: If the candidate does not draw any conclusion from this chart, ask him/her which quadrant should receive more attention from Direct Relief. Possible Answer: The client Direct Relief’s drug allocation method to Medicine Banks is not adequate, as evidenced by the information that states in most need are also in many cases members of the bottom 10 states in terms of boxes of drugs received from Direct Relief. The candidate might assume that since the top 10 states are performing well and the bottom 10 states are not, then Direct Relief’s donations are tilting the balance and influencing how states fare. That concept can be dismissed once the candidate finds out that Direct Relief sources only 10% of all donations to medicine banks.

A suggested allocation method would be to evaluate the state’s total demand for drugs and its trend, and supply according to that need. Also, since agencies eliminated end-user screening processes, more medicine is being directed to endusers who don’t need that support (which can be seen in the change in end-user demographics) A new screening process should be instituted to guarantee that agencies are not overburden with the new process, thus directing drugs to the needy Finally, the client Direct Relief should implement inspections at hospitals to make sure that source medicine is not being used to fill regular inventory gaps and impose sanctions to hospitals that adopt such practice. An outstanding candidate would evaluate that since Direct Relief is only responsible for 10% of donations to medicine banks, Direct Relief should focus in public education and advocacy targeting to increase donations made by the general public and to increase funds from government programs. Lobbying could play a much larger role and should be one of the uses of liquid assets available to Direct Relief.

Boehringer Ingelheim to Optimize Current R&D Portfolio Case Type: organizational behavior; operations strategy. Consulting Firm: Siemens Management Consulting (SMC) first round summer internship job interview. Industry Coverage: healthcare: pharmaceutical, biotech, life sciences. Case Interview Question #00670: You are part of our management consulting team working for German pharmaceutical company Boehringer Ingelheim. Founded in 1885, the Boehringer Ingelheim group now is one of the world’s top 20 leading pharmaceutical companies. Headquartered in Ingelheim, Germany, it

operates globally with  145 affiliates and more than 44,000 employees. The company’s key assets of interest are: respiratory disease, cardiovascular disease, Parkinson’s disease, HIV, thromboembolic disease, cerebrovascular disease, oncology, diabetes and hepatitis. Since it was founded in 1885, the Boehringer Ingelheim company has been committed to researching, developing, manufacturing and marketing novel products of high therapeutic value for human and veterinary medicine. We’ve been retained by the CEO of Boehringer Ingelheim to look at their current research and development (R&D) portfolio and assess whether they’re maximizing their potential with their current projects. Specifically, the CEO’s questions to you are:  How can you help them decide what information you would need?

What components do you think would round out an R&D portfolio for Boehringer Ingelheim pharmaceutical company? Additional Information: (to be given to candidate if requested)  Current staffing needs at Boehringer Ingelheim are at equilibrium.  Layoffs and hiring are not expected to be necessary. Note to the Interviewer: Remember to make the interviewees brainstorm. You want them to come up with categories for this pharmaceutical company to consider when developing their R&D portfolio. For any questions, make sure you keep them on track with this goal. They should be naming different areas to look at, i.e. investing in current projects versus investing in creating new technology or starting new business. Possible Answer: For this case, a successful interview would follow this “logical path”: 

Develop a small framework from which they follow. Potential framework categories include Revenues and Expenses, Regulatory Environment, Internal Issues, etc;  Start asking appropriate questions based on this road map. Example: “I would really like to look at the current state of finances for the client company. There is the possibility that for expense reasons, they may not be able to heavily invest in a lot of R&D Projects.”  When prompted, conclusion should be short and sweet, like “In my opinion, this pharmaceutical company should really focus on 4 different categories. These are large investment versus short investment, longer projects versus shorter projects, new business versus existing business, and the mix over different business units. These will allow it to maximize its spending investment and reduce their risk by not being overly invested in one area.”  The 2×2 matrix should follow. It doesn’t matter which areas they pick to compare but the math should be easy. If there is a problem with the math, an easy prompt can be given without too many “points” being taken off. The client company should be considering looking at multiple areas of its business such as: 

 whether the R&D is for new business or current business  new technologies versus current technologies  spread of R&D projects across business units  big projects versus small projects  high dollar investment versus low dollar investment  short duration projects versus long duration projects There may be a few others but these listed above are the pertinent ones. Note to Interviewer: When the interviewee has provided their conclusion for the areas they think the pharmaceutical company should be looking at for a successful mix, please ask them to pick two and create a sample 2×2 diagram for you. Ask them: “If you were to look at these two areas, how would you suggest breaking up the expenditure.” See Exhibit 1 below for the appropriate 2×2 graph and sample calculations.

If asked how much the R&D expenditure is annually, answer with $100 million, but this information is not relevant to calculations and they can be done using percentages only. Exhibit 1: Sample 2×2 matrix to break up R&D expenditure

Capital Requirement of Investment

Duration of Projects Long 40%

Short 60%

High 40%

Total % of All Projects: 40% * 40% Total % of All Projects: 60% * 40% = 16% = 24%

Low 60%

Total % of All Projects: 40% * 60% Total % of All Projects: 60% * 60% = 24% = 36%

The main point to make sure the interviewee gets correct is that all four boxes together should add up to 100% and that each of the columns and rows should equal the percentage allocated for that particular category, i.e. 40% for long term projects or 60% for low capital investment projects.

Software Firm VMware to Set Up Engineering Unit in India Case Type: operations strategy, outsourcing; market entry. Consulting Firm: Gallup Consulting 2nd round full time job interview. Industry Coverage: software, information technology (IT). Case Interview Question #00657: The client VMware, Inc. (NYSE: VMW) is an American software company that provides cloud and virtualization software and services to both personal and corporate customers. The company’s desktop software runs on Microsoft Windows, Linux, and Mac OS X, while

VMware’s enterprise software hypervisors run directly on server hardware without requiring an additional underlying operating system. VMware has its headquarters in Palo Alto, California, United States, and established an R&D Center in Cambridge, Massachusetts, as well as one at the Time Warner Center in New York City. Recently, the Chairman of VMware wants to offshore/outsource its engineering and designing unit to India, as well as to penetrate into the India software engineering market. Should they do it? Why or why not? Additional Information: (to be given to candidate upon request) The candidate is supposed to figure out the following information by asking appropriate questions. The interviewer is supposed to read this information well before he/she gives the case. The interview style should be pleasant but reserved.

1. Market Share – in the US the client VMware is the industry leader in cloud service and virtualization software with several close followers chasing behind. 2. Profitability – profitability is declining (unknown reason, but increasing labor costs can be a reasonable assumption). 3. Capability – VMware has strong engineering department in the US. 4. Cost – R&D is the major cost and Indian software engineers are estimated to be only 1/4 of the cost of the US engineers with the same technical capability. 5. Customer The client VMware has a strong existing customer base in the US. Customers care about the quality of service, but are also considering lowering cost in the long run. 6. Competitions  

Competition – US:  Key US competitors (Microsoft, IBM, Oracle, HP, Google, etc) are all off-shoring in order to lower the R&D cost.  Competition – International:  The growth of the international market is impressive compared to the more mature and stable US market.  Key competitors are expanding their international business aggressively.  India is one of the fastest growing international markets as well as the one with the largest market size. Possible Solution: Since the goal of this “market entry” case is two-fold, a good candidate will begin with laying out a clear scope and then gather relevant information to analyze the situation. A decision matrix can be created to guide the analysis. 

Off-shoring is critical to access local market? Other benefits of offshoring Yes

No

Significant

Definitely offshore

Offshore but also find out other critical factors to enter the India market

Minor

Evaluate the India market independently with market, customer, competition, etc.

Stay in current market and strengthen competitiveness

Once the candidate has gathered the necessary information regarding VMware’s customers, a strong candidate should follow up with additional questions about Customer Segmentation. It turns out most of VMware’s clients are medium to large companies in the US. The most profitable clients are large companies in developed countries where VMware already has a strong base.  Currently, VMware doesn’t have any international presence yet. At this point, a strong candidate will get the hint that entering the international market, especially the India market, is critical for the client company to both fulfill current customer’s emerging needs of cost saving and grow its future business. The candidate should then start to compare the competitive advantages between large US companies off-shoring and local Indian players. 

US companies

Indian players

Ability to fulfill customized needs

strong

medium

High quality services

strong

medium

Access to most profitable clients

strong

weak

Cost advantage

weak

strong

Local India market knowledge

weak

strong

Local client / government / supply chain relationships weak

strong

Low legal risks

strong

strong

An outstanding candidate will summarize his/her findings and make a final “Go / No go” recommendation. Conclusion Based on the above analysis, the short answer is “Yes”, the client VMware should outsource its engineering/designing unit to India. The situation falls into the upper left corner of the matrix because: 





In the long-run, even current customers with established relationship will need to look for cheaper alternatives. VMware can offshore its R&D to lower the cost but still keeps its customer service team in the US to maintain the high service quality. Although clients in the developed countries are more profitable, the actual growth of the market is limited. Developing markets like India might not be as profitable as the US, but with the huge and growing market size, even capturing a small percentage of the market can provide substantial profits. VMware might lack knowledge of the Indian market, but its strong customer relationship management skills, large existing customer bass, and the understating of unique customer needs can



be further leveraged in India. In addition, hiring local talent or partnering with local companies can help solve the concern of the lack of local knowledge. The legal risk as well as the political risk in India can be considered very low.

Warner Bros. to Do Simultaneous Global Release for Batman 3 Case Type: operations strategy. Consulting Firm: NERA Economic Consulting first round full time job interview. Industry Coverage: entertainment; mass media. Case Interview Question #00651: The client Warner Bros. Studios is an American producer of film, television, and music entertainment. The company is a subsidiary of media and entertainment conglomerate Time Warner (NYSE: TWX), with its headquarters in Burbank, California and New York

City. As one of the  major film studios, Warner Bros. develops and distributes full-length feature films for the U.S. and international markets. Warner Bros. Studios is planning on releasing its big budget film of the year six months from now and is considering a new release strategy. The first component of the strategy is to release the movie in theaters on the same day around the world. The second component of the strategy is to release the DVD of the movie day-and-date, meaning globally releasing the DVD the same day as the theater release. Warner Bros. is interested in knowing whether this is the best release strategy for this movie and, if not, Warner Bros. would then like to know what its release strategy should be. How would you go about analyzing this case and helping the client make a decision? Additional Information provided upon request:  Warner Bros. has done simultaneous global theater releases for other movies before, but never simultaneous with the DVD release.  Only one movie has ever been simultaneously released theatrically and through DVD, and it was by a small, independent film company.  The new film, “The Dark Knight Rises”, is the third movie of a superhero action series known as the “Batman Trilogy”.The previous two films were produced by another studio, so we have no financial data on them.  The simultaneous theatrical and DVD release is expected to cause a 20% decrease in revenue from theater ticket sales and a 25% increase in DVD sales.  The marketing campaign for the movie is expected to be $50 million, while the marketing campaign for the DVD is expected to be $5 million. With the simultaneous release, no DVD marketing campaign is required.  Pirating movies accounts for a 20% loss in total theater revenue. A simultaneous global theater release will cut this down by 25%.

Additional Exhibits 1. Revenue from theatrical releases of big budget films (last year)

Category

Number of films Gross revenue

Blockbuste r 4

$500M

Success

10

$250M

Breakeven

15

$100M

Failure

8

$50M

2. Revenue from theatrical releases of third sequels of action films (last 5 years)

Category

Number of films Gross revenue

Blockbuste r 14

$500M

Success

3

$250M

Breakeven

2

$100M

Failure

1

$50M

3. Revenue breakdown of big budget films Possible Solution: The main question to answer is if the new strategy Warner Bros. is considering is better than the usual movie release strategy commonly used in the movie industry. To assess this we need to see which of the two release strategies brings more profits to the client Warner Bros. Regular Strategy: We need to estimate the revenues that could be generated and costs involved with this type of release (which is from my knowledge: theater release in the US followed by theater release internationally, then DVD release internationally). In order to estimate the expected revenues, I would look at similar type of movies and their economics.  New strategy: I would start by checking if this new strategy has ever been used and the results it brought. Then I would estimate the changes in revenues, costs of this new strategy versus the regular one. 

Finally, I believe we need to check if there are any regulation regarding international theater release and how these regulations will affect our new strategy.

Essentially, it comes down to deciding between two options on two dimensions:  Releasing the movie in the same day globally or first in the US and then around the world  Releasing the DVD together with the movie or separately The candidate should then calculate the expected revenue from the theater release and from the DVD release using second and third Exhibits and other information that he/she gathered. 

Estimated theater revenue = ($500M∗14 + $250M∗3 + $100M∗2 + $50M∗1) / (14 + 3 + 2 + 1) =

$400 million  Estimated DVD revenue = $400M * 30% / 60% = $200 million To decide between going simultaneously with the theater and DVD release versus going separately, the candidate need to calculate the profits generated by the two strategies.

Release strategy

Simultaneous

Separate

Theater revenue

$400M * 80% = $320M

$400M

DVD revenue

$200M*125% = $250M

$200M

Marketing cost

$50M

$55M

$320 + $250 – $50 = Profit (before other common costs) $520M

$400 + $200 – $55 = $545M

To decide if the client should go separately for the theater release or simultaneously, the candidate needs to calculate the impact on piracy too. $400M = revenue earned (80% of total possible revenue) Unearned revenues (losses due to piracy) in the regular release strategy (theater release in the US followed by theater release internationally): $400M / 80% – $400M = $100M  Unearned revenues (losses due to piracy) in the new release strategy (simultaneous global theater release): $100M * (1 – 25%) = $75M A good candidate will now make a Final Recommendation I recommend that the client Warner Bros. should do a simultaneous global theater release but a delayed DVD release because:  

 Delayed DVD release would generate $25M additional revenue  Simultaneous global theater release will save $25M by decreasing piracy However, there are some risks involved with this strategy. With global release we cannot reduce spending in case the movie fails in the US. There are also risks regarding the problems that may be caused by governments impeding or delaying the release in their respective countries. We need to investigate further these aspects before going forward with the plan.

Newmont Mining Corporation to Exploit Gold Mine in Peru Case Type: math problem; operations strategy. Consulting Firm: Schlumberger Business Consulting (SBC) first round summer internship job interview. Industry Coverage: mining, metals production. Case Interview Question #00641: The client Newmont Mining Corporation (NYSE: NEM, TSX: NMC) is a gold mining company based in Greenwood Village, Colorado, USA. The Company’s operating segments include North America, South America, Asia Pacific and Africa. As one of the world’s

largest  producers of gold, Newmont has active mines in Nevada, Indonesia, Australia, New Zealand, Ghana and Peru. As of December, 2010, Newmont produced approximately 5.4 million equity ounces of gold annually. Other metals that the company mines include copper and silver. Recently, Newmont is going to exploit a mine in Peru that is expected to be full of a very unique gold ore. The CEO of Newmont has asked us in which order his mining team should extract the gold ore from the field. The gold ore will then be sent to a nearby factory that his company already owns. How would you go about analyzing the case and coming up with a solution? Additional Information: (to be given to candidate if requested)  The field is divided in cells of equal size. Once you blow one cell there is no way the truck can pass through it to go to the factory.  Each cell has different gold ore content and therefore each cell has a different cost of extraction.  Trucks enter the factory with rocks containing 1% of gold ore. The output is 100% gold ore.  For sake of simplicity, assume there will be no growth in population. Below is a plan of the gold mining plant in Peru:

Recommended Approach: The interviewer is testing whether the candidate can identify a useful algorithm to determine the best way to extract gold ore from this field. The candidate should not initially waste any time developing one idea

too fully; instead he/she should come up with a few reasonable suggestions and test them for validity with the interviewer. The candidate should, like in most cases, be aware that he/she is missing critical information and should identify what he/she needs to solve the case and then ask the interviewer for this data. Possible Solution: Interviewer: So, tell me how we might approach this problem for our client? Candidate: First, I would like to make sure that I fully understand our client’s needs. Let me repeat the key issues: our client wants us to provide him with a recommendation on what approach his mining team should take to extract the gold ore in order to maximize profits. Interviewer: Yes, that pretty much sums it up. So, what’s next? Candidate: I can imagine a number of different ways of prioritizing which cells to take out first, such as: 1. Profit: Profits = Revenues – Costs = Price * Volume – Costs. 2. Demand: Outlook of the price of the different types of gold ore. If the price is low (perhaps because demand today is lower), but we believe that this same ore will be sold for much more in the future (perhaps due to an increase in demand), it could make sense to begin extracting lower concentration cells. I am imagining that this could be similar to the oil industry. Think for example about the price of oil. The oil is in the field anyway, so it’s better to exploit the places with the least of oil when the oil price in the market is at a low level. Unless, of course, we can store the gold ore for long periods of time without a large opportunity cost by tying up capital and space. 3. Capacity of the plant: Maybe there is a minimum level we need to provide to the plant as input. Maybe there is a maximum. This could determine how much we want to extract in any given time period, especially, if there are penalties for operating outside of this min-max range. 4. Operational issues: For example, if I dig a hole in the ground and the truck cannot pass through that cell, it may be wise to start at the cells that are further from the factory so that I do not block off routes. Interviewer: OK, let’s assume that the points that you have raised are all reasonable. This is a complex business and I was pleased to hear you touch on many of the important points. But, first, let’s focus on profits. How would you define profit? Candidate: OK, in this case profits would be (price of a ton of gold ore) * (expected content of ore in cell) – (the cost of extraction for that cell). I will then rank each of the cells based on its overall profit contribution and determine my extraction path from there. Of course, we need to consider that if we

extract from one highly profitable cell, but it cuts off access to a lesser, but still profitable, cell, this is a cost of doing business under this assumption and should reduce my overall expected profits. Interviewer: Let’s assume that we map the field and this is what we get:

x = tons of gold ore in that cell C = cost of extracting in that cell in thousand dollar Candidate: Do we have any information on today’s price of the gold ore? Interviewer: Yes, our client expects to get $10,000 dollars per ton of gold ore. Candidate:  For Cell 1 the profit will be: 1 * $10,000 – $3,000 = $7,000.  For Cell 2 would be: 1.3 * $10,000 – $10,000 = $3,000.  For Cell 3 would be: 1.1 * $10,000 – $5,000 = $6,000. So I will start for Cell 1, then Cell 3 and finally exploit Cell 2. Interviewer: OK. What if price changes? Does it change your decision? Candidate: Yes. Interviewer: Why?

Candidate: Let’s say price is now $30,000 per ton.  For Cell 1 the profit will be: 1 * $30,000 – $3,000 = $27,000.  For Cell 2: 1.3 * $30,000 – $10,000 = $29,000.  For Cell 3: 1.1 * $30,000 – $5,000 = $28,000. So I would recommend extracting Cell 2, then Cell 3 and finally Cell 1. This is based on the fact that the costs appear to be fixed and do not change despite the overall price fluctuations that we are discussing. Interviewer: Let’s move on. Our client has told us that per ton Profit and Loss (P&L) statement is as follows…wait, perhaps you could tell me how you would construct it? Candidate: I would try to find out the average price, the average cost of goods sold (COGS) and then assign some part of the SG&A (Selling, General and Administrative Expenses) costs. Interviewer: OK. Here are the figures that our client provided. What is the company’s break-even point?

Item

$1,000 per ton

Revenue s 1.0 COGS

0.8

SG&A

0.6

Profit

-0.4

Candidate: OK. The company has $200 gross margin per ton of gold ore, before we factor in SG&A costs. So, if I divide SG&A by the gross margin I will find how much they have to produce. In this case, $0.6 / $0.2 = 3, which means they have to produce 3 times more to cover fixed costs. Interviewer: How the same P&L would look like? Candidate: Basically, we have to divide the SG&A by 3. We are in essence spreading the fixed SG&A costs across more units. However, we would need to make sure that SG&A does not increase with volume of gold ore extracted.

Item

$1,000 per ton

Revenue s 1.0 COGS

0.8

SG&A

0.2

Profit

0.0

Interviewer: Good, can you summarize what we have identified so far? Candidate: Sure. Our client was looking for a framework to identify the order in which he should extract cells of ore from a field. We chose to focus on profitability as the main driver. As mentioned, but not discussed in depth, profitability includes the core components of price per ton of ore, cost to extract the cell (which appears to be fixed), but also the cost of not being able to access a cell for extraction. Once we have calculated the profits per cell our client can draw out a map to understand which cells to extract and when in order to maximize his P&L. Next we discussed the company’s P&L and identified that SF&A costs are relatively significant and appear to be fixed. This means that we would want to extract more ore in order to spread these costs across production and increase overall profitability. Interviewer: Great. Thank you.

Cisco Closes Minnesota Plant to Remove Excess Capacity Case Type: operations strategy. Consulting Firm: Siemens Management Consulting second round summer internship job interview. Industry Coverage: telecommunications & network. Case Interview Question #00636: Our client Cisco Systems, Inc. (NASDAQ: CSCO) is a global multinational telecommunications company headquartered in San Jose, California, United States. Cisco designs, manufactures, and sells telecommunications infrastructure and networking equipment. The

company’s current portfolio of products and services is focused  upon three market segments – Enterprise and Service Provider, Small Business and the Home. The management of Cisco have hired us to help determine if they have any excess manufacturing capacity. If there is any excess capacity, our client would like Siemens Management Consulting to suggest a course of action to ensure they are profitable and efficient. How would you go about this case? Additional Information: (to be given to candidate if requested) The interviewer does not offer the necessary data and so the interviewee should ask directed questions to extract the following data.

The client Cisco Systems currently has plants in both Minnesota and Mexico, close to the Texas border. The plants manufacture similar products and the facilities are about the same size. The company expanded capacity by replicating their first plant (using it as a template for others). Each plant operates an 8am – 5pm shift Industry wide demand is flat or declining The client Cisco has a relatively high market share of 80% Customers are mostly phone companies and there are very high switching costs Our client recently acquired a competitor and there are very few players in the market right now. The client is not highly concerned about new entrants because the industry growth is flat or declining, this is not an attractive market. Possible Solution: The candidate should formulate a logical structure (internal vs. external factors, supply/demand, etc.) and then determine if the client has excess capacity or not.      

Candidate: I would start by looking into the company’s manufacturing process to determine if they are experiencing low utilization rates compared to the overall capacity of production. Interviewer: The company operates an 8am – 5pm shift and is running at fairly close to 100% utilization rate. However, they have considered shifting to multiple shifts in each plant and running a 22-hour workday. What do you think the overall demand capacity looks like in this particular industry? Candidate: I would assume that it is either flat or declining because the US telecommunication market is relatively saturated, with little new customer growth. Interviewer: Yes, that’s correct. Candidate: Ok, well, I would now like to investigate the overall market and determine where our client sits in the industry. Do we have any information about their market share? Interviewer: They have a relatively high market share of 80%. Candidate: Who are they supplying the products to? Consumers, business customers, or phone companies? Interviewer: They are a telecommunication infrastructure manufacturer, so their direct clients are the phone companies. Candidate: Does our client have any reason to believe that the market share could change dramatically? What are the switching costs in this industry?

Interviewer: There are very high switching costs in the industry. Also, our client recently acquired a competitor so they are not anticipating any big declines in market share. There are also only a few competitors in the industry with little threat of new entrants due to the flat or declining growth. Candidate: Given the overall macro trends in this industry as well as their specific manufacturing set-up, I believe that our client does have excess capacity? Interviewer: Why? Candidate: Well, the industry is declining and there is less demand for these products over time. Also, opportunities exist to streamline their manufacturing operations with more shifts per plant across less overall plants. Interviewer: OK. What should they do? Candidate: They could close some existing plants, or they could generate revenues from manufacturing products for competitors (or for companies with similar products). Interviewer: Good. Looking at the first option, how would you determine which plants to close? Candidate: I would look at the overall cost per unit at each plant and determine which would result in the most efficient overall location. Costs could be classified into major subsets such as labor, distribution (from Mexico vs. Minnesota). I would also want to know if quality of product differs across any of the plants. Finally, it would be valuable to analyze whether our client could change their plant size and configuration to generate further efficiencies. Interviewer: Good. Quality is fairly consistent across all of the plants and they are not looking into plant reconfiguration at this point. Labor costs are the most significant contribution to cost per unit. Given that, what do you recommend? Candidate: I recommend our client closes the Minnesota plants and increases overall utilization of the plants in Mexico by adding shifts. Interviewer: What are the major risks/concerns of this recommendation? Candidate:    

Negative PR resulting from closing US plants Scalability issues, e.g. can the Mexican plants quickly increase capacity? Quality issues stemming from increased production Reliance on one location/region for all manufacturing

Cummins To Reduce Warranty Claims for Diesel Engines Case Type: new product; operations strategy, supply chain optimization. Consulting Firm: Siemens Management Consulting first round summer internship job interview. Industry Coverage: industrial equipment. Case Interview Question #00634: The client Cummins Inc. (NYSE: CMI) is a Fortune 500 corporation that designs, builds, distributes and services engines and related technologies. Headquartered in Columbus, Indiana, United States, Cummins sells in approximately 190 countries and territories

through  a network of company owned and independent distributors and dealers. The company manufactures and markets a complete line of diesel and natural gas-powered engines for on-highway and off-highway use. Its markets include heavy and medium-duty truck, bus, recreational vehicle (RV), light-duty automotive and a number of industrial uses. Recently, Cummins has designed a new diesel engine with significant new technology and significant new content. It will meet all federal guidelines for at least 10 years. The client has spent significant research and development (R&D) money on this new engine and expects to be able to charge a premium price to recoup the investment. However, the client’s largest customer, representing 60% of all sales, has been complaining about the quality of the new diesel engine. Our preliminary research has shown that although our client Cummins promised their customers in terms of soot emissions less than 3,000 ppm, but they are currently seeing 20,000 ppm (1 ppm = 1 parts per million). What is wrong and what should the client do to rectify the situation? Additional Information: (to be given to candidate upon request) 1. Product Design The design process was unchanged for this new diesel engine. All testing was done the same as with historical products. However, the FMEA analysis (Failure, Modes, and Effects Analysis, used to identify and assess the ramifications of possible product defects) was skimped on. The significant new technological components of the diesel engine were developed according to the client’s standard process. 2. Manufacturing Cummins’ manufacturing plant uses a highly skilled and educated workforce. There is no learning curve associated with the assembly of this new product. 3. Service

The client Cummins utilizes field technicians at the customer site. They are the ones reporting the warranty data to the client. They are all well educated on the product. 4. Sub-assembly procurement Traditionally, the client Cummins used few suppliers who delivered large sub-assemblies. Now, in an effort to save costs, they have sourced many more suppliers to get the best price for each component and are choosing to do more of the assembly themselves. However, all of these suppliers are delivering products that meet the specifications delivered by engineering. 5. Defect Details: See the chart below detailing the frequency of top warranty issues (Figure 1).

6. Customers Customers are concerned mostly regarding the quality of emissions. Possible Answer: Interviewer: So, how would you go about this case? Candidate: OK, so in order to identify the problem we need to assess all the stages that the new product is going through from the design to the moment it is ready to be sold to the customers. We first need to understand what is changed in the design vs. the last model and if the new design is meeting the customer requirements. Then we need to look at the manufacturing process in terms of technology used, labor, if there is something that was needed to be changed and was not. We also need to investigate the parts supply for the product and understand if anything changed in regards to the components that we are using for the new engine.

Last but not least I would like to see the main symptoms that customers claimed and compare them with the previous engine. Interviewer: Great. (provide chart and additional information) Candidate: From the information provided it seems that there is nothing wrong with the new technology. Interviewer: All right, so where do you think the problem comes from? Candidate: I believe that one major change that the company made is in the base of suppliers, going from a limited number of suppliers to a bigger number of suppliers. Is the fact that they have more suppliers and are assembling the components in house a cause of the claims that we see in the warranty claim chart? Interviewer: Yes. You are right. Although each supplier is delivering to specification, there are tolerances in these specifications. Since the client is accustomed to specifying sub-assemblies, the tolerances were not tight enough for all of the individual components. Therefore, the stack-up of tolerances causes the parts to have poor fit resulting in leaks and lost parts. Candidate: OK. So the root cause for the problems is the stack-up of tolerances. Note: the interviewer needs to lead the candidate to identify this problem which is the stack-up of tolerances; all the other information given initially is a red herring meant to throw the interviewee off; after the interviewee has identified the problem, no matter how much help he/she gets, they need to come up with a list of recommendations and their risks on how to solve the problem. Interviewer: So, what do you recommend the client Cummins to do now? Candidate: The company can reduce its warranty claims by increasing tolerances on procured parts, increasing end product testing, or revising procurement agreement to order sub-assemblies. The first opportunity is more expensive but we could look at ways to minimize costs. The second will increase manufacturing time but would definitely decrease our warranty claims. The third option is more expensive but we could perform a cost-benefit analysis between the 1st and the 3rd options. Interviewer: Great! Thank you for your analysis.

Allure Magazine to Close Printing Facility in Idaho Case Type: operations strategy; reduce costs Consulting Firm: ZS Associates first round summer internship job interview. Industry Coverage: mass media, communications, publishing. Case Interview Question #00633: Our client Allure is a U.S. women’s beauty and lifestyle magazine, published monthly by magazine publisher Conde Nast (a division of American media company Advance Publications) in New York City. As a national monthly magazine, Allure focuses on beauty, fashion, and

women’s health. The magazine’s circulation, initially 250,000 in 1991, is over 1 million as of 2011. Recently, Conde Nast, the publisher of Allure Magazine, wants to restructure its printing division. They currently have 2 facilities where they print their magazines and want to move to only one facility. Conde Nast has hired us to tell them if this is a good idea or not. How would you go about analyzing this case? Additional Information: (to be given to candidate upon request)  Total number of magazines they sell per month: 1 million  Locations of the two printing facilities: one in Idaho and one in New Jersey  Both facilities can accommodate the total number of magazines they need. They each currently print 50% of the total magazines. Cost structure:  Total Costs = $1.50/unit  Fixed costs = $0.70/unit  Moving to one facility would save 20% of the fixed costs  The move would increase the distribution costs:  moving to New Jersey : by $0.04/unit  moving to Idaho: by $0.16/unit Possible Answer: Candidate: I have a question before going further with the case. What is the reason behind the magazine publisher’s decision to restructure the printing division? Interviewer: the main reason is reduction of costs. Candidate: OK. In order to assess the validity of their idea, I would start by looking into the cost structure of the company, specifically for the printing division. I would assess both the fixed and variable costs involved.

Then I would like to identify the cost savings that a consolidation of printing facilities would give to the company, and whether the company has the capability to do the consolidation both in terms of capacity or infrastructure involved by such a transition (distribution, operations, and feasibility of the move). Finally, I would like to assess the risks involved (regulations, market situation, etc). The interviewer should now direct the candidate to answer the following question: What will be the total cost per unit after the consolidation? What facility should they keep if at all? Possible Solution: Savings in the fixed costs: 20% * $0.70/unit = $0.14/unit Increases in costs from distribution:  New Jersey: -$0.04c/unit  Idaho: -$0.16/unit Total savings:  New Jersey: $0.14 – $0.04 = $0.10/unit  Idaho: $0.14 – $0.16 = -$0.02/unit Therefore, they should choose the New Jersey facility. The interviewer should then ask the following question: What are the risks involved in moving to only one printing facility? Possible Solution:  unemployment rate in New Jersey  local regulations by state government  operational cost: gas price  delivery time  cost of closing the other facility in Idaho  cost of materials  future increases in sales A good candidate will make a final recommendation in the end, like I suggest they close the Idaho printing facility because the New Jersey one offers them the best cost per unit. However, they need to understand that there are other things to consider when taking this decision as the ones I mentioned in the risks involved.

Trader Joe’s Chain to Centralize Company’s Sourcing Case Type: operations strategy, supply chain optimization. Consulting Firm: A.T. Kearney first round summer internship interview. Industry Coverage: retail.

Case Interview Question #00625: Trader Joe’s is a privately held chain of specialty grocery stores headquartered in Monrovia, California. As of February 2012, Trader Joe’s had a total of more than 360 stores. Approximately half of its stores are in California, with the heaviest concentration in Southern

California, but the company  also has store locations in 30 other states and Washington, D.C. The Trader Joe’ chain has offices in both Monrovia, California and Boston, Massachusetts. Describing itself as “your neighborhood grocery store” or “your unique grocery store”, Trader Joe’s stores sell mostly private label staple foods, organic foods, and specialty products. Unlike typical grocery stores, which may carry 50,000 items (or SKUs), Trader Joe’s stocks about 4,000 items, 80% of which bear one of its own brand names. You are a consultant working with A.T. Kearney. The other day you were on the golf course with billionaire businessman Mr. Theo Albrecht, who owns the Trader Joe’s chain. Mr. Albrecht posed a question that you found very interesting. His question was, why should he continue to centralize his company’s sourcing rather than split it to procurement and distribution at the individual store level. How would you best assess the question? Possible Solution: This is a pure conceptual “supply chain optimization” case in which the candidate will have to come up with a number of reasons to go one way or the other. Be creative in the way you think about it, the case is open ended, but look for guidance from the interviewer if you start to go too far. Candidate: May I take some time to think about how to go over this problem? Interviewer: Sure, take your time…..So, why should he centralize sourcing and distribution? Candidate: In order to come with an answer I would like to go over the pros and cons of centralizing and then weigh them in order to come with an answer. Interviewer: Fine with me. Candidate: Pros  

Ability to “squeeze” suppliers by aggregating demand Flexibility to change what to deliver to individual stores based on daily or weekly demand changes

Economies of scale in warehouse size, operation, etc. Reduce cost of warehouse by having it in a low cost area and not in the store, which I assume is a high cost area  Local people (at the store level) are not trained to negotiate. Having them at the store level is an extra cost Cons  

Local people can understand better the necessities (e.g.: because of format, size, place) for some niche categories  If distributed directly to the store the cost of distribution is paid by the supplier  There may be some products that are not bought at the centralized (headquarter) level because of limited size order. Managing at the store level make it viable. Interviewer: All good reasons, but say you have talked to Mr. Albrecht about many of those things and he keeps telling you that there may be something else. What could it be? 

Candidate: So, we have covered most things and we are still not finding the reason? Interviewer: Yes. Candidate: Looking at the ideas that I put in the pros vs. cons comparison, it could be similar to the notion of some products not being bought at the headquarter because of some quantities asked for an individual store. What if the supplier refuses to ship to individual stores because of limited volume? He may not have the trucks to do so in such a small volume or he may be unwilling to do so because is not profitable to do in that way. Interviewer: You got to the point. This was one of the cases I worked on. After a couple of hours in the golf course with my friend I actually came to the same conclusion, which later he validated with his team as the main reason. You did a great job. Thank you for your time.

Airbus to Build New A380 Assembly Facility in France Case Type: operations strategy, supply chain optimization; new product. Consulting Firm: Booz Allen Hamilton (BAH) first round summer internship job interview. Industry Coverage: aerospace & defense; airlines. Case Interview Question #00624: Our client Airbus SAS is an aircraft manufacturing subsidiary of EADS (European Aeronautic Defence and Space Company, Euronext: EAD), an European aerospace and

defense company. Based in Blagnac, France, a suburb of Toulouse, and with significant activity across

Europe,  Airbus produces approximately half of the world’s jet airliners. The A380 is a new double-deck, wide-body, four-engine jet airliner manufactured by the Airbus company. It is the world’s largest passenger airliner which is able to hold up approximately 800-900 people. For instance, the Airbus A380-800 provides seating for up to 853 people in all-economy class configurations, and the future version A380-900 will have a seating capacity of approximately 900 passengers in economy-only configuration. Originally designed to challenge Boeing Company (NYSE: BA)’s monopoly in the large-aircraft market, the Airbus A380 made its initial flight on 27 April 2005 and entered initial commercial service in October 2007 with Singapore Airlines. Parts for the A380 airplane are manufactured in multiple sites across Europe. The client Airbus has provided us the following information on site production: Cockpits (flight deck, the area usually near the front of an aircraft from which a pilot controls the aircraft) are produced in Northern Spain  The fuselage (an aircraft’s main body section that holds crew and passengers or cargo) is produced in Northern Germany  Wings are produced in Northern England  Landing Gears/Interior are produced in Southern France The client Airbus has asked us to determine where it is most economical to assemble the A380 airplane. How would you go about the case? 

Suggested Approach: This is an operations strategy case with a major focus on supply chain optimization. It involves the application of processes and tools to ensure the optimal operation of a manufacturing and distribution supply chain. This includes the optimal placement of inventory within the supply chain, minimizing operating costs, including manufacturing costs, transportation costs, and distribution costs. The candidate should form a structured approach to investigate the components that make up assembly, including costs and capabilities. Possible Solution: Interviewer: So, how would you determine the most economical site to assemble the A380? Candidate: Well, I would like to look into the specific costs incurred from the A380 assembly process and also determine the capabilities for assembling the planes across different locations (labor skill and supply, etc).

Interviewer: OK, I like your approach. Let’s focus on costs for now. Candidate: Great. I assume that the major costs of assembly would be transportation, labor, and PP&E (property, plant and equipment). Also, I would think that there would be tax incentives to assembling in one country versus another. Interviewer: Good. Let’s focus on transportation and labor for now, as these are the most significant cost drivers. What are some of the areas that you might analyze for potential assembly locations? Candidate: I would look at the existing sites where our client manufactures parts, as well as other areas in the region that may have low cost propositions, such as Eastern European countries that may have cheaper labor costs than Western Europe. Interviewer: Let’s take France as one location since it is the mid-point of the four manufacturing locations and compare it to the Czech Republic where labor is significantly less expensive than Western Europe. Candidate: Well, the location in France would incur less cost because the client would already have the landing gears and interior at the facility, and would have to transport the other three parts over a shorter distance. However, the labor costs in France would likely be significantly higher than those in the Czech Republic. Do we have any information about transportation and labor costs in each of these areas? Interviewer: Well, we have determined that labor accounts for about 70% of the total costs of assembly and transportation accounts for the other 30%. Disregard the other costs such as real estate, etc. Candidate: Great. Since labor accounts for 70% of the cost, let’s start there. Do we have any details about how much labor costs in the Czech Republic relative to France? Interviewer: Labor in France is 1.5 times higher than labor in the Czech Republic. What do you think that total transportation costs would be in France relative to the Czech Republic? Candidate: Since these parts are so large, I would assume that a major part of transportation is actually packing and loading the pieces on a large truck. Interviewer: You’re right. In fact, the marginal cost of each extra mile transported is close to zero. The most significant cost incurred is labor costs associated with the loading of parts for transportation. Candidate: I would assume that the landing gears and interior are fairly small and wouldn’t incur as large expenses for loading/unloading compared to the wings or the fuselage. Interviewer: You’re right. In fact, the interior and the gears are the easiest to pack of all the four parts and the fuselage is the hardest (most expensive). What would you recommend to our client Airbus?

Candidate: Well, I would recommend they assemble the A380 airplane in the Czech Republic. Since labor accounts for 70% of the total cost, and the labor in Czech Republic is about 33% less expensive than France, the client would save a significant amount of money, assuming that the skill level is constant across the two locations. Additionally, the transportation costs would not be lower in France since the majority of transportation expense is incurred by loading/unloading the parts and the interior/gears are the easiest to pack. It might be worthwhile to analyze Germany as a potential assembly location because the fuselage represents a significant portion of our transportation costs. The transportation savings may balance out the higher cost of labor in Germany (as compared to Eastern Europe). Interviewer: Very good. One last question – our client Airbus knows the demand for A380 jumbo jet will be 100 airplanes per year and they are trying to determine how large a new assembly facility they should build. How would you go about figuring this out? Additional Information: (Candidate should ask for this data)  It takes 9 months to assemble one A380 plane  One A380 airplane requires a space of about 300 feet by 300 feet Candidate: Well, we know that it takes 9 months or 0.75 of a year to produce one A380 airplane, so we need space to produce 75 planes in the facility (100 * 0.75 = 75). Also, one plane requires 300 * 300 = 90,000 square foot of space. So the total assembly facility would need to be 75 planes * 90,000 sq. ft. per plane = 6.75 million sq. ft. Interviewer: Excellent. Let’s wrap it up here. (The interviewee will now be asked to summarize high level findings and make a recommendation to the client Airbus regarding where it should produce and assemble A380 airplanes and the requirements for this assembly facility and operation).

Revol Wireless to Focus Investment in In-bound Call Centers Case Type: operations strategy. Consulting Firm: Arthur D. Little first round full time job interview. Industry Coverage: telecommunications, network. Case Interview Question #00620: Our client Revol Wireless is a regional wireless carrier and cell phone operator in the Midwest of the United States. Headquartered in Independence, Ohio, and operated by Cleveland Unlimited, Inc., the company offers completely unlimited wireless service, on a high quality all

digital  CDMA wireless network with flexible plans and unlimited service at an affordable rate. Customers pay a set amount of money each month for unlimited talk, text and data. The CEO of Revol Wireless has hired Arthur D. Little to advise him on his strategy for the next five years. You are one of the consultants working on this project, and you will be responsible for the sales effectiveness strategy. Basically, the CEO wants to focus the company’s investments in the most effective and profitable channels. The company currently has three direct sales channels: own retail stores, inbound (responsive) call center, and an outbound (proactive) call center. Based on the economics of each sales channel, what do you advise him to focus on? Suggested Approach: The interviewer was expecting a basic knowledge of the telecommunications industry as well as the three sales channels, and the case requires the interviewee to focus on the calculations. Customer Lifetime Value is a major framework to be used to approach this marketing and strategy case. Also, the interviewee has to drive the questions and collect – in a structured manner – all the numbers needed to solve the case. It is important to ask the appropriate questions as well as state the reasoning behind each question. Some important elements are:  Discuss customer service versus sales  Understand the mix of variable costs and fixed costs Additional Information: (to be given to candidate if requested) Economics of each of the three sales channels

Out-bound Call In-bound Call Center Center

Owned Retail

Success rate

10%

50%

50%

Traffic (# of customers)

20 millions

1 million

1 million

Average revenue per customer $40

$50

$85

Fixed / Variable cost

50/50

25/75

80/20

Total cost

$1 million

$1 million

$10 millions

Customer service portion

50%

0%

0%

Possible Solution: Interviewer: So, how would you go about this case?

Candidate: (Summarize the case and work on a framework) I want to understand the customers (size of the market, customer segmentation and consumer behavior), the competition and key figures of each sales channel (acquisition rate, sales volume, success rate, fixed cost, variable cost and traffic). Interviewer: Very good. Our client, the CEO of Revol Wireless, does not have insight about customer segmentation in each of the channels though. Candidate: Right. Let us see which figures the client has available. Do we know traffic in each channel? Interviewer: We do. In-bound call center and retail stores are accessed respectively by 20 millions and 1 million customers a year. The out-bound call-center accesses 1 million customers. Candidate: I would imagine some of these customers are also demanding customer services, right? Interviewer: You are right for the in-bound call center, which also serves as a customer service channel in a proportion of 50-50 (sales: 50%, customer service: 50%). However, our client believes that every customer visiting a retail store generates revenue, and the out-bound call center only focuses on sales. Candidate: What about success rate per each channel? Interviewer: In-bound call center, out-bound call-center and retail stores have respectively 10%, 50% and 50% success rates. Candidate: In looking at lifetime customer value, do we know the churn for each channel? Interviewer: Actually we don’t. But our client believes it is the same across all segments. Candidate: What about revenues per channel? Interviewer: Sales generated by in-bound call center, out-bound call-center and retail stores generate respectively $40, $50 and $85 in revenue on average per customer. Candidate: Perfect, we already have enough data for sales. What about cost? Do we know either total cost or cost per “call” for each channel? Interviewer: Our client has provided us with cost broken down by channel. In-bound call center, out-bound call center and retail stores generate respectively $1 million, $1 million and $10 millions in costs. Candidate: Do we know how much is fixed vs variable cost? Interviewer: We do know the current percentages. Out of total costs, fixed cost for inbound call center, out-bound call center and retail generate are respectively 50%, 25% and 80%.

Candidate: Fair enough. This is what we’ve got so far:

In-bound Call Center

Out-bound Call Center Owned Retail

Success rate

10%

50%

50%

Traffic

20 million

1 million

1 million

Average revenue per customer

$40

$50

$85

Customer service portion

50%

0%

0%

Sales portion

50%

100%

100%

Total Revenues

20 million * 10% * 50% * $40 = $40 million

1 million * 50% * $50 = 1 million * 50% * $85 = $25 million $42.5 million

Revenues per new customer

$40 million / (20 million * 50%) = $4

$25 million / 1 million = $25

$42.5 million / 1 million = $42.5

Fixed / Variable cost

50/50

25/75

80/20

Total cost

$1 million

$1 million

$10 millions

Fixed cost

$1 million * 50% = $500K

$1 million * 25% = $250K

$10 million * 80% = $8 million

Variable cost

$1 million * 50% = $500K

$1 milion * 75% = $750K

$10 million * 20% = $2 million

Variable cost per relevant traffic

$500K / (20 million * 50%) = $0.05

$750K / 1 million = $0.75

$2 million / 1 million = $2.00

From the calculations, we see that the average revenue per new customer generated by retail stores is the highest while inbound call center generates the lowest. Also, on the cost side, although the fixed cost by out-bound call center is the lowest, the variable cost for the in-bound call center is the lowest. Additionally, the out-bound cost center’s revenue is the lowest. This means that the out-bound call center generates a lower contribution margin, as its fixed cost is proportionally the lowest portion. Thus, out-bound call center’s total cost could increase the most should our client decide to invest more in this channel versus other channels – retail stores with a 80% fixed cost and in-bound call center with 50%. Because of the high variable costs, out-bound cost centers are a higher risk.

Interviewer: Very well. What is your recommendation then? Candidate: I would advise our client to focus its investments in the in-bound call center. The relevant variable cost for this channel is low, but so is the revenue for new customers. The client should examine the causes for the low revenue with an eye towards generating more revenue from customers through this channel. Interviewer: Excellent. Thanks.

Mayor of Baltimore Aims to Grow City Population by 5% Case Type: add capacity, growth; operations strategy. Consulting Firm: Mars & Co full-time first round job interview. Industry Coverage: government & public sector. Case Interview Question #00616: Your client is the current Mayor of the City of Baltimore, Ms. Stephanie Rawlings-Blake. Baltimore is the largest city in the U.S. state of Maryland and the 24th largest city in the country. It is located in the central area of the state along the tidal portion of the Patapsco

River, an arm of the Chesapeake Bay. The City of Baltimore has a population of 600,000 as of 2012. Over the past several years, the city has experienced a population decline which has negatively impacted the economy. In year 2000, the city population was 650,000. When the mayor ran for re-election her political platform was based on economic growth, so the recent population decline is an issue that is of great concern for her. The mayor has hired your consulting firm Mars & Co to understand what she can do to revitalize Baltimore. How can you help her? Suggested Approach: There’s no real framework to use here for this operations strategy case. The key is to be structured throughout, to prioritize your thoughts, and to be sure to be able to understand the interdependencies of the population growth and the city’s unemployment rate. Additional Information: (to be given to candidate if requested)  Population of Baltimore (2012): 600,000  Current unemployment rate: 8%  The mayor’s goal is to increase population growth by 5%  Companies in Baltimore employ 500 people on average Possible Solution: Interviewer: First question is, in general what could have caused a declining population?

Candidate: Well, the most obvious reason would be jobs or employment. But, it could also be quality of life issues directly related to city services, like crime, schools, roads, and so forth. It could also be quality of life issues related to other issues, like culture. Finally, it could be cost of living issues for the residents. Interviewer: Great. Let’s say the mayor has done some preliminary research and the major reason people are leaving is because of lack of employment opportunities. What reasons can you think of for why companies are not hiring in Baltimore or even leaving Baltimore? Candidate: I think one reason would be financial. Perhaps the tax rates in Baltimore are too high, or property costs are too high, or labor costs are too high. Another reason would be around talent. It might be that the companies can’t find the kind of skilled labor pool they need. A third reason could be around location – it could be that as commerce changes Baltimore is no longer in a convenient location for companies to conduct their business. Interviewer: OK, so how can the mayor drive business growth and create opportunities to increase population? Candidate: Well, first, let’s approach the business growth. The mayor could identify what all the problems are that companies have and define strategies to address them. For example, if companies are leaving Baltimore because of financial issues then the mayor can consider lowering the tax rate or give tax incentives or offer property tax benefits. She can also perhaps offer labor subsidies to entice companies to come back to Baltimore.  If the reason is around talent, then the mayor can implement a training program to give the labor pool the skills needed to be attractive to potential employers.  If location is an issue for companies, the mayor can consider implementing a transportation subsidy or offer transportation services to help them conduct their business. On the second issue of creating opportunities for population growth, I would think about it in terms of cost of living and quality of life. 

On the cost of living side, the mayor can consider reducing tax rates, property tax rates, and other issues that make the cost of living more expensive in Baltimore.  On the quality of life side, the mayor can address issues like transportation (making it easier to get around), improving the environment, creating more outdoor recreation spaces, creating more cultural attractions.  Also, the mayor can also create a promotional campaign to attract more people to the city. Interviewer: Shifting gears, how many employers would the mayor need to attract in order to increase the city population growth by 5% and decrease unemployment rate by 3%? 

Candidate: Well, the current population is 600,000. What is the current unemployment rate and how many employees does one company employ? Interviewer: On average one company employs 500 people and the current unemployment rate is 8%.

Candidate: And the unemployment rate affects only people who can be employed? Interviewer: Yes, that’s correct. Candidate: So the current population is 600,000 and assuming the average life span is 80 years, we can segment the population like this: 0-20, 20-40, 40-60, 60-80. I’ll make the assumption that the working age population is from 20-60. Assuming an even distribution then we have 300,000 of employable people. Interviewer: That sounds reasonable. Candidate: OK, so currently the unemployment rate is 8% which means that there are 300,000 * 8% = 24,000 unemployed people, and 300,000 * 92% = 276,000 people are employed. We want to reduce the unemployment rate to 5%. Assume that the population growth is evenly distributed across all age segments, then the new population of employable people would be (1 + 5%) * the current employable population of 300,000 = 300,000 * (1 + 5%) = 315,000 people. Therefore, the number of people that are employed under the new unemployment rate of 5% would be: 315,000 * 95% = 299,250 = ~300,000 So, the mayor would have to help create jobs for 300,000 – 276,000 = 24,000 people. You mentioned that each company employs 500 people on average, so 24,000/500 = 48. The city will need to attract 48 new companies to accommodate the 3% decrease in unemployment rate and 5% increase in population. Interviewer: Good. So what can the mayor do to attract businesses to the city? Candidate: The mayor can consider lowering the tax rate or give tax incentives or offer property tax benefits. She can also perhaps offer labor subsidies to entice companies to come back to Baltimore. If the reason is around talent, then the mayor can implement a training program to give the labor pool the skills needed to be attractive to potential employers. If location is an issue for companies, the mayor can consider implementing a transportation subsidy or offer transportation services to help them conduct their business. Interviewer: Let’s say that I am the mayor and you have to summarize your recommendation for me, what would you say? Candidate: (Summarize the case and give the recommendation)

R&R Ice Cream to Reduce Sales Seasonality Case Type: operations strategy. Consulting Firm: Mars & Co second round summer internship interview. Industry Coverage: Food & Beverages. Case Interview Question #00615: Richmond Foods is the leading ice cream manufacturer in the United Kingdom. Headquartered at Leeming Bar, North Yorkshire, England, the company controls more than a third of the UK’s take home ice cream market. It supplies ice cream to many of the UK’s leading

supermarkets, grocery stores, and restaurants. In May 2006, Richmond Foods announced that it was to be taken over by Oaktree Investments, an American Investment Company, who will unite Roncadin, the biggest German own-brand label ice cream manufacturer, and Richmond, to make the largest ice cream manufacturer in Europe. The new company is now known as R&R Ice Cream. Now R&R Ice Cream is the European market leader with a share of 31% of the overall ice cream market. The client, the CEO of R&R Ice Cream, is concerned because its ice cream sales are more seasonal than those of competitors, even though overall profits are excellent. How would you help the CEO to reduce sales seasonality? Suggested Approach: A good possible framework is to look at the 3C’s: Company, Customer, Competitors Company  Product mix: characteristics, seasonality, how perishable it is, etc.  Operations: is the manufacturing capacity used continuously or seasonally throughout the year? can we use the capacity to produce less seasonal products than ice creams?  Customer  Segments, trends, characteristics in terms of seasonality  Competitors  How do they differ from the client with regards to the two previous points: Company and Customer It is important that the candidate recognizes that there are two ways to reduce seasonality here: 

 Modify the seasonality of sales themselves  Modify the production or the products to come to a less seasonal use of capacity. Additional Information: (to be given to candidate if requested) 1. Products

 Ice creams, sorbets  Very perishable products (by nature and regulation)  Very extensive to store 2. Operations  The client’s pduction follows seasonal sales pattern (too expensive to store)  The production line could be used to produce other types of items.  90% of the production costs are fixed. 3. Customers: 3 main types of customers  Ice cream stores: highly seasonal  Retail / supermarkets: moderately seasonal  Restaurants: almost not seasonal 4. Market situation

Channel

Overall market split

Client’s market share on this channel

Contribution to its overall market share

Retail / Supermarket

40%

40%

16%

Restaurants

30%

10%

3%

Ice cream stores

30%

40%

12%

Total

100%

31%

Possible Solution: Interviewer: How would you go about approaching this problem? Candidate: (The candidate should come up with a framework. See the 3C’s framework in the “Suggested Approach” section above.) Interviewer: Good points. Where do you want to start? Candidate: Let’s talk about the operations first. The demand for ice cream is by nature seasonal. Does the client currently produce ice cream continuously or seasonally throughout the year? Interviewer: There are peaks in production to match demand. Why do you ask? Candidate: Considering that 90% of the current costs are fixed, it might be less expensive to smooth production over the year. The capacity would not need to be as high since there would not be peaks anymore, therefore the fixed costs would be reduced. However, this would involve storage of ice cream during the months of low demand.

Interviewer: This is an interesting thought. However, our ice cream is too perishable to be stored for more than 2 weeks. Candidate: Maybe we could modify our products by adding preservatives and stabilizers. Interviewer: Not a bad idea. However, regulations prevent us from doing this. Candidate: OK. It seems that production of our ice cream will remain seasonal then. To maximize the use of capacity and reduce our fixed costs burden, could the plants be used to manufacture other types of products during non-peaks periods? Interviewer: This would be a possibility, but we have not explored it in details. What else would you need to know? Candidate: We discussed the possibility to reduce the seasonality of operations. Now, I would like to talk about reducing the seasonality of sales. Who are our current customers? Interviewer: Who do you think they might be? Candidate: As a customer, I can buy ice cream at supermarkets, ice cream stores/chains, and restaurants. Do we sell to all of these channels? Interviewer: Yes, they are all our customers. Which do you think has the most seasonal demand? Candidate: Ice cream stores would definitely be the most seasonal. Supermarkets would be less seasonal because people still eat ice cream at home during winter watching TV, even though they buy less than during summer. Finally, I would say that restaurants are the least seasonal because they always have ice creams on their menu and add it with apple pies, chocolate tarts, etc. Does it seem reasonable? Interviewer: Yes, it makes sense. Candidate: How do we split our sales to these channels? And how does it compare to our competitors? Interviewer: Let me give you the following data. We miss some client information here. Could you please calculate what’s left for me?

Channel

Overall market split

Client’s market share on this channel

Contribution to its overall market share

Retail / Supermarket

40%

40%

16%

Restaurants

30%

?

?

Ice cream stores

30%

40%

12%

Candidate: You mentioned initially that the total market share of the client is 31%. Since retail/supermarket and ice cream stores contribute to 16% and 12%, restaurants must contribute to the rest (31% – 16% – 12% = 3%). Therefore, the client’s market share on restaurants channel must be 10% (3% / 30% = 10%) Interviewer: Good. Based on everything we discussed, what would you recommend the client do to address its seasonality concern? Candidate: Firstly, I would recommend targeting more strongly the restaurants, as their demand is less seasonal and the client is relatively weakly positioned there compared to competitors. Secondly, I would recommend looking at products we could manufacture in the current plants during non-peak times of the year, so as to optimize the use of the fixed cost capacities. Interviewer: Great. Thank you.

Red Bull’s Marketing Campaign Not a Huge Success Case Type: operations strategy, marketing. Consulting Firm: LEK Consulting first round job interview. Industry Coverage: food & beverages. Case Interview Question #00552: Our client Red Bull GmbH is a large global beverage company that sells the popular Red Bull energy drink. In terms of market share, Red Bull is the No. 1 energy drink in the world, with about 4.5 billion cans sold each year. In 2011, a total of 4.63 billion cans of Red

Bull  were sold in over 161 countries. Headquartered in Fuschl am See, Austria, the client company Red Bull GmbH has more than 8,300 employees generating 4.25 billion euros in revenue worldwide. Last month, the client Red Bull’s marketing department ran a distinct trade promotion in three separate stores in the United State. They’ve hired your consulting team to tell them whether or not the promotions were successful. How would you go about helping them figure this out? Were the promotions a “success”? Additional Information: (to be given to candidate if asked) 1. Promotion

During the promotion month,  Store 1 was instructed to give a 10% discount on all Red Bull purchase;  Store 2 was given a “buy 4 get 1 free” deal;  Store 3 was given $10,000 to promote the Red Bull product however they wanted. 2. Sales Volume In a non-promotion month, each store typically buys 10,000 cans of Red Bull. During the promotion month,  Store 1 purchased 12, 000 cans;  Store 2 purchased 15,000 cans;  Store 3 purchased 14,000 cans. 3. Revenue and Cost Each can of Red Bull is sold for $5. The cost to produce a can of Red Bull is $2.50. This figure includes all relevant costs (i.e. raw materials, manufacturing, distribution, etc.). Possible Answer:  

The candidate must think through what constitutes a “successful” promotion, and then synthesize data to provide a recommendation to the client. For this case, a “successful” promotion is one that generates more profit than would be attained without running any promotion at all. However, the interviewer should ask the candidate to define how success might be measured, and should encourage the candidate to brainstorm several other barometers of success, besides profits. This marketing case scenario is not complex, but it requires a proper initial structure. The candidate should not be bogged down by the numbers and fail to see the bigger picture. 1: Calculate Store Profits Store 1: (90% discounted * 12,000 cans * $5.0 revenue per can) – (12,000 cans * $2.5 per can) = $24, 000 profit  Store 2: (80% discounted * 15,000 cans * $5.0 revenue per can) – (15,000 cans * $2.5 per can) = $22, 500 profit  Store 3: (14,000 cans * $5.0 revenue per can) – ($10,000 promotion cost) – (14,000 cans * $2. 5 per can) = $25,000 profit  Normal, or baseline profits: (10,000 cans * $5.0 revenue per can) – (10,000 cans * $2.5 cost per can) = $25,000 profit 2: Business Insight 

From the above calculations, all three stores are profitable. However, no store’s profits exceed that of a non-promotion, normal month.

3: Additional Discussion The interviewer can add to this case by asking the candidate to discuss other factors that could possibly be considered. Some of the factors include:    

External factors: economy, market, competitors Sales trends: perhaps baseline is shrinking Long term effects of the promotion: market share, brand loyalty , affecting customer buying habits Controllable factors: may be it would work better in different geographic locations, with different customer segments, or at a different time of the year, e.g. summer v.s. winter.

Adobe Systems to Not Introduce “Suite” Software Product Case Type: new product; operations strategy. Consulting Firm: KPMG Strategy second round job interview. Industry Coverage: software, information technology (IT). Case Interview Question #00548: Your client Adobe Systems Incorporated (NASDA: ADBE) is a large multinational computer software company founded in 1982 and headquartered in San Jose, California, United States. The company has historically focused upon the creation of multimedia and creativity

software products,  with a more-recent foray towards rich Internet application software development. Currently the company makes photo editing, web design, video editing, and desktop publishing software. The client Adobe Systems has historically been the market leader in two of the four software product categories (photo editing and desktop publishing). Its software products in the other two categories (web design and video editing), however, are newer and have failed to achieve significant market share so far. The company has traditionally sold its four point products separately. Recently the CEO of Adobe is considering introducing a new business model where customers can purchase an integrated “Suite” of the four products for $1,000. Furthermore, it will raise the prices of the products in the two categories it dominates (photo editing and desktop publishing) to motivate customers to purchase the Suite instead. Adobe conducted a concept test on the Suite product, and would like your assistance in deciding whether to go forward with the Suite product launch. Case Part #1: Will the proposed “Suite” strategy increase company profitability or not? Additional Information: Exhibit 1: Results of Concept Test – Customer preference shares (Show Exhibit 1 to the candidate up front)

Software Category

Photo Editing

Web Design

Video Editing

Desktop Publishing

Customers who will purchase the Suite

75%

70%

70%

40%

Customers who will purchase only the point product

20%

30%

30%

50%

Customers who balk

5%

10%

Note: customer overlap  Of the desktop publishing customers who chose to purchase the Suite, 50% would have also bought photo editing software.  All of the web design customers who chose to purchase the Suite would have bought photo editing software as well.  All of the video editing customers who chose to purchase the Suite would have bought photo editing software as well. Instruction to Interviewer: Once the candidate has asked appropriate questions and layed out a logical framework to the first three exhibits, ask him/her to calculate the profitability of this strategy based on the results of the concept test. Caveat: Don’t worry about upgrades or software lifecycles, assume all customers buy full versions of the product. Possible Answers: Profitability Analysis

1. Question: What are the current prices of each of the four products, and how will they change in the new business model?

Answer: See Exhibit 2. Interpretation: the increased pricing of photo editing and desktop publishing products may motivate some customers to buy the Suite instead, but also may cause some customers to balk. 2. Question: How many customers will upsell to the Suite? Answer: See Exhibit 1. 3. Question: How many customers will the company lose because of the increased prices for the photo editing and desktop publishing products? Answer: See Exhibit 1. 4. Question: Will the company capture any new customers who will purchase the Suite who would not have purchased any point products? Answer: Assume that all Suite customers are upsold from point products. 5. Question: What are the variable costs of producing each of the products?

Answer: See Exhibit 2. Interpretation: the photo editing and desktop publishing products yield the most profitability, but the new Suite will also be fairly profitable 6. Question: Are there additional fixed costs that the company will incur to launch the new product? Answer: Assume that additional fixed costs and synergies gained from the product strategy are a wash in the short term. 7. Question: How many units of each product does the company sell?

Answer: See Exhibit 3. Calculations for Base Case Scenario

Desktop Software Category Photo Editing Web Design Video Editing Publishing Base case units

600,000

200,000

200,000

800,000

Price per unit

$600

$300

$500

$500

Revenue

$360 M

$60 M

$100 M

$400 M

Costs per unit

$60

$90

$125

$100

Profits

$324 M

$42 M

$75 M

$320 M

Total

$920 M

$761 M

Calculations for Suite Scenario

Software Category

Photo Editing

Web Design

Video Editing

Desktop Publishing

Suite

Balkers

-30,000

0

0

-80,000

0

Upsell

-450,000

-140,000

-140,000

-320,000

1,050,00 0

Overlap

Total

-440,000

Revised Units

120,000

60,000

60,000

400,000

610,000

Price per unit

$700

$300

$500

$600

$1,000

Revenue

$84 M

$18 M

$30 M

$240 M

$610 M

Costs per unit

$60

$90

$125

$100

$250

Profits

$76.8 M

$12.6 M

$22.5 M

$200 M

$769.4 $457.5 M M

$982 M

A good answer to the analysis is that although the Suite strategy is slightly more profitable than the status quo ($769.4 M vs. $761 M), it’s not a clear winner. An outstanding candidate will easily grasp the impact of cannibalization and price increases on the product mix. Case Part #2: Start by giving answer to Case Part #1 if the candidate did not calculate correctly. Background: It appears that there is marginal upside for the company in pursuing the Suite strategy from a near-term profitability standpoint. Question: What are some additional long-term factors that the company should consider to make a decision on the Suite strategy? Possible Answers: Do a market opportunity analysis. 1. Question: Examine growth, market share, and size of each category. Answer: See Exhibit 3. Interpretation: the newer categories represent significant market opportunities in terms of both size and growth. 2. Question: What is the overlap between customers in each category? Answer: Almost all customers own products from at least three of the four software categories when you include their ownership of competitive products. The majority of our client’s customers own competitive products in the web design and video editing categories.

3. Question: Will customers actually adopt the video editing and web design software as a result of owning the Suite if they were previously using competitive products? Answer: Research shows that after customers who own our client’s market leading software products and try the company’s video editing and web design software products will switch from competitors 75% of the time. 4. Question: What does the competition look like in the video editing and web design markets? Answer: Both categories are currently fragmented by numerous small companies. Based on the direction of market growth, it is imperative that the company gain success with its newer products in order to maintain long-term growth. Given the high amount of customer overlap, it is probably wise for the company to leverage its current market leadership in the photo editing and desktop publishing categories.

Hedge Fund Firm Citadel to Hire More Quant Analysts Case Type: operations strategy, business process optimization. Consulting Firm: Cambridge Associates second round job interview. Industry Coverage: Financial Services. Case Interview Question #00546: Our client Mr. Kenneth Griffin is the Chief Executive Officer (CEO) of the famous hedge fund firm Citadel Investment Group. Based in Chicago, Illinois and founded in 1990 by Mr. Kenneth Griffin, the firm Citadel today deploys capital across multiple asset classes and strategies.

Currently  Citadel manages $11 billion in assets under management (AUM) and is one of the world’s largest hedge fund managers. Mr. Kenneth Griffin originally developed the hedge fund’s proprietary investment strategy over numerous hard working hours and many years of painstaking research. The investment strategy can be summarized as such:    

Citadel invests in and selects from a universe of companies that have tradable securities (debt, equity and derivatives). The portfolio manager (PM)’s and analysts performs in-depth fundamental research and analysis on these companies. The PMs decide on a position to take (short or long, type of securities). The PMs seek to exit/unwind typical position within 1-2 years.

The hedge fund combines a proprietary computer program along with its fundamental analysis to enhance its security selection process. The computer program first short-lists the companies based on criteria such as market capitalization, liquidity of securities and foreign exchange risk. If so desired it then works out a recommended position to take based on weighted assessments along dimensions such as market multiples-based valuation and bond yield-curve shape. The fundamental analysis department then takes over, performing further research and analysis, and finally deciding on the position that the fund should take. The fundamental analysis department currently has a staff of 9 consisting of: 3 senior analysts (who, incidentally, are also the 3 partners of the firm), 4 junior analysts (MBA level), and 2 assistants (fresh college graduates). Currently, of the 250 companies that the proprietary program shortlists, based on preliminary criteria, only 100 or so get analyzed every year. The hedge fund partners feel that significant opportunities are being left on the table, and would like to see how they can improve the situation, such as possibly hiring more staff or investing money in more advanced technology. What would you recommend Citadel to do? Possible Answer: This is an operations strategy and business process optimization case. The candidate should recognize it and attempt to draw a process flow diagram, such as Figure 1 below.

Clearly, the firm is not realizing the full potential of the computer program in shortlisting companies. The candidate should quickly realize that the bottleneck is in the Fundamental Analysis Department, and that the solution is to hire more staff. The next question, then, is what level of staff to hire. The hedge fund does not want to change the number of senior analysts that it already has. So it will only consider hiring additional junior analysts or assistants. The interviewer should prompt the candidate to ask questions about information they would seek in order to assess what type and how many people to hire. As the candidate asks about the nature of work that each level of staff does, the following information can be revealed: Additional Information:  There are 3 steps to the process of fundamental analysis: Research, Analysis, and Strategy Formulation. These are typically 50%, 30% and 20% of the work (man-hours) that needs to be done on a company.  Of the Research work, 20% needs to be done by a partner, 60% can be done by a junior analyst and up, and 20% can be done by an assistant and up.

Of the Analysis, 30% must be done by a partner, and 70% can be done by a junior analyst and

 up.

Of the Strategy Formulation, 60% must be done by a partner, and 40% can be done by a junior analyst and up.  All levels of staff are equally productive, provided they are doing the work they appropriate for their level. At this point, the candidate should realize that it is important to work out what proportion of the work must be done by a partner and by other staff respectively. Good candidates will work out the following data table without too much prompting: 

Type of work

Percentage of work

Partner

Junior Analyst

Assistant

Research

50%

20%

60%

20%

Analysis

30%

30%

70%

Strategy Formulation

20%

60%

40%

100%

20% * 50% + 30% * 30% + 60% * 50% + 70% * 30% + 50% * 20% = 60% * 20% = 31% 40% * 20% = 59% 10%

Total

From the above calculations, we know that  31% of the work must be done by a partner,  59% of the work can be done by a junior analyst and up,  10% of the work can be done by anyone. So the optimal ratio of staff should be approximately 3:6:1 (partners: junior analysts: assistants). Since the hedge fund wants to keep the number of partners at 3, it can hire 2 more junior analysts without having to bring in another partner. This makes a total of 6 analysts. The firm is also free to let go one assistant. Ask the candidate the pros and cons of letting an assistant go (marginally lower cost but potentially important to have more then one assistant in case one leaves to business school abruptly). The candidate may approach the question from the direction of marginal revenue and marginal cost. This is not wrong, but the interviewer should guide the candidate back to the operations based approach by hinting that each additional company analyzed has an expected return of tens of millions of dollars, while each staff can be hired for hundreds of thousands of dollars (or less), and so the candidate should be able to conclude that the real constraint of the case is that the number of partners is capped at 3. Higher Level Questions: Good Candidates will get through the above analysis fairly quickly. They will also demonstrate ability to focus on what is important and engage the interviewer. A good question for candidates with operations

backgrounds that they want to leverage in interviews or focusing on going into operations include the following bonus questions. Bonus Question #1: If the hedge fund hired 2 more junior analysts as you recommend, what number of firms will the team be able to process a year? Possible Answer: If the candidate assumes that none of the staff were idle before (a wrong assumption, since there is idleness in the system), then the candidate would answer that the throughput would increase by 2/(3+4+2) = 2/9, or about 22%. If the candidate assumed that only junior analysts were fully utilized before, and idleness existed at both the partner and assistant levels, then the candidate would respond that the throughput would increase by 2/4 or 50% (since junior analysts increased from 4 to 6). This is again wrong, because all work that junior analysts do can also be done by partners, and so it is wrong to assume that partners are idle in the system. The candidate should be able to realize that in the previous system, partners and junior analysts are fully utilized, but idleness exists at the assistant level. So increasing the junior analysts by 2 is increasing the fully utilized staff by a factor of 2/(3+4) = 2/7 or 29%. So the fund’s throughput should increase to 100 + 100 * 29% = 129 companies analyzed a year. Bonus Question #2: If there is additional time, the candidate could be asked what other ways the hedge fund might optimize their procedure. Possible Answer: This bonus question is about relaxing the simplifying assumptions that were made in the model used for addressing the questions above. The model assumed, among other things, that the companies analyzed all took a fixed amount of work and gave a standard (constant) expected return. If the proprietary program could give each company a unique expected return and an estimate of the time required for fundamental analysis, the fund should prioritize its analysis according to the ratio of expected return to time required for analysis. For example, if Company A had an expected return of $10 million and required 5 man-days to analyze, and Company B had an expected return of $5 million and required 2 man-days to analyze, then Company B should have a higher priority ($2.5 million per man-day vs. $2.0 million per man-day). Priorities are necessary since the hedge fund can only analyze a fraction of the 250 shortlisted companies.

Riva Group to Exit Hot Roll Commodity Steel Production Case Type: operations strategy; math problem. Consulting Firm: Capgemini Consulting second round job interview. Industry Coverage: metals production; manufacturing.

Case Interview Question #00542: Your client Riva Group is a large European steel manufacturing company headquartered in Milan, Italy. The company had annual revenues of $6.8 Billion and operating profits of $650 Million last year. Currently it is the world’s eighteenth and the third largest steel producer in

Europe. Its products include  hot rolled flat steel, bars, iron rod and slabs. The Riva Group operates integrated steel mills. Their competitors in Europe are 10 other integrated mills (IM’s), using 150 year old blast furnace technology and 30 mini-mills (MM’s), all of which are smaller than the IM’s. They use most modern technology, a method commercialized during the late 1970s. A process flow diagram for both the technologies and the end product of each stage is given below (Figure 1).

Raw materials

Integrated mill

Mini-mill

Product

Melt

Cast

Roll

Finish

Coat

Continuous caster

Hot rolling line

Cold rolling line

Hot dipping line or Electro-galvanizing line

Electric arc furnace

Continuous caster

Hot rolling line

Cold rolling line

Hot dipping line or Electro-galvanizing line

Melt

Slab

Hot roll

Cold roll

Hot Dip, Electrogalvanized

Blast furnace -> Ore, Coke, Basic oxygen Coal furnace

Scrap

Question #1: Ask the candidate what he/she can understand from the flow diagram. (Guide if they don’t get there soon) Possible Answer: Only the melting process is different. All others are same for integrated mills and mini-mills. The basic product (hot rolls) feeds the upstream stage, gaining value at each stage. Question #2: A new steel commodity exchange is under development in the European Union. The company will be able to buy and sell commodity steel (hot rolls) on the exchange. With the opening of the online exchange, should Riva Group continue to make its own commodity steel or should it buy commodity steel? Additional Information: If the client decides to exit the production of commodity steel (hot rolls) and decide to buy from exchange, the bought steel will be used as inputs for manufacturing value added steel product. Possible Answer: How to solve this case? Let us use the hypothesis driven problem solving method. Say the hypothesis here is “Riva Group should buy commodity steel from the exchange”. To prove this we need to answer the following questions. Hypothesis: Riva Group should buy commodity steel from the exchange 1. Savings from buying on commodity market will be more than profit lost from abandoning commodity steel production  

How much revenue and profit currently comes from commodity steel? How much would the company need to save by buying commodity steel on an exchange to make up for profit lost by not making it?

2. Riva Group’s cost disadvantage in commodity steel will make this an unprofitable business  What are Rive Group’s production costs compared to mini mills?  What will the market price for commodity steel be? – Candidate has to make assumptions Additional Information: Provide the following data about Riva Group to the candidate when asked.

Volume (million tons) Price ($/ton) Unit Cost ($/ton) Commodity steel Hot roll

13.0

275.00

260.00

Cold roll

5.0

320.00

280.00

Hot dipped

2.0

410.00

360.00

Electro-galvanized 1.0

560.00

475.00

Value-added steel

Total

21.0

To produce commodity steel the operating costs for mini-mills is $220 per ton, compared to $260 per ton for integrated mills. The candidate is expected to perform the following calculations for Riva Group

Operating Sales in $m costs in $m

Operating profit in $m

Individual revenue as Individual profit as % of total revenue a % of total profits

275 * 13.0 260 * 13.0 = = 3575.0 3380.0

3575.0 – 3380.0 = 195.0 3575/6555 = 54.54% 195/580 = 33.62%

Cold roll

320 * 5.0 = 280 * 5.0 = 1600.0 1400.0

1600.0 – 1400.0 = 200.0 1600/6555 = 24.41% 200/580 = 34.48%

Hot dipped

410 * 2.0 = 360 * 2.0 = 820.0 720.0

820.0 – 720.0 = 100.0

Commodity steel

Hot roll Value-added steel

820/6555 = 12.51%

100/580 = 17.24%

Electrogalvanized

560 * 1.0 = 475 * 1.0 = 560.0 475.0

560.0 – 475.0 = 85.0

560/6555 = 8.54%

85/580 = 14.66%

Total

6555.0

580.0

100.00%

100.00%

5975.0

Key Conclusion from the calculations: The profits and revenues contributed by commodity steel (hot roll) is significant to Riva Group, accounting for 33.62% and 54.54% respectively. It is highly likely that the price of commodity steel on the exchange will drop below $260 per ton which is the variable cost per ton for integrated steel manufacturers. This means that our client Riva Group will make losses. Question #3: How much should be the minimum saving from the ‘Buy’ decision so that we can maintain the same profits as now? (The steel bought will be used as inputs to the value added steel) Possible Answer: Calculations to be performed by candidate: If the client decides to stop producing their own commodity steel, profits that the client will give up = $ 195 million  Number of tons produced after exiting commodity steel = 8 million tons (Only value added steel = 5.0 + 2.0 + 1.0 = 8.0 million)  To keep the same profits extra savings needed = 195,000,000 / 8 million tons = $ 24.375 per ton Therefore, buying commodity steel in the exchange should result in at least $24 to $25 per ton saving in order for the client to keep the same profits as now. 

Recommendations: The numbers and environment suggest that Riva Group should exit commodity steel production and start buying from the exchange. Question #4: What are the potential risks of such a decision to exit commodity steel production? Possible Answer:  Union and labor issues from laying off labor  Pressure from the government  Loss of control on quality of steel  Competitive weakness  Subject to fluctuations in the market price

City of Atlanta to Develop Strategic Plan to Revive Economy Case Type: add capacity, growth; operations strategy. Consulting Firm: Oliver Wyman first round job interview. Industry Coverage: government & public sector. Case Interview Question #00535: The client Kasim Reed has just been elected to be the Mayor of Atlanta, the capital and the most populous city in the U.S. state of Georgia. As of 2011, the city of Atlanta has a population of about 500,000. Atlanta is also the cultural and economic center of the Atlanta

metropolitan area,  which is home to more than 5 million people and is the ninth largest in the U.S. Over the last few years, the city of Atlanta has experienced some hard economic times and there has been a slight decline in the city population (in year 2009 the population is estimated to be 542,000). The Mayor’s election platform was centered on a message of economic revival with a strategic plan to be launched in the first 100 days of assuming office. The Mayor has hired you to help develop the plan. What information would you like to know about the city of Atlanta and what is your plan? Instruction to the Interviewer: The objective of this operations strategy case is  To see if the candidate can come up with a comprehensive framework to get to a solution  To see if the candidate can complete some quantitative analysis  To see if the candidate can demonstrate creativity Suggested Structure:  Location of the city and the surrounding cities  Current businesses and environment  Government support – tax incentives, infrastructure support, etc. Question #1: What could have caused the population/economic decrease in Atlanta? Possible Answers:  Poor macro-economic conditions  Tax increases  Aging population  Deteriorating infrastructure  Surrounding cities are having poor times  Increasing crime rates  Major airlines cut numerous flights too and from the city Question #2: (Quantitative Analysis) The city of Atlanta’s unemployment rate is currently 8% and the Mayor would like to increase the population by 5% and to decrease unemployment rate to 5%. How many new companies will they need? Additional Information: 60% of the city population can work and the average company size is about 500 employees. This key information is needed to solve this problem, only give it to the candidates if they know they need to ask for it. Possible Answer:

Population

500,000

% Eligible to Work

60%

Possible Workers

500,000 * 60% = 300,000

Current Unemployment

8%

Current

Goal

Employed

300,000 * 92% = 276,000 299,250 = 300,000 * (1 + 5%) * 95%

Unemploye d

300,000 * 8% = 24,000

15,750 = 300,000 * (1 + 5%) * 5%

This requires creating 299,250 – 276,000 = 23,250 new jobs. At 500 jobs per company, the city will need 23,350 / 500 = ~50 new companies/plants. Question #3: What could be done to get the ~50 new companies/plants? Possible Answers:  Get current companies to expand locally  Offer to build roads to and from the plants  Tax Incentives  Lobby other governors of “at capacity” cities to suggest your city  Lobby federal government for more federal jobs  Actively marketing of the city as a destination spot Question #4: The state of Georgia is looking to build a new university and is considering this city. Is that a good thing or bad thing? Should the Mayor support this? What are some of the possible concerns for the new Mayor? Possible Answers:  The city already has more than 30 institutions of higher education, including Emory University and Georgia Institute of Technology  The city population is getting older (retirement community) and not ready for this  No central location geographically to put a large university  How will the new university be funded? What will the city’s role be?  What is the time frame for building the new university?  Will progress be able to be made before the next elections come up? Question #5: Ask the candidate to give a 30-second summary of the case.

Canadian Oil Sands Concerned about Cost & Time Overruns Case Type: operations strategy; math problem. Consulting Firm: Schlumberger Business Consulting (SBC) final round job interview. Industry Coverage: Oil, Gas & Petroleum Industry.

Case Interview Question #00523: The Canadian Oil Sands Limited (TSX: COS) is a Canadian company headquartered in Calgary that generates income from its oil sands investment in the Syncrude Joint Venture. Syncrude operates an oil sands facility and produces crude oil through the mining of oil sands

from ore deposits. As of  January, 2007, the company holds a 36.74% interest in Syncrude, which is the largest stake of any of the joint owners. Recently, the Canadian Oil Sands company are planning to build an oil production and refining facility in the Athabasca region of northern Alberta, Canada (Shaded regions in Figure 1 show an area where client wants to build the refinery). The estimated cost of building a facility is $5 billion and estimated time for completion is 5 years. The company is concerned about cost and time overrun to complete the project. For your information, 5 other competitors have already started building production and refining facility in the nearby areas. You have been hired as a consultant to assess the situation and make them aware of the potential issues. How would you go about the case? Notes to the interviewer: The objective of this case is To see if the candidate possesses a good clock speed, i.e. if the candidate can quickly explore several possible reasons for time and cost overruns.  To see if the candidate presents creative solutions to specific questions posed by the interviewer and to see if the candidate can quickly do basic math. Suggested Structure: 

The case should start with the candidate hypothesizing the reasons that could lead to cost and time overruns. To guide the candidate, the interviewer may ask the candidate to list the possible steps in the project. Question #1: What are some of the possible reasons for cost over-runs? Possible Answers:  Uncertainty or Changes in the field profile resulting in increased construction costs.  Changes in price of commodities like steel and other materials used for construction.  Environmental regulations resulting in additional investment.  Suppliers operating at full capacities resulting in additional investment. Hence the supplier may ask the client to share some investment.  Skilled labor shortage for construction resulting in very high salaries. This is the biggest problem in this industry and the case will focus on this problem.

Question #2: What are some of the possible reasons for time over-runs?

Possible Answers:  Design and Field development planning delays  Manufacturing lead time for machinery and equipments  Transportation lead time for plant machinery and equipments  Labor shortage during construction, installation and commissioning  Poor productivity of untrained labor  Government approvals and regulations  Approval from environmental authorities Notes to the Interviewer: At this stage, the case will now focus on the labor issues. Provide the following additional information to the candidate.  The components of the oil production and refining facility are fabricated in another country, however joining the components and installation take place on-site.  The peak demand for labor will occur in 3 years from now.  The period sadly coincides with the peak labor demand for competitors in the same region. Especially we need several welders and we are quite sure that they can’t get welders. Question #3: Can you think of any ideas to manage the situation? Possible Answers: Some possible suggestions could be:     

Automation of construction, material movement, fabrication and welding of components Explore opportunities to get pre-welded components from supplier Design components in such a way so that the need for welding itself is reduced Share some facilities like piping, storage etc with competitors thereby eliminating the need for separately building them Recruit labor from other countries (But cultural issues, immigration issues, opposition form labor unions in Canada, not many want to move to Alberta, Canada as it is very cold)

Recruit labor from US as there seems to be a decline in manufacturing industry and hence the client may be able to obtain skilled welders  Tie-up with technical training institutions and develop students from now onwards with a guaranteed employment after 3 years Question #4: Ask the candidate to calculate the breakeven number of barrels of crude oil and years to break-even assuming no over runs. Additional Information: Provide the following data when asked 

Data

Current – Without Over-runs

Project Cost (Fixed Cost in dollars)

$5 billion

Estimated production capacity (Barrels per day)

100,000

Average price per barrel of refined crude (Dollars)

$38

Variable cost of production per barrel of crude (Dollars)

$10

Variable cost of refining per barrel of crude (Dollars)

$13

Break-even quantity (Barrels)

To be calculated by candidate

No. of years to break even (Assuming 330 days of operation per year) To be calculated by candidate Possible Answer: Costs = Fixed Cost + Variable Cost = $5 billion + ($10 + $13) per barrel * X barrels Revenues = $38 per barrel * X barrels To break even, $5 billion + $23 per barrel * X barrels = $38 per barrel * X barrels Break even quantity X = $5 billion / $15 = 333.3 million barrels approximately Number of years to break even = 333.3 million barrels / (100,000 barrels per day * 330 days per year) = 10 years approximately Question #5: Ask the candidate to calculate the break-even number of barrels of crude oil to be produced and years to break even with Cost over runs Additional Information:

Data

New Scenario – With cost over runs

Project Cost (Fixed Cost in dollars)

$6 billion

Estimated production capacity (Barrels per day)

100,000

Average price per barrel of refined crude (Dollars)

$38

Variable cost of production per barrel of crude (Dollars)

$10

Variable cost of refining per barrel of crude (Dollars)

$13

Break-even quantity (Barrels)

To be calculated by candidate

No. of years to break even (Assuming 330 days of operation per year)

To be calculated by candidate

Possible Answers: Break even quantity = $6 billion / ($38 – $10 – $13) = 400 million barrels Number of years to break even = 400 million barrels / (100,000 barrels per day * 330 days per year) = 12 years approximately Question #6: Assume there are cost over-runs. If the company wants to break-even within the same time frame we found with no over runs, what should be the price per barrel of crude oil? Additional Information:

Data

New Scenario – With cost over runs

Project Cost (Fixed Cost in dollars)

$6 billion

Estimated production capacity (Barrels per day)

100,000

Average price per barrel of refined crude (Dollars)

To be calculated by candidate

Variable cost of production per barrel of crude (Dollars)

$10

Variable cost of refining per barrel of crude (Dollars)

$13

Break-even quantity (Barrels)

333.3 million barrels (no cost overruns)

No. of years to break even (Assuming 330 days of operation per year) 10 years (no cost over-runs) Possible Answer: Let the price per barrel to break even within 10 years = P, solve $6 billion / (P – $10 – $13) = 333.3 million, P = $41 Question #7: Ask the candidate for a 30 second summary of his/her findings.

MDP to Re-design Supply Chain for Metal Processing Factory Case Type: reduce costs; operations strategy. Consulting Firm: AlixPartners second round job interview. Industry Coverage: Freight Delivery, Shipping Services; Transportation.

Case Interview Question #00520: Our client Madison Dearborn Partners (MDP) is a private equity firm specializing in leveraged buyouts of privately held or publicly traded companies, or divisions of larger companies; recapitalizations of family owned or closely held companies; balance sheet restructurings;

acquisition  financings; and growth capital investments in mature companies. The firm was founded in 1992 and is based in Chicago, Illinois, United States. Madison Dearborn Partners has recently bought a metal processing company in Chicago. You have been hired as a consultant by MDP and they want you to re-design the supply chain system for the finished goods produced by this Chicago metal processing company. How will you proceed? Additional Information: The client MDP has three specific objectives. 1. They are concerned only about the outbound finished goods 2. Minimize Cost 3. Reliable product delivery Suggested Structure: For a supply chain design and optimization case like this one, the candidate should be willing to explore the following issues:  Number of Products  Number of customers  Product demand  Transportation  Inventory  Any warehousing decisions  Location of factory  Location of customer  Flow of information The case is essentially a transportation case. After initial structuring, the interviewer should ask the candidate to focus only on the transportation part. The focus here is to minimize transportation cost and ensure reliable product delivery. Provide the following additional information when asked. But if the candidate doesn’t get here in a few minutes, just provide this anyway. 1. The metal processing company has only one factory located in the suburb of Chicago. 2. The metal processing company has only one customer located in Denver, Colorado.

3. All shipments are direct from the Chicago factory to the Denver customer. The distance between the two is 1,000 miles. Question #1: At this stage, ask the candidate about various modes of transportation possible for the product delivery. Possible Answers:  By air: air cargo, airfreight  By train: railroad freight  By truck: truck freight The client finally settles in for trucks. Additional information: 4. The customer demand is 40 truck loads per week. 5. Currently the Chicago factory is using truck freight companies which charge about $3 per mile for a truck load. Question #2: The client is now considering reducing this transportation cost. How can the client do it? Possible Answers:  Negotiate with contractors to reduce cost.  Look at alternate freight companies.  Buy trucks on its own.  Lease trucks.  Shift the plant location to Denver. Question #3: The interviewer should insist on the leasing option. Ask the candidate what information he/she wants to decide if leasing is a cheaper option. The candidate should ask for the following things:  Leasing cost per truck – $1750 per month  Labor cost (Salary of truck drivers) – $0.5 per mile  Fuel cost – $0.25 per mile  Vehicle maintenance cost – $0.4 per mile  Freight loading and unloading cost – negligible  Assume 4 working weeks per month  The lease provider pays the fleet insurance cost  A truck can make only 2 round trips per week. What is the total monthly cost of transportation and what is the expected saving in transportation costs? Possible Answers: 1. Current system by using Truck Freight Company Monthly demand = 4 weeks * 40 truck loads per week = 160 truck loads per month Total miles traveled = 160 * 1,000 miles = 160,000 miles (Truck freight companies charge only one way) Total cost = 160,000 miles * $3 per mile = $480,000 2. New system with leasing truck

Weekly demand = 40 truck loads. We need 20 trucks as a truck can make only 2 round trips a week. Leasing charges per month = 20 trucks * $1,750 per month = $35,000 Labor cost per month = 20 trucks * 16,000 miles per truck per month * $0.5 per mile = $160,000 (Here an empty truck returns from Denver to Chicago after making a delivery to the customer and for each delivery a truck travels 2,000 miles. Thus in a month a truck travels 2,000 miles per round trip * 2 round trips per week * 4 weeks = 16,000 miles) Fuel cost per month = 20 trucks * 16,000 miles per truck per month * $0.25 per mile = $80,000 Vehicle maintenance cost = 20 trucks * 16,000 miles per truck per month * $0.4 per mile = $128,000 Total cost per month = $35,000 + $160,000 + $80,000 + $128,000 = $403,000 3. Expected saving = $480,000 – $403,000 = $ 77,000 per month Question #4: What other ways can you suggest to make more money with this new transportation system of leasing trucks? Possible Answer: A good candidate will identify the opportunity to make use of the empty-return trucks from Denver to Chicago. They may suggest transporting any raw materials or sell the space to some other truck freight company. Question #5: What are the negatives of leasing trucks? Possible Answers:  Managing the truck drivers  What if the vehicle meets with an accident?  We have to maintain accounting systems for all the expenses. Question #6: Will this new transportation system ensure reliable delivery? Question #7: Ask the candidate to provide a 30 second summary of his/her final recommendation.

AGCO Optimizes Operations Among Different Business Units Case Type: operations strategy, supply chain optimization. Consulting Firm: Diamond Management & Technology Consultants (now PwC Advisory) second round job interview. Industry Coverage: industrial equipment; agriculture, farming. Case Interview Question #00489: Your client AGCO (NYSE: AGCO) is an agricultural equipment manufacturing company based in Duluth, Georgia, United States. As a leading global manufacturer of agricultural equipment, AGCO offers a full line of tractors, combines, hay tools, sprayers, forage and

tillage equipment, which are  distributed through more than 2,700 independent dealers and distributors in more than 140 countries worldwide. Although geographically constrained, the client company has been very aggressive in developing new products. However, with sales of USD $7 billion, AGCO is showing slow growth (3%) recently. Although this growth rate is higher than industry average, the client is concerned about the slow growth rate. They would like you to find out why this is happening and to come up with a solution. How would you go about the case? Possible Answer: Interviewer: What immediate thoughts come to your mind (or should have) when you heard the problem statement of this case? Candidate: (As soon as I heard “Geographically Constrained”, I wanted to know what exactly that meant) What exactly do you mean by “ geographically constrained”? Interviewer: In this case, the client’s product mix is such that 80% of products are growing at 2% and 20% of the products are growing at 20% – 30%. Why do you think this is happening? Candidate: Are there too many businesses? Interviewer: There are ten discrete businesses. Candidate: What kind of products are we talking about here? Interviewer: For simplicity, let us assume all ten product lines are agricultural equipments, such as farming tractors, combines, hay tools, sprayers, forage tillage equipment, etc. Candidate: Are there any synergies or complimentary functions existing between these businesses? Interviewer: Right now each business has its own Product Development, HR, Accounting, Sales and Marketing, Finance, etc. How would you prioritize the issues and what information do you need to filter out? Candidate: I would like to assess the possibility of finding synergies between the different businesses, thus lowering costs. The biggest benefits would come from synergies in procurement. Interviewer: OK, what additional information do you need?

Candidate: How much would costs go down if we converge the operations of each of the business units? At this point, the interviewer provided some visual representation of the supply chain, and some data about the client’s business units. The whole supply chain was laid out as follows: Raw Materials => Manufacturing => Distribution => Sales => Customers The candidate further discussed possible operational convergence among different business units:  Procurement, Capacity utilization, Distribution  Product Development, HR, Accounting, Sales and Marketing, Finance, etc. Interviewer: We are running out of time. Can you summarize your findings for the client? Candidate: OK. It is very clear that the main problem in this case is that the different business units of the company do not coordinate with each other. The existence of different product development, procurement, manufacturing and distribution departments suggests that there is a huge opportunity to optimize operations. Given the fact that one of the main problems faced by this company is that 20% – 30% growth rate comes from only 20% of the products and the rest 80% of the products are growing only at 2%, coordination of the product development groups is a must. It is very likely that 80% of the products manufactured by the company are those that the market is not ready as yet. Interviewer: Great! I think you did a good job. Let’s stop here for now.

Perot Systems to Provide Free Cell Phones to Project Teams Case Type: operations strategy. Consulting Firm: Ernst & Young (EY) first round job interview. Industry Coverage: software, information technology (IT); consulting, business services. Case Interview Question #00488: Perot Systems is an information technology (IT) services provider and IT consulting firm headquartered in Plano, Texas, United States. The company maintains offices in more than 25 countries around the world, including the United States, Europe, India, China and Mexico. Perot

Systems provides information technology services in the industries of health care, government, manufacturing, banking, insurance and others. Perot Systems is especially strong in

health care industries with services such as digitizing and automating medical records. Before its acquisition by Dell Inc. (NASDAQ: DELL) in 2009, Perot Systems was a Fortune 1000 corporation with more than 23,000 associates and 2008 revenues of $2.8 billion. While on his way to the airport, the CEO of Perot Systems wants to know whether it is a good idea to provide mobile phones to all company employees. What do you think? Possible Answer: Candidate: What kind of IT consulting service does your firm Perot Systems specialize in? Interviewer: Mostly software development and programming, also some functional consulting. Candidate: How big is your firm? Interviewer: We currently have about 23,000 employees. Candidate: What about the size of management? Interviewer: About 15% are managers and the rest are programmers and software engineers. Candidate: It doesn’t seem to be a good idea to provide mobile phones to programmers, because the utilization may not be worth the investment. There is little reason for programmers on the job to use cell phones as the nature of job involves more of online transferring of data (if the need arises) and less of real-time information exchange. Moreover for the rest of the employees, I would suggest doing a detailed cost-benefit analysis. Costs should be based on assumptions of attractive corporate rates decided with cellular companies and the average usage expected. Benefits will have to be decided by taking into account expected increase in revenue by providing real-time access to information. Interviewer: Don’t you think you may need some more information before you can jump to that conclusion. Candidate: Hum…let me think about that for a few moments. Where do your consultants do most of their work? Interviewer: They work mostly at the client site. Candidate: What kind of resources do they use while at the client site? Interviewer: They work on computers. Sometimes they use the PC’s on the client site, and sometimes they use their own laptops. Candidate: Do they mostly exchange voice communication or data? Interviewer: Mostly data, but they often call into headquarters or one another for help solving tricking technical issues.

Candidate: I see. Do the client site locations have telephones? Interviewer: Actually they do, but very often there is only one telephone line in a given room, even though 4 or 5 consultants may be sharing that room. Candidate: Do the consultants have network connections? Interviewer: Sometimes they do, though often they are forced to tie up the phone line because they have to dial out to get a connection to transfer data. Well, we’ve run out of time. What do you think we should provide mobile phones to all of our employees? Candidate: Well, based on what you have told me, I would say that there is a definite need for some employees to have cell phones. Instead of purchasing cell phones for all of your employees, you might explore the possibility of providing phones to project teams on a needed basis to keep cost down. Interviewer: Do you see any problems with that solution? Candidate: Well, you would have to set up some sort of system where you track usage by employee just to ensure some accountability. If you could do this, then I think this solution would lead to higher productivity at a minimum cost. Interviewer: Great. Thank you for your time!

Robert Joss to Step Down As Dean of Stanford GSB Case Type: operations strategy; HR, organizational behavior. Consulting Firm: Cornerstone Research first round job interview. Industry Coverage: education & training services; non-profit organization. Case Interview Question #00483: Let’s imagine that you are Robert L. Joss, the Philip H. Knight Professor and the Dean of the Graduate School of Business (GSB) at Stanford University. The Stanford GSB offers a general management Master of Business Administration (MBA) degree, the Sloan Master’s

Program and a Ph.D.  program, along with a number of joint degrees with other schools at Stanford University. The school is the number one ranked business school in the United States by U.S. News & World Report for year 2012.

After 10 years of service as the dean of Stanford GSB, you will step down at the end of the current academic year. Right now you are interviewing new deans and thinking about the kinds of candidates you want to replace you. What issues would you want them to be aware of? Possible Answer: This is a multi-part business strategy case, which challenges candidates to think strategically about what kinds of things a CEO of a large corporation, or in this case the dean of a prestigious business school might need to be concerned with. The interviewer should look for evidence of top-down thinking. The candidate should be able to look at a large number of issues and identify which ones would be a priority for the dean of a business school. The interviewer will look out for the small minded issues or complaining by the candidate. Candidate: I would like the potential dean to consider some of the following issues:  Fundraising issues, building the endowment.  Attracting and keeping top academic talent to the faculty.  Updating and maintaining the physical plant.  Updating or maintaining the technology infrastructure.  Student satisfaction levels and curriculum requirements.  School administration issues and organization structure.  Overall school strategic objectives and direction. Interviewer: Good. You have now been placed on the Dean’s Committee. Pick one of the issues you talked about that you believe to be a priority and talk about how you would recommend framing the issue. Candidate: I would like the potential dean to explore the following issue first: student satisfaction levels and curriculum requirements, because this directly impacts the business school’s ranking and therefore, the quality of students that are attracted to the school, which in turn affects the school’s competitive position among recruiting firms. Student satisfaction levels – Are the students happy with the following:  Faculty  Career services  Staff  Facilities (student lounge, computer labs, library etc.)  Academic competency of their peers  Interpersonal interaction with their peers Curriculum requirements – I would like the potential dean to explore the following:  Is the recommended core course satisfactory?  Are the electives meeting students’ requirements?  Are the faculty members competent? After exploring the above, I would expect the potential dean to recommend solutions to fix problems such as – “An inefficient bidding system for firm interviews”.

Interviewer: Great. Going one step further: you have been asked to head up a committee to create the change you just talked about. How would you go about setting up a timeline and project plan? What are some the actual steps you would take to make this change? At this stage, the interviewer is now looking to see if the candidate can get out of the strategic clouds and get their hands around an actual strategy. The more specific the plan, the better. Don’t just repeat what you have already said, but talk about action steps, time-tables, and the structure of the team.

Marriott Upgrades Service to Compete with Four Seasons Case Type: operations strategy; business competition. Consulting Firm: Roland Berger Strategy Consultants second round summer internship interview. Industry Coverage: tourism, hospitality, lodging. Case Interview Question #00465: Four Seasons Hotels, Inc. is a Canadian-based international luxury, five-star hotel management company. Travel + Leisure magazine has consistently ranked the hotel chain’s 84 properties among the top luxury hotels in the world. The company is the operator

of  luxury hotel brand Four Seasons Hotels and Resorts. Marriott Hotels & Resorts is Marriott International’s flagship brand of full service hotels and resorts. The company Marriott International (NYSE: MAR) is a worldwide operator and franchisor of a broad portfolio of hotels and related lodging facilities. Recently, Marriott wants to compete directly with Four Seasons. What should they do? What recommendations would you give to Marriott to help them compete with Four Seasons? Possible Solution: This is a quick mini case the interviewer can throw in the middle of a fit or personal experience interview. The candidate should brainstorm as many possible actions as possible, and think about what Marriott needs to change in its operations. As this is a pure hypothetical business case, provided here is only one of many possible answers. Essentially, Marriott must upgrade its facilities and staff service levels to compete with Four Seasons. 1. Facilities Hotel rooms would have to be refurbished, lobbies would need redecorating, and restaurants and bars would need to be improved. 2. Service

Additionally, the hotel service levels would have to be improved. Staff would require additional training and their compensation schemes would have to be changed as well.

What Operating Metrics to Look at in a Consulting Firm? Case Type: operations strategy. Consulting Firm: Roland Berger Strategy Consultants second round job interview. Industry Coverage: consulting, HR, business services. Case Interview Question #00464: Imagine you are the CEO of Roland Berger Strategy Consultants, a global management and strategy consulting firm headquartered in Munich, Germany with 45 offices and 2,000 consultants in 33 countries. What operating metrics would you want to look at to make sure you

were doing a good job? Possible Answer: This is yet another quick mini-case to throw into the middle of a full 45-minute session of second round interview. Just like every other case, there is no right or wrong answer here. The candidates will have to present a sound framework and discuss their reasons for selecting the particular components. As a professional services company, a consulting firm like Roland Berger Strategy Consultants must satisfy two key constituencies: its customers and its employees. Additionally, it faces strong competition from other similar firms, so the metrics the firm should look at could be broadly assigned to these three areas. Some possibilities include: 1. Customer Focused  Customer Retention  Increased Profits  Number of New Customers 2. Employee Focused  Employee Retention  Employee hire rate 3. Competitor Focused  

Competitive Bids Won Growth

Kaiser Permanente Develops Strategy to Manage Suppliers Case Type: improve profitability; operations strategy. Consulting Firm: Aon Hewitt 2nd round job interview. Industry Coverage: healthcare: hospital & medical; insurance: life & health; non-profit organization. Case Interview Question #00462: Your client Kaiser Permanente is a large not-for-profit HMO (health maintenance organization, an organization that provides managed care for health insurance contracts in

the United States as a liaison with health care providers such  as hospitals and doctors). Based in Oakland, California, United States, Kaiser Permanente is made up of three distinct groups of entities: the Kaiser Foundation Health Plan and its regional operating subsidiaries; Kaiser Foundation Hospitals; and the autonomous regional Permanente Medical Groups. As of 2006, it is the largest managed care organization in the United States with 8.7 million health plan members, 167,300 employees, 14,600 physicians, 35 medical centers, and 431 medical offices. Recently, the client Kaiser Permanente is interested in developing a strategic approach to managing all their suppliers. Their suppliers fall into four broad categories:  non-medical supplies (printers, printing paper, copy machines, etc)  non-medical services (janitorial, lawn-care, etc)  medical products (Q-tips, MRI machines, sutures, medical devices, etc)  medical services (specialist doctors, nurses, etc) The client wants to know: how should it manage each of these four suppliers groups to improve its overall profitability. How would you approach this case? What recommendations will you give to the client? Possible Answer: 1. Note to Interviewer This case is purposefully ambiguous. Let the interviewee struggle to grasp what you are looking for. The key is to let the interviewee determine different approaches for segmenting their supplier base. A tip to give to the interviewee is to ask “What do you want from your suppliers?” The interviewer should help guide the interviewee towards the solution outlined below.

2. Possible Solution This case is a difficult one because it doesn’t provide any details. How a large health insurance firm manages its suppliers will vary according to what the firm seeks from its suppliers. A simple way to frame this problem is to think about what a firm wants from its suppliers. An obvious desire is to obtain the lowest cost possible, not the lowest per unit price, but the total costs after taking into account defect rates, late deliveries, etc. Another key variable you would desire that is less obvious is revenue enhancement. This means that suppliers can provide value-added supplies/services that help you gain some competitive advantage in the market place. What you end up with is a simple 2×2 matrix shown in Figure 1. The text within each corner describes how you would want to manage the different groups of suppliers.

McDonald’s to Offer More Health Food Options in the U.S. Case Type: new business; operations strategy. Consulting Firm: LEK Consulting first round job interview. Industry Coverage: restaurant & food services. Case Interview Question #00459: McDonald’s Corporation (NYSE: MCD) is the world’s largest chain of fast food restaurants, serving around 64 million customers daily in 119 countries. Headquartered in the Chicago suburb of Oak Brook, Illinois, United States, McDonald’s primarily sells hamburgers,

cheeseburgers, chicken,  french fries, soft drinks, breakfast items, shakes and desserts. The company recently has also expanded its menu to include healthier food options like salads, wraps, smoothies and fruit.

Recently, in response to changing consumer tastes, McDonald’s is thinking about offering more health food in the United States. In broad terms, it is considering three strategic options:  Roll out dedicated health food restaurants  Integrate a health food menu into their current restaurants  Forget the entire health food business all together How would you analyze this business opportunity for McDonaId’s? Which of the three strategic options would you recommend McDonald’s to adopt? Possible Answers: The point of this strategy case is to have the interviewee analyze various related issues McDonald’s must face in regards to its decision and then conclude with an actual recommendation. The interviewer’s task is to assist the interviewee, drive towards a solution, and then assess the thoroughness of the recommendation. There are many ways to solve this case. However, the interviewee should be able to discuss the following basic analyses during the interview: 1. Market Definition What is health food? McDonald’s defines health food as all non-red meat-based food, for example food from restaurants like Kenny Rogers Roasters, Koo Koo Roo, Boston Market, etc. *Red meat in traditional culinary terminology is meat which is red when raw and not white when cooked. In the nutritional sciences, red meat includes all mammal meat, e.g. pork, beef, lamb. 2. Competitor & Industry Analysis Determine overall industry attractiveness based on the degree of competitor rivalry and each competitor’s positions. Clearly there are a large number of competitors but they share common weaknesses: they are smaller than McDonald’s; they lack the multi-unit management expertise of McDonald’s, and lack the advertising resources of McDonald’s. 3. Market Analysis Determine overall market demand for health food. The market analysis can be conducted in numerous ways: Look at sales of other health food restaurants; look at sales of health food products in grocery stores; survey current McDonald’s customers; survey potential customers. 4. Company Capabilities What core competencies and transferable skills would McDonald’s be able to leverage from its traditional fast food business? Clearly McDonald’s will be able to leverage its marketing skills and administrative skills (e.g. overhead activities). However, McDonald’s will need to develop an entirely new set of operational routines. They will need a new “assembly line” , identify new “raw materials”, and probably new equipment.

5. Costs and Risks What are the costs (fixed and variable) in establishing a chain of new health food stores. What are the risks involved? Both of the potential “Go” options involve high costs and risks. Key issues to consider: How does any new initiative affect McD0nald’s brand name. The firm will most likely have to develop a new name if they launch a full scale chain of health food stores. What additional costs will be incurred with each option: training, new store locations, menu design costs, store start-up costs, etc. 6. Recommendation Clearly there is no right or wrong answer here. Rather it is more important to be able to discuss what approach you would take and what you would want to analyze. Moreover, your recommendation should be based on the logic you discussed during the interview. If you said a new brand name was important, you shouldn’t recommend that McDonald’s should launch a health food menu in its existing stores.

Poland Springs Create Private Label Bottled Water for Walmart Case Type: new business; operations strategy. Consulting Firm: LEK Consulting first round job interview. Industry Coverage: consumer products; food & beverage; retail; general merchandisers. Case Interview Question #00457: We are going to look at a growing trend in the Consumer Packaged Goods industry. Our client in this case is Poland Springs, a bottled water company. As a wholly owned

subsidiary of Vevey, Switzerland based food and beverage company Nestlé (SIX: NESN), Poland Springs sells its bottled water in the United States only. Named after the Poland Spring in Poland, Maine, it is one of the top selling bottled water brand in North America. Recently, a major retailer Walmart (NYSE: WMT) approached our client Poland Springs with a new business proposal. Walmart wants to create a private label version of our client’s product. In other words, in addition to our client’s bottled water which they already carry, they want the client to make an additional, lower-priced bottled water which will be sold under their own brand label, e.g. Walmart Great Value Bottled Water. Should the client take Walmart’s offer? What are the pros and cons of doing this? Possible Answer: As it was given in the interview setting, this case question is more of a situation analysis and brainstorming exercise rather than a business case to drill down on and crack. As such, there may be many more pros/cons beyond what is listed in the example solution below. In general, it is important to be structured and try to put the pros and cons into clear buckets. Considering the 3C’s framework (e.g.

impact on Customer relationship, Company’s operational issues, and Competitivedynamics) could also be helpful in this case. 1. Pros  Improved relationship with a powerful merchant. Client may get better shelf space and better terms on other products Walmart purchases from us as well as cross-marketing arrangements.  Larger production may achieve better economies of scale in both fixed costs (cost/unit to produce, especially if certain operational procedures are synergistic) and variable costs (delivery, distribution, etc) in addition to the potential to negotiate better terms with suppliers due to larger orders.  Huge potential revenue growth for our client company. Lower price but potentially very high sales volume. 2. Cons  Cannibalization of our own private label’s water sales (cost/benefit analysis of whether the volume of Wal-Mart’s brand will make up for it).  Possibility of lessening our power with the merchant as a supplier if private label takes off and supplants our own product.  Higher costs (Fixed costs such as additional plant requirements due to potential capacity constraints – will the investment be worth it?)  More complicated distribution adding additional SKU (stock-keeping unit) into the mix.  Potentially a different market for our product which does not work synergistically with our marketing focus, for example if our product is about prestige and image, we would be catering to a different customer segment with a lower priced product.

Whole Foods Grocery Store to Cut Tomato Price by Half Case Type: operations strategy; pricing. Consulting Firm: Cornerstone Research final round job interview. Industry Coverage: food & beverage; retail. Case Interview Question #00455: Whole Foods Market (NASDAQ: WFM) is a grocery and health food supermarket chain based in Austin, Texas, United States which emphasizes organic products. Whole Foods Market only sells food products that meet its self-created quality standards for being “natural”,

which  the store defines as: minimally processed foods that are free of hydrogenated fats as well as artificial flavors, colors, sweeteners, preservatives. The chain has 304 store locations as of May 2011. Our client is the manager of a Whole Foods grocery store located in a Boston neighborhood. Recently, their supplier of tomatoes has made them an offer: the Whole Foods grocery store will get their tomatoes for half the price if they agree to sell the tomatoes to the consumers for half the current retail price. The

store manager wants advice on whether to take the offer and reduce the price of tomatoes or not. If so, then how? If not, why not? Possible Answers: This business strategy case can be solved with several different structures (profit = revenue – cost, 3C’s, etc) or with no framework at all. This should be up to the interviewee. 1. Customers The first issue to probe should be whether customers are price sensitive. If not, there will be little change in the market. To judge whether customers are price sensitive, the candidate should mention points like socio-demographics of the neighborhood, past price promotions on tomatoes or similar products. Once the candidate brings it up, the interviewer should mention that there is information on past promotions and ask the candidate what information he/she needs. It turns out that in past promotion on tomatoes with a significant price reduction, the same store sold 3 times more tomatoes. Additional Information:  Current retail price of 1 pound of tomatoes is $4, it will go down to $2/pound.  Current costs are $3 per pound, will go down to $1.5 /pound.  Current sales are 300 pounds/day, are expected to go up to 900 pounds. Calculation: Therefore, current profit is 300 * ($4 – $3) = $300; post-promotion profit will be 900 * ($2 – $1.5) = $450. However, it is important to look at total customer profitability with the possibility of Cannibalization of other products. We need to differentiate between new customers that might be attracted to the store due to the promotion and current customers who will switch or increase their purchases. It is not illogical to assume that customers have a fixed budget for grocery shopping. Therefore, current customers will just switch from buying other products to tomatoes. The question is then, which products have the higher margins. The Whole Foods grocery store is expected to sell more complementary products and less substitute products. What are the some of those products? (The interviewer should push the candidate to see how he/she thinks under pressure). — Pasta, ketchup, olive oil, garlic bread, cucumbers, peppers, onions, tomato sauce, curry, etc (assume whatever cost/benefit on complementary/substitute products). Based on the past promotion, we have the following information on the existing sales and gross profits as well as the effect of the promotion on tomatoes sales on some other products: (data table to be handed out to candidate)

Product

Sales ($) Costs ($) Gross Profits ($) Change in profit due to promotion (%)

Pasta

500

425

75

100

Olive Oil

300

240

60

80

Ketchup

750

525

225

-60

Tomato Sauce

400

300

100

-80

Curry

250

175

75

-75

Garlic Bread

200

140

60

75

Calculations: Therefore, due to cannibalization gross profits will be reduced by $100 ($75 * 100% + $60 * 80% – $225 * 60% – $100 * 80% – $75 * 75% + $60 * 75% = -$103.25). So the change in profit after factoring in cannibalization is still $450 – $300 – $100 = $50. In addition, the promotion may also act as a customer acquisition strategy by attracting new customers to the store. Therefore, it makes economic sense. 2. Competition: Analysis of competitor is required here in this case. Assume there is only one carbon copy competitor in the neighborhood. No influence from outside the region (would someone drive 30 minutes to save $3? probably not!) What will the competitor do? Does it buy from the same supplier? Will the supplier offer the same deal to the competitor? (Assume only one regional supplier that gave the offer to our client first; need to sign exclusivity. Any sound and logical analysis can be accepted) 3. Company/Store  Stocking – at the end of the promotion customers may be stocking up 1 week supply of tomatoes.  Shelf space – if candidate has done too well so far, and you have extra time, you might want to discuss store shelf space. For simplicity’s sake, assume that the Whole Foods grocery store only sells fresh fruit and vegetables that arrive in the morning and sell until the end of the business day. The tomatoes are then displayed on shelves (assume no warehouse). Calculation:  Currently:  Shelf space for tomatoes: 20 sq ft.  Sales: $60/sq ft, Profit: $15/sq ft  Post-promotion:  Shelf-space: 40 sq ft have to be added (as sales become threefold)  Sales: $30/sq ft, Profit: $7.5/sq ft There are obviously no free shelves, so some space has to be created. Therefore other products will be taken off the shelves. If on average profits of store are $20/sq ft, what does it mean? Does average mean anything? Why not take the lowest profitability items – e.g. shoe polish items which only make $2.5/sq ft.   

What will be the profitability in this case?  Used to be: 20 sq ft * $15/sq ft = $300  Now post-promotion: 60 sq ft * $7.5/sq ft – 40 sq ft * $2.5/sq ft = $350 Recommendation to client: Based on our analysis, the Whole Foods grocery store should take the offer and reduce the price of tomatoes (Note: The candidate should summarize the three sources of increase or decrease in profit discussed before, and make a final go/no-go recommendation; if he/she does not, the interviewer should ask him/her to do so). In the recommendation a good candidate should also talk about the following: 



When answering “how to go about doing it”, it is important that the candidate recognize the importance of advertising to ensure that customers will be aware of the promotion (low awareness => no change in sales). Advertising is also necessary to attract new customers. Again, it is necessary to check the capabilities of the Whole Foods store to take on more customers in terms of space, labor etc.

The Hartford to Develop 5-year Strategic Plan for Operations Case Type: operations strategy; organizational behavior. Consulting Firm: Ernst & Young (EY) final round job interview. Industry Coverage: Information Technology (IT); Financial Services; Insurance: Life & Health; Insurance: Property & Casualty. Case Interview Question #00442: The Hartford Financial Services Group, Inc. (NYSE: HIG) is a Fortune 500 company headquartered in Hartford, Connecticut, USA. As one of America’s largest investment and

insurance companies, the Hartford Group is a leading provider of life insurance, group and employee benefits, automobile and homeowners insurance and business insurance, as well as investment products, annuities, mutual funds, and college savings plans. With 2010 revenues of $22.4 billion and more than 1.5 million plan participants, the company’s earnings are divided between propertyand-casualty operations and life operations. Your client is the Operations division of The Hartford Group. The Operations division is worth $1 billion and is responsible for 50% of the operating expenses of the overall company. The general perception within the company is that this Operation division: 1. Costs too much 2. No one knows exactly what they do 3. The division should be outsourced

You have been asked to produce a 5-year strategic plan for the company, and specifically to address the problems posed by this Operations division. How would you go about the case? Additional Information: (to be given to you when asked)  The other two divisions of the company are Investments and Insurance.  The Operations division is responsible for accounting, documents, data warehousing, hosting services, information technology (IT) and communication infrastructure like telephones, etc.  The Operations division does a significant amount of back end processing for the other two divisions, such as processing sales.  The Operations division does not directly generate revenue, but the work they handle is essential for the company.  There are no regular meetings between the representatives of the three divisions. Possible Answer: This operations strategy case relates to improving the internal processes of a financial services and insurance company. Focusing too much on the company’s products, customers, competitors or other such external factors is unproductive. It should be clear that the Operations division is a cost center and the goal should be to determine the worth of this division, and help the other two divisions realize this worth. Communication management is the key in this case, and this can help integrate the division into the larger company. The Operations division’s employees should first be able to quantify the benefits of the work they do and explain it to the rest of the company. Suggestions may include working towards activity based costing, where the cost bucket can be identified and associated with the other divisions. Also, the formation of cross-functional teams could help all three division better understand their needs and how best to meet those needs. In addition, the Operations division may be able to identify some standard tools, technologies or other backend functions, which can be outsourced to shave off some costs. Finally, proposed solutions should include an expected time frame, since the case asks for a 5-year strategic plan.

Rockwell Collins to Diagnose Problems in Supply Chain Case Type: operations strategy, supply chain optimization; reduce costs. Consulting Firm: A.T. Kearney final round job interview. Industry Coverage: aerospace & defense. Case Interview Question #00387: The client Rockwell Collins Inc. (NYSE: COL) is a large international aerospace and defense company headquartered in Cedar Rapids, Iowa, United States. It primarily

provides aviation and information technology (IT) systems and services to governmental agencies and

aircraft manufacturers. As a major defense contractor, Rockwell Collins procures materials from multiple vendors and supplies products to US Department of Defense (DoD) and other governmental agencies. The company’s main products are aviation related communication electronics and systems (broadcast transmitters, shortwave transmitters, radio receivers, transceivers and systems, etc). Recently, the client is experiencing problems meeting its cost and delivery commitments. The company’s Vice President of Operations has sought the help of A.T. Kearney to help diagnose problems in the company’s supply chain. Question #1: At a very high level, how would you breakdown the supply chain for such a company? Possible Answer: For a supply chain design the candidate should be willing to explore the following  Number of Products  Number of customers  Product demand  Transportation  Inventory  Any warehousing decisions  Location of factory  Customer location  Flow of information Question #2: We find that Material Costs are 75% of total costs, and fixed overhead and labor are 25%. What are some ways of reducing Material costs? Possible Answer:  Consolidate suppliers or Change suppliers.  Bring in a procurement team that can negotiate better prices with suppliers.  Launch a continual improvement program with vendors (like what auto maker Toyota does). Question #3: The following is how the buying process of our procurement department works: 1. Engineers submit a set of designs to a procurement representative. 2. Our procurement representative makes an RFP (Request for Proposal) to several suppliers. 3. Suppliers return with bids and the procurement representative chooses a supplier. Let us take the example of buying printed circuit boards. 1.

What are the criteria that you would consider to select suppliers? Assume that we’re buying about 10,000 circuit boards per month. 2. What is the advantage of a large supplier vs. small? Which would you go with?

3.

Would you choose suppliers with their own fabrication units or those that outsource manufacturing? Should the fabs be within the US or outside? What are the security implications? Question #4: What are some ways of making our procurement process more efficient? How can we cut inventory and inbound delivery costs? Question #5: You bump into the VP in an elevator. Give him a 20 second elevator pitch of all your findings and hypotheses.

Emerged from Bankruptcy, Tronox Looks Ahead to the Future Case Type: operations strategy; business turnaround Consulting Firm: Deloitte Consulting final round job interview. Industry Coverage: chemical industry. Case Interview Question #00372: Tronox Inc. (NYSE:TRX) is an Oklahoma City, Oklahoma based chemical company involved in the manufacturing of titanium dioxide (TiO2) pigment. The company is the world’s fifth-largest producer and marketer of titanium dioxide pigment, holding a 12% market share

worldwide and serving customers in more than 100 countries. Titanium dioxide is an inorganic pigment that whitens and brightens hundreds of consumer products – from paint and plastics to paper, cosmetics and more. Tronox produce a full range of superior pigment grades for a variety of end-use markets. In January 2009, Tronox filed Chapter 11 bankruptcy protection. Recently, it successfully emerged from bankruptcy. They have hired you as a consultant to help them chart a future course of action. First of all, they want you to evaluate if their current products are viable. How would you go about this case? Possible Answers: This case tests how a candidate thinks about the fundamentals of turning a company around. For this purpose, the candidate has to understand the current situation first. Suggested Structure: Overall the candidate should explore if the client can generate positive cash flow in the future. If not the candidate should look at options to generate cash flow. Step 1: Understand what led to client’s bankruptcy. Step 2: Understand the products and the industry. Step 3: Understand the current product economics and future projections. Step 4: Propose recommendations – Both for top line and bottom line improvement. 1. What led to bankruptcy?

Simple the company was not making money and was incurring losses. This led to default of loans and resulted in Bankruptcy. They refinanced the company through some means and have very limited funding. Additional funding is not possible. 2. Products and Industry: Their main product is Chemical X (titanium dioxide TiO2) and the by-product is Chemical Y (iron sulfate, FeSO4).  Both products X & Y are commodity chemicals.  The chemical industry related to Chemical X & Y is highly cyclical and now it is at its peak.  The industry related to Chemical X & Y is highly competitive.  Client’s customers are other large chemical companies. 3. Product economics: data to be given only when the candidate asks. 

Annual production capacity: Totally 500,000 tons per year. Price of Chemical X = $95 per ton. Price of Chemical Y = $75 per ton. The end product from the manufacturing process is a combination of Chemical X and Y. It has to be separated into X & Y. Each ton of the end product they produce has X and Y in the ratio of 1:1.5.  Client’s manufacturing facility has a fixed cost of $25MM per year.  Variable costs of production are $60 per ton of end product. At this stage of the case, the candidate has to perform the following calculations to quantify whether client’s current products are viable.   

Annual production volume in tons 500K Chemical X in tons

500K * (2/5) = 200K

Chemical Y in tons

500k * (3/5) = 300K

Price of X per ton

$95

Price of Y per ton

$75

Fixed Cost

$25MM

Variable Cost per ton (X&Y)

$60

Annual Revenue

200K * $95 + 300K * $75 = $41.5MM

Total Annual Cost

$25MM + $60 * 500K = $55MM

Net Annual Profit

$41.5 – $55 = – $13.5MM

From the calculations, it is clear that the current products are making losses. The interviewer should provide the following response for questions that may arise. Can we raise prices? – No, we cannot. Industry is very competitive and also is at the peak of its cyclical. This means that going forward the price will only go down.  Can we exit the business? – It is a possibility. But push the candidate to look at ideas to turn the company around. Question #2: How can we increase the top line? Possible Answer: Assume that both Chemical X and Y have a saturated market and the market has several competitors. With the cyclical demand and a downward trend expected in the future it is difficult to sell more. So the candidate has to look for opportunities to launch new products or services. 

Question #3: What new products or services can the client company launch? Allow the candidate to explore options creatively. However, the candidate should remember that funding to invest is very limited. Possible Answers:  Products that can be manufactured without major modifications to existing plant set up.  Products that can be bought from manufacturers outside the US but can be sold in US.  Products that will hedge against cyclical business issues. For example: if the chemical is used in the cooling systems of machines, then the customers would like the manufacturer to administer the chemical into the cooling system on a routine basis. The client can look at any such opportunities.  Start a chemical distribution company.  Start a chemical research laboratory and provide R&D advice to other large chemical companies.  Move into chemicals retailing business. Question #4: It is obvious that the cost of production for client is higher than the price they can command. What are the areas they should look at to reduce costs? Possible Answers:  Improve plant efficiency  Reduce raw material cost by changing suppliers or negotiating costs down  Reduce the fixed costs like corporate workforce and so on  Outsource manufacturing to low cost countries  Reduce energy costs by improving the energy efficiency Question #5: Why can the client not exit the business? Sell this business to someone else and start something that is profitable? What issues do you foresee if the client has to do so? Possible Answers:  Who will want to buy this business?  How much can the client sell the business for?  Issues with laying off people. Question #6: Ask the candidate to give a final recommendations to client.

Marathon Petroleum to Offer Co-branded Credit Card Case Type: marketing/operations strategy; new product. Consulting Firm: American Express 2nd round job interview. Industry Coverage: oil, gas & petroleum industry; financial services. Case Interview Question #00356: Your client Marathon Petroleum Corporation (NYSE: MPC) is a U.S. based oil refining, marketing, and pipeline transport company with its headquarter in Findlay, Ohio. Marathon Petroleum operated as a subsidiary of Houston, Texas-based Marathon Oil Corporation (NYSE:

MRO)  until July 1, 2011. Marathon Petroleum has its own credit card that many of its customers use at Marathon gas stations when they fill up their car with gas. Recently, other oil companies have begun entering into partnership agreements with major banks for co-branded credit cards (i.e. Shell and CitiBank, BP and Chase). The client Marathon wants to know if it should do the same. Either way, should Marathon continue to offer its own credit card? Possible Solution: Basically, the 3C’s — Customer, Company, Competitors, seems like an appropriate framework to solve this “launching a new product” type of cases. 1. Customer Candidate: Do current customers value the Marathon credit card – do they use it? Interviewer: About 20% use it and like it. Candidate: Would a co-branded credit card draw new customers or increase customer loyalty? Interviewer: Yes, people indicated they’d be more likely to visit our gas stations if we had the co-branded credit cards. However convenience is the main consideration when selecting a location. 2. Company Candidate: Is the client company currently performing the credit card processing in-house – is it profitable? Interviewer: Yes, they’re doing it in-house but are not making much money on it. Candidate: Would the co-branded credit card processing be out-sourced – does it offer attractive margins? Interviewer: That’s an option; the margins appear to be better if processing is out-sourced.

3. Competitors Candidate: How many competitors have co-branded credit cards? Interviewer: Only two (Shell, BP) so far, but others are looking. Candidate: If they don’t move now, would their choice of partners be limited? Interviewer: Probably. Recommendation: Offer the co-branded credit card, but out-source it. Given that existing customers use and like the company card, they should continue offering it, but attempt to shift customers to use the co-branded card. The data processing operations for its credit cards should also be out-sourced. They should research marketing (co-marketing) tactics that could improve the attractiveness of the credit card to the company’s customers – with the intent of increasing the number of customers. Additional Issues to discuss about:  Consider the marketing research that had been conducted.  Explore the focus of the client company (they made no money trying to run credit card processing).

Scotch Whisky Johnnie Walker Develops Marketing Strategy Case Type: operations strategy/marketing; business competition. Consulting Firm: Fidelity Business Consulting (FBC) Group 2nd round job interview. Industry Coverage: tobacco & alcohol; food & beverages. Case Interview Question #00351: Your client Diageo plc (LSE: DGE, NYSE: DEO) is a global alcoholic beverages company headquartered in London, United Kingdom. It is one of the world’s leading manufacturers of distilled spirits with a number of major brands. Its brands include Smirnoff (claimed the

world’s best-selling vodka),  Johnnie Walker (claimed the world’s best-selling scotch whisky), José Cuervo (claimed the world’s best-selling tequila), Baileys (claimed the world’s bestselling liqueur) and Guinness (claimed the world’s best-selling stout). You have been hired by the marketing department of Diageo to help them develop marketing strategy. Diageo’s Chief Marketing Officer has asked you to address two specific questions: What should the focus of their advertising budget be? What sized advertising budget would be needed to follow through on your strategy?

Possible Solution: The first thing that I did here was to separate this case into 2 distinct parts: I first addressed the focus question and then addressed the budget issue. I used a basic 3 C’s framework to address what the focus of the advertising strategy should be. Customers: The first thing that they need to do is determine what their target segment is: What market segment is the client in? I discovered that they were competing in the premium scotch segment. Once they determine this segment, they can do a detailed analysis of the demographics of this segment. For arguments sake I assumed that the scotch was Johnnie Walker Black and the target market was affluent males age 30+. Then they would need to do analysis of advertising media to determine how to reach this segment, i.e., print-ads in selected magazines, billboards, sponsorship of sporting events (golf, for instance). Once they had data on what media effectively reaches their target segment, they would decide exactly what mix of media to use, and the specifics of the campaign to address the qualities that they wanted to associate with their brand (status, taste, distinguished, etc.). Market research could benefit the client in terms of determining what qualities they wanted to stress in their ad campaigns. Company: After I completed the analysis of the potential customers, I looked at the company as well as the industry: Who are the major players in this industry and where are we in the market? I found out that currently we were the #3 brand with a little less than 20% market share, the #1 brand had a little higher than 20% and the #2 brand was a little lower than #2. The cost of advertising was not something I got too deeply into. I just noted that advertising was a significant portion of the cost structure of premium scotch brands (over 25% of sales). Another point to note is that because of the high margins on this product, the more volume that a brand sold, the more resources the company would have to invest in advertising. There is a certain minimum scale that is required to be able to profitably advertise in this market. Competitors: The third issue that I addressed was what the competition was doing: How is advertising related to market share? How much do the competitors advertise? I learned that last year, the #1 player had spent a lot of money on advertising and increased market share, while the #2 player spent nothing and lost market share. We spent a little more than nothing and increased market share a little. The distant #4 player also spent nothing on advertising. Recommendations for Client: Based on the above analysis, I hypothesized that there was a pretty positive correlation of ad spending and sales volume increases. Once I outlined what the focus of the advertising strategy should be, I

addressed the budget issue. I proposed three alternatives to determining our advertising budget for next year: 1. The first was to match the #1 competitor to attempt to keep pace. 2. The second was to increase ours a little relative to our historical (last year) ad spending. 3. A third way would be to do a detailed cost analysis of our proposed strategy, and actually quantify what it would cost to have the number of ads and sponsorships in the types of media that we determined would satisfy our strategy.

P&G Considers Consolidating Purchasing Efforts Case Type: operations strategy. Consulting Firm: L.E.K. Consulting 2nd round job interview. Industry Coverage: household goods & consumer products. Case Interview Question #00337: Procter & Gamble (P&G, NYSE: PG) is a large diversified Fortune 500 multinational corporation headquartered in downtown Cincinnati, Ohio, United States. P&G manufactures a wide range of consumer products and household goods with nearly $79 billion dollars in

sales in fiscal year 2010. The company’s operations are categorized into several Business Segments:  Beauty segment  Grooming segment  Health Care segment  Snacks & Pet Care segment  Fabric Care & Home Care segment  Baby Care & Family Home Care segment In Procter & Gamble, each business unit currently operates autonomous purchasing departments. Recently, however, the Vice-President of Procurement of P&G is considering consolidating their purchasing efforts within the company. He has hired you as a consultant to advise him on the consolidation plan. What are the considerations of consolidation? What would you recommend P&G’s VP of Procurement to do? Possible Solution:

This case is a tough one to use a framework for, I basically applied a cost-benefit analysis to see what benefits there would be in consolidation, and what costs the client would incur. Candidate: In P&G, how much synergy would there be in consolidated purchasing, i.e., how much of the same types ofproducts do each division purchase? Interviewer: Although each division orders specific materials, there is significant overlap in some divisions. Candidate: How does purchasing work currently? Interviewer: Each division has their own purchasing department, and materials are purchased and delivered to each of the 13 divisions’ central warehouses. Candidate: Where are these warehouses located geographically? Interviewer: They are all in the U.S., with 9 on the East Coast, 1 in Texas, 1 in Chicago, and 2 in California. Recommendations for Client: I recommend that the client consolidate the purchasing department if the benefits outweigh the costs: 1. Benefits: Economies of scale from larger purchasing quantities. Reduction in number of buyers, managers. Closing of some East Coast warehouses (there can’t be a need to have 9!). Reduction in administrative costs (billing info, secretarial support, etc.). Buyers may be able to establish closer relationships with vendors (we now order bigger quantities) and become more knowledgeable about products they are purchasing. 2. Costs:     

Making the materials purchased uniform may be a problem for some divisions, i.e., order products with features that division 1 and 2 need but not that divisions 3 and 4 need – features that might make the materials more expensive than they would otherwise be if divisions 3 and 4 bought them on their own.  Distribution costs may be prohibitive, if warehouses were closed. Overall, I anticipate the benefits would outweigh the costs, but would have to perform the analysis to determine what recommendation I would make. 

HSBC Bank Strives to Improve Customer Service Quality Case Type: operations strategy; organizational behavior. Consulting Firm: KPMG Consulting first round job interview. Industry Coverage: banking; financial services.

Case Interview Question #00331: Suppose you are working for the internal Strategic Group of HSBC Holdings plc (LSE: HSBA, NYSE: HBC), a global banking and financial services company headquartered in Canary Wharf, London, United Kingdom. The bank has around 7,500 offices in 87 countries and

territories  across Africa, Asia, Europe, North America and South America and around 100 million customers. As of June 2010, it had total assets of $2.418 trillion. Recently, HSBC Bank is facing a increasing number of problems with the quality of its customer service. Its customers are complaining about long waits on the phone, being transferred to multiple branches, separate accounts requiring multiple calls, etc. The customer service function is currently performed at each individual branch. The questions to you: How can HSBC Bank’s customer service be improved? What strategic plan would you recommend? Possible Solution: This strategy case question is rather open ended. No standard framework can be easily applied into the case. The following is only one of many possible ways to approach this case. 1. Start with diagnosis of the current customer service operations:  How is each call handled at customer service center?  What do customers complain about?  Why did this mistake happen?  Are there enough customer service representatives? 2. Do benchmark analysis across the different branches within the entire HSBC Bank:  Do all the branches see the same ammount of complaints?  Are there any branches that are doing better than others?  What are the best pratices? 3. Do the same analysis but benchmarking against other competing banks:  Do competing banks experience similar situations?  How do they handle their customer service? 4. Benchmark against other industries, which are conducting similar customer service operations, for example, telecommunications industry.

Johnson Controls to Boost Its Return on Net Asset by 5% Case Type: operations strategy; reduce costs. Consulting Firm: Mercer Management Consulting 2nd round job interview. Industry Coverage: manufacturing; electronics.

Case Interview Question #00325: The client Johnson Controls Inc. (NYSE: JCI) is a manufacturing company based in Milwaukee, Wisconsin, United States. It is a Fortune 500 diversified, multi-industrial company with more than 142,000 employees in 1,300 locations across six continents. This case involves

only Johnson  Controls’ European operations. The client’s European subsidiary is a $1B (revenues) company with 10 manufacturing sites throughout Europe. Its various products include thermostats, room climate controls, air conditioning units, switches for microcomputers, and other piece parts for assembly, such as automotive interiors, car seats and batteries, etc. The distribution channels that the company currently uses are the following:  Wholesalers (i.e., local mom and pops for do-it-yourself home projects)  Outlets (who sell to contractors)  Construction companies (used mostly for hotels and restaurants)  OEM (original equipment manufacturer, including switches)  Other manufacturing companies (used for control processes, refineries). One of the client company’s measures of operating performance is “return on net asset”, and right now the company would like to increase this measure from 17% to 22% in two years. The CEO and Chairman has retained you to help him accomplish this ambitious goal. What information would you gather? How should the client increase its “return on net asset” by 5%, from 17% to 22%, in two years? Possible Solution: I started with a market overview, then moved to the product line, and then the three C’s. Detailed discussions are presented below: Candidate: First of all, I would like to have an Overview of the Market. What does the market look like? Interviewer: Every family of products that the client offers is its own market, but nevertheless, each is experiencing average growth; they are relatively mature markets. The client has two primary competitors. Candidate: (I then switched to the client company’s Product Line). Can you tell me more about the product line? How are they different from the competition? Is the range of products appropriate for the market? Are all of them profitable? Is the product range too broad, or too narrow? Interviewer: The products do not differ very much from the competition. Some products are more advanced than others; but they are all high-quality products. The product mix seems to be appropriate: we make margins on all of our products, and the range seems to be what the market wants. However, what does differentiate the client from its competitors is the client’s service: it focuses on quick delivery. The client guarantees that orders arrive within 24-48 hours.

Candidate: I now wanted to find out more about the quick delivery. How is the quick delivery operation administered? Where are the facilities for delivery vs. production? Interviewer: The client’s ten production facilities are located in four countries: Germany, Scotland, France and Switzerland. Anyone of these manufacturing plants delivers all over Europe. (The client just recently focused its plant operations to achieve economies of scale.) When products are manufactured, they are delivered to local affiliates, which store the inventory at sites closer to the customers. This allows the client to offer quick delivery despite highly variable demand. Unfortunately, even after the recent focusing of plant operations, the company has not seen an increase in market share. Assume for now that the client cannot achieve additional profits by further consolidating its production facilities. Candidate: (At this point, I switched back to my original structure of the three C’s: Company/Costs). What are the costs associated with providing the quick delivery? Interviewer: The operating costs are actually not a big impact on the client’s cost structure. What is important, though, is the cost of holding stock at the local affiliates: the working capital tied up at these sites is estimated at $300 million. Candidate: (Next, I moved to Competition of 3C’s). Did competitors provide these services? How did their costs compare? Interviewer: We don’t have exact information on the competition; the client believes that it is spending more than others due to this working capital issue. Candidate: (I finally arrived at Customers). What are the customer needs and preferences? How highly do they value the quick delivery? Interviewer: Through a targeted analysis, the client determined that customers were willing to give up time on delivery: the most important thing to them was consistency of delivery, rather than speed alone. Customers are willing to wait more like a week or so if they know it is going to arrive on time. Candidate: Aaah! The client was providing an expensive service that was not highly valued by the customer. Problems with this strategy were: 1. Working capital/inventory that was tied up was very costly. 2. Competitors did not have this cost. Interviewer: Excellent! So, what would be your recommendations for the client? Recommendations for Client: Based on the above analysis of the case, I recommended the client to cut its operating costs by taking the following measures: 1. Change its delivery policy to 7 to 10 days instead of 2-3. 2. Centralize delivery by reducing the number of local stock affiliates. 3. Reduce costs associated with working capital levels.

Interviewee’s Comments: This is basically what the client ended up doing, as per the consultant’s recommendations. It turned out that the client saved about $200 million in working capital. The company saved 10% of total assets by reducing its stock sites from 10 to 5. Note that you should not get caught up with the original measure of an increase of 5% on “return on net assets”; I dug into the issue of the quick delivery/costs, as opposed to doing a detailed quantitative analysis.

Ford Kills Competing Models in Fear of Cannibalization Case Type: business competition; operations strategy. Consulting Firm: A.T. Kearney 2nd round job interview. Industry Coverage: automotive, motor vehicles. Case Interview Question #00319: The client Ford Motor Company (NYSE: F) is a multinational automaker based in Dearborn, a suburb of Detroit, Michigan, United State. Ford is the second largest automaker in the U.S. and the fifth largest in the world based on annual vehicle sales in

2010.  In addition to the Ford and Lincoln brands, Ford Motor Company also owns a stake in Mazda in Japan and Aston Martin in the UK. Ford’s former UK subsidiaries Jaguar and Land Rover were sold to Tata Motors of India in March 2008. In 2010 Ford sold Volvo to Geely Automobile and Ford discontinued the Mercury brand after the 2011 model year. Ford is a major OEM (Original Equipment Manufacturer) auto manufacturer in the US. The company is extremely vertically integrated. It has a large number of nameplates that it fears are cannibalizing their own products. You have been hired by the CEO of Ford to come up with a strategic plan to address this issue. What do you propose they do? Possible Answer: I proposed that we look at three things:  The competitive landscape in each of client’s product segments.  The customer segments and how our client’s vehicles are positioned in these segments.  Our client’s cost position, and whether or not it can be improved through a portfolio rationalization. 1. Competitive landscape Candidate: How competitive is each of out client’s product segment? Are we competing against ourselves in some segments? Interviewer: Yes; competition is intense in each segment. Our individual products compete against our own products and those of our competitors. However, the relative competition differs for each segment.

As a follow-on comment, I added that I would want to further analyze the segments, in order to determine what the competition is and how these vehicles are positioned. This could lead to information regarding which segments we would want to target. 2. Customer segments and product positioning The first piece of analysis that I did for the customer segments and product positioning was to draw a number of 2 X 2 matrices with a number of different attributes (speed, price, fuel efficiency, safety, etc.). Once I did this, I hypothesized that there were more than one product in each of the segments. I highlighted this as a serious problem. I mentioned that this might have happened because through the evolution of the company, and that it may have been a combination of a number of formerly independent brands. The main problem with this is that the consumer may be having tremendous difficulty in differentiating the brands. I theorized that therefore some automobile models might still be around because they always have been, without much thought about optimizing the portfolio to maximize company profits. Once I analyzed this, I proposed two ways to eliminate models that overlapped within the same segments: kill one of the models/brands, leaving behind the more profitable brand; keep both models/brands, but invest in advertising and product improvements such that consumers would be able to differentiate the two and thus place the second vehicle into a different segment. I mentioned that this second option might be difficult, as there is significant inertia in changing people’s minds about a brand that has been a certain type of vehicle for a very long time. I suggested that perhaps the best way to do this would be to keep the most profitable vehicles in each segment that we are in, and eliminate duplicate brands.  

3. Costs The consolidation of models could lead to significant cost savings, as fewer number of models (with greater volume of each) will lead to greater scale efficiencies in purchasing, production, engineering, advertising, etc. Recommendations for Client: Get rid of competing models in the same segment in order to better serve customer segments, reduce costs and reduce competition. The use of matrices was a hit!

GM Research Lab to Prioritize Products Development Case Type: operations strategy. Consulting Firm: McKinsey & Company 2nd round job interview. Industry Coverage: electronics; automotive, motor vehicles. Case Interview Question #00310: Your client is the president of GM Research Lab, which is essentially an electronics Research & Development (R&D) think tank for General Motors Company (NYSE: GM). Headquartered in Warren, Michigan, the GM Research Lab is a very innovative place which employs

about 100 scientists  and engineers who buy components and create prototype innovative products for cars. This place is bursting with energy, and they already have many great products to their credit. For instance, they have already developed a prototype car autopilot navigation system. Similarly, they discovered that many of the road deaths due to car accidents are preventable. The problem is that medical services don’t reach there in time either because they don’t get to know about the accident at all or they don’t know exactly where it happened. In most such accidents, the victims either lose consciousness or are decapacitated. So they have created this device which is a combination of GPS, airbags and cell phones. The way it works is that on car accident impact, the airbags inflate, and the cell phone calls 911, pinpointing the directions using the GPS (think of GM OnStar, Ford Sync, BMW Assist, Toyota/Lexus Safety Connect, etc). These guys are full of ideas, and some of the projects under consideration are night-vision for cars (using F-14 technology), backing sensors and so on. However, their costs range is high. The fundamental issue is that the engineers are like kids in a candy shop. They are doing very exciting work and love their jobs. However, there are random projects going on all over the place. The president wants to streamline the operations and focus on 10-20 projects rather than the more than 100 that are going on right now. The president has asked you to help him address the following questions: How will you prioritize the projects, deciding which ones to focus on and which ones to abandon? How will you convince the engineers, who are basically innovative brains and do not understand business, of your decision? Possible Answers: Interviewer: This is a case on which I worked on about 8-10 years ago. I like this because it is different from most usual problems. How would you go about it? Candidate: Interesting case. So, to prioritize the projects, I will probably start by the basic economics, focusing on theprojects that are likely to bring in the most profits. Thus, some of the factors I will look at are:  Probability of successful completion of the projects  Probability of an automobile manufacturer buying the technology for commercialization  Demand and price elasticity in the market  Speed to market because money has a time value  Practicality of the idea in general. To convince the engineers of the decision, I would focus on the following issues: 

Speed to market

 Ease of selling  Practicality of mass-production  Share of royalty  Profits: Success of the company, and individual success Interviewer: Okay, interesting. Let’s take these one by one. You said that you want to prioritize based on the economics. How do you get a handle on the expected profits? Candidate: I will proceed in a structured manner, starting off with Profits = Revenues – Costs (or expected profits = expected revenues – expected costs). We can find these figures by subjectively assessing probabilities of success in different durations and to different levels and calculating the expected monetary values. I am assuming that the costs aspect could be as follows: I would probably not want to break this down into fixed and variable costs as you said the company is engaged in creating prototypes. Thus, variable costs are wiped off the slate since quantity produced is equal to one. They only have R&D costs which can be split into human resources, material and overheads apportioned. The human resources costs would likely follow a similar pattern, and would depend on the time taken in project completion. To deal with the revenues, I will use the simple equation: Revenues = Price x Volumes.

Please understand that since we are dealing with prioritization, I am looking at revenues, costs and profits for individual projects rather than the entire company. I guess we can have some estimate of the volume by looking at the product life cycle (see Figure 1). At prodcut launch we would have only the innovators and the early adopters and we’ll have to cross the chasm before being able to reach an early majority. Normally, I would think, the only role costs play in pricing is to make sure that at the volume and pricing, costs including R&D and fixed+variable costs of mass production are covered. The pricing is typically based on what the consumer is willing to pay for it. To be able to make a better analysis of the revenues, both from the prices perspective and the volume perspective, I will need to understand how this firm earns its revenues. Can you help me with that? Interviewer: Sure. The company has good relations with an automobile manufacturing company to which it sells its ideas. The automobile company hands it a royalty check based on estimated revenues

depending on how they can price it. They also have the option of earning the royalties over 5-6 years based on actual revenues. Candidate: So that rules out cost-considerations, pretty much? Interviewer: It does. Candidate: Is it fair to assume that volumes lose much of its relevance as the automobile company will fit the product in all cars of a particular model. Interviewer: That is a fair assessment. Candidate: Since pricing for end-consumer seems to be an important factor here, I’d want to explore what the end-consumer is willing to pay for the product. Interviewer: And how would you go about doing that? Candidate: By conducting some market research. Maybe even conducting a conjoint analysis on automobiles with different products, and prioritizing on the basis of that? Interviewer: What is conjoint analysis? Candidate: Conjoint analysis is a market research technique wherein the customers are presented with products with different attribute options, and their responses are analyzed to figure out which attribute options they prefer and how much they value each. Ummm…but yeah, there’s a problem that to conduct this analysis we need prototypes, and here we want to do the analysis before the prototype is created. I guess that could be solved by paper prototyping or giving out the product idea in some way. Interviewer: Hmmm….but this method assumes that consumers know about their preferences. The engineers would argue that since these are very innovative products, consumers don’t have enough information to develop sensible preferences before they can actually use them. Candidate: Perhaps we could try using an economic-value based pricing model then. Interviewer: What does that mean? Candidate: It essentially means we find out how much economic benefit the consumer gains from using the product. That economic benefit is then split between the consumer and the manufacturer. Interviewer: Okay, so how do you gauge the economic benefit a GPS has over a $5 map?

Candidate: I guess we could look at the average time saved, put it against the average worth of the consumers’ time. Ummm….I guess we are not reaching anywhere with this approach. Could we possibly compare the products to similar products produced by competition? Interviewer: For normal products you can do reference pricing. But, as I said, these are highly innovative, completely new products. There is nothing you can compare any of these new products to. Candidate: I guess the only way left is to look at the products sold in the past or selling at present. I suppose it should be possible to determine price-elasticities of demand for different categories of products. For instance, demand would normally be less elastic for safety products. In other words, people would be willing to pay more for products that they perceive as making them safer on the road, just like they are normally willing to pay more for kids’ pharmaceuticals.

Interviewer: Right! That’s exactly what we did. We are almost out of time here, but let me tell you what we did in this case. Following the same logic as you did, we concluded that there are different attitudes that people have towards different types of products. We plotted all their products on two axes – value and frequency of use. We found that there was a pattern like this (Figure 2): The products that are infrequently used but are perceived as having high value are safety products – in fact you wish you never have to use them. The GPS-Airbag-Cellphone combination I mentioned earlier falls in this category. Then there are products that are useful, but are used relatively frequently. These are convenience products like GPS. Then there are items that are less useful but are very frequently used….like power windows. These, we called utility items. Invariably, products in the top-left corner were more successful. That’s where we took the leap of faith, and said that in general people are willing to pay more for safety products, and are more likely to buy them. We recommended that the client company prioritize their products on the basis of this chart…based on where the proposed product falls on this chart. Which, of course, they did. Candidate: Wow! This was indeed a very different and interesting case. I thoroughly enjoyed it. Interviewee’s Note: This business strategy case mainly involves process optimization and prioritization. It was an extraordinary case in several ways. For one, the interviewer spoke more than I did, which is very uncommon for a case interview. Then, the way the case is set up, applying traditional frameworks to crack it takes up a lot of time, though at the end you do reach close to the answer as I did. Lastly, this is the only McKinsey case without numbers that I have seen.

What is Biggest Challenge for a New University President? Case Type: operations strategy; organizational behavior. Consulting Firm: Bain & Company first round job interview. Industry Coverage: education; non-profit organization. Case Interview Question #00304: Suppose you have just been appointed to be the new president of the University of Pennsylvania (or the university the candidate is currently attending) by the Board of Trustees. What do you think the biggest challenge facing you might be? How would you address this

challenge? Note to the Interviewer: The objective of the interviewer should be to take the job candidate through a series of steps and see how she/he can tackle the various levels. The steps can range from identifying a critical issue, to breaking a problem into component parts, to finally identifying one or more solutions. Step 1: Big picture thinking. The interviewer lays out the facts and asks the candidate to articulate the critical issues facing the business. “Big picture thinking” requires a focus on the key issues and not a “laundry list” of every problem facing the business. The interviewer then asks why the candidate chose the answer she/he did. Possible Answer: The University has several powerful constituencies: undergraduates, graduate students, professors, alumni, government and community. However, the University doesn’t have unlimited resources, and therefore it must prioritize where it spends its dollars. Long term success for the University is driven by figuring out what the most important areas are, and at the same time ensuring that all constituents feel they are being treated fairly. Step 2: Problem solving logic The interviewer asks the candidate to lay out all the elements of the problem: What factors should be considered? Question #2: How should you prioritize your limited resources? Possible Answer: Think about what drives the University’s success: reputation, economics, quality of students, quality of research. Understand what “focusing resources” really gets you. Each factor has a different set of implications/impact. Step 3: Focus on value

The interviewer then lets the candidate choose which avenues to pursue, leaving the discussion open ended: without suggesting focus, the interviewer determines if the candidate is instinctive about which path to choose and then asks why the candidate selected this path. Question #3: Which opportunity do you think will have the biggest impact? Possible Answer: Reputation. If you have a good reputation it can influence other areas. Reputation can be a long term asset. Also, it is probably the hardest to fix once it is broken. Step 4: Depth and breadth—business intuition After getting the recruit to focus on a key area, the interviewer asks the candidate some probing questions about how (s)he would analyze that area. Question #4: Okay, so how would you improve the University’s reputation? Possible Answer: I would focus on improvement in two areas: • Promote activities that build reputation • Eliminate factors that negatively impact reputation Discussion then continues down both of these paths with specific improvement ideas. Step 5: Results orientation The interviewer returns to a key area of discussion, asking the candidate how (s)he would implement his/her solution. The interviewer could pose a tricky or hostile client situation and ask how the candidate would get results in a difficult environment. Question #5: Well, those seem like pretty sound strategies. How exactly would you implement those ideas? Possible Answer: We will need to start with a PR blitz, so I would encourage several articles to be written about “The New University” in the popular press. Also, I would heighten the pressure on professors to publish research articles.…(The answer continues with other implementation ideas).

Symantec Turns Down PC Maker’s Product Bundling Offer Case Type: operations strategy; mergers and acquisitions. Consulting Firm: Bain & Company first round job interview. Industry Coverage: software, information technology (IT); computers. Case Interview Question #00286: The client Symantec Corporation (NASDAQ: SYMC) is a manufacturer of security software for computers. Headquartered in Mountain View, California, Symantec is a Fortune 500 company and a member of the S&P 500 stock market index. Symantec offers shrink-

wrapped security software application products and has grown over the last few years – mainly through multiple acquisitions. In the recent time, however, their stock price has declined significantly. The sales have declined but their customer service department has shown impressive growth in revenues over the last few months. Recently, the CEO of Symantec has an offer from one of the largest personal computer OEMs (Original Equipment Manufacturers) in the country to bundle his software product with their PC products and he’s unable to decide what to do. You have been brought in to help him analyze the situation and charter the path ahead. What would you recommend? Possible Answer: Candidate: Before I begin analyzing the case, I would like to clarify a few aspects of the case. Interviewer: Sure, go ahead. Candidate: You mentioned that the sales have declined recently but the customer service revenues have shown significant growth. How significant has the decline in sales been? Also, does the company have other sources of revenue besides product sales and customer service? Interviewer: The sales have not declined much but they have been stagnant amidst a growing sector. Customer service revenues have been increasing steadily over the last year or so, though. These are the only two sources of revenue for the company. Candidate: I see. I would also like to understand the type of offer that OEMs have made. What is the revenue sharing model going to be? Interviewer: The OEMs are desktop and laptop computer manufacturers who are offering a preinstallation of trial versions of the company’s software on some of their product lines. In return, the company will have to pay a fixed fee to the OEM for every new pre-installed computer sold and a 20% commission for every user who moves from the trial version and purchases a full version of the software. Candidate: I understand. So, it appears to me that the OEMs’ proposal can help us expand our sales volume, though at lower margins. I am not sure whether this addresses the core issue that we are facing, so I’d like to analyze the current profitability situation of the company. As you mention, the customer service revenues have been rising. Do we have some data on the types of customer service incidents that are commonly seen here?

Interviewer: I do not have specific numbers on customer service requests. However, I can tell you that a major category of service requests are around software configuration issues. Sometimes the company’s software applications provide multiple ways to accomplish the same goals. A significant number of callers seek help on how to navigate different software. Candidate: I see. One of the objectives of software design is to keep it simple and intuitive with in-built easy-to-use help. It appears to me that the company’s products could improve on this dimension. I suspect that some of this is the result of integrating technology from aggressive acquisitions that we have made in recent years. I see broadly three ways to integrate acquisitions. First, the company can simply add an acquired product as a new standalone offering in its product portfolio. Second, the new technology from acquired companies could be utilized to add new features to the existing products of the company. Third, existing products of the company could be replaced by superior acquired products. What approach has this company taken in integration? Interviewer: That’s a good way to think about it. In this case, the company has primarily followed the first two approaches. There has been both a portfolio expansion as well as product enhancements. Candidate: I suspect that the rising customer service revenues actually point to poor integration of the acquisitions. There are two separate aspects to a successful integration. First, the product portfolio has to be rationalized at a technical level. If the company offers multiple offers to accomplish the same goal, it tends to be confusing for the customer. Similarly, products need to have a consistent look and feel so that it is easy for customers to navigate and configure various features. Second, the sales and marketing team needs to be well trained about the company’s evolving product portfolio. They should be able to help customers make good choices for their needs. Based on the type of customer service incidents, it appears that the company has not done very well on either front. Customers are finding it difficult to configure and navigate the company’s software products. Also, acquisitions should have led to an increase in sales. But stagnant sales point to deficient sales and marketing function. I think without addressing these integration issues, it will be futile to pursue the OEMs offer. It isn’t beneficial in medium and long-term to acquire disgruntled customers. Interviewer: So what would your advice to the CEO be? Candidate: I would recommend that the sales force be overhauled. There needs to be a strong mandatory training program for the sales staff to make them fully conversant with the company’s products. I’d also recommend that the redundancy in the product portfolio be reduced so as to alleviate customers’ confusion. A technical push should be launched to make the software look and feel consistent across products. This should improve customer satisfaction, drive repeat business and improve sales. As for the OEMs offer, I would advice to hold off until these core issues are addressed. Interviewer: That seems fair. I think you have identified the main issues in the case and provided crisp recommendations. We’ll close here. Thank you.

Unilever USA Retains PwC to Improve Profitability Case Type: operations strategy; organizational behavior. Consulting Firm: PricewaterhouseCoopers (PwC) final round job interview. Industry Coverage: household goods & consumer products. Case Interview Question #00280: You have been recently hired into the Strategic Change (SC) group, a strategic business unit of PricwaterhouseCoopers (PwC). The Strategic Change group is the strategy thought leader in PwC. The engagement partner in Consumer Products group has come to SC to help

develop a strategy for Unilever USA,  a wholly-owned subsidiary of British-Dutch multi-national conglomerate Unilever Group (LSE: ULVR, NYSE: UN for Unilever N.V., NYSE: UL for Unilever PLC). This engagement has the potential of positioning PwC to Unilever USA and its parent conglomerate Unilever Group for the next five, possible ten years. You have been assigned to this engagement, how would you go about developing a business strategy for the client? Additional Information: The following additional information will be provided to you during the case interview. Client Company Background Unilever USA is a wholly-owned subsidiary of multi-national conglomerate Unilever Group. The conglomerate owns ten companies operating in the U.S. Unilever USA is the largest of the ten. Unilever USA is a consumer goods company distributing well-known branded products through grocery, drug, mass merchant and club channels. With $4 billion in revenues, Unilever USA is one of the top three players in the consumer goods industry in the U.S. The company has been marginally profitable over the past ten years. Last year the company made a profit due largely to an accounting change. Although Unilever USA only operates in the U.S., it owns several manufacturing and distribution facilities around the world to support its production and distribution systems. Unilever USA takes advantage of lower labor costs in Mexico, Canada and Southeast Asia to lower its manufacturing costs. Unilever USA still maintains three plants in the U.S. Because of the over-capacity that Unilever USA has experienced, Unilever USA has negotiated deals with sister companies overseas to manufacture and direct-ship product. Unilever USA has four market segments that operate as profit centers. The market segments are: Personal Hygiene, Consumer Tissue, Soaps and Detergents and Personal Care. Even though the revenues are roughly evenly divided among all four market segments, Personal Care contributes 90% of the company’s profits. In Personal Care, Unilever USA owns the two top branded products, in the other categories the company has the number two brand, and in one segment, number three. Unilever USA has

committed to building a consumer franchise through aggressive advertising and in-store merchandising support. Consumer Products Industry Trends In the U.S., brands are under attack from private labels, who are now competing on both price and quality. Brands are looking to justify their price premiums. The value of being the number one brand cannot be taken lightly. The return on sales of the top brand is almost twice that of the number two brand. The return on sales for the number two brand is twice that of the number three brand. The power of the retail industry in the U.S. has increased dramatically over the past five years. The retailers are driving additional costs upon manufacturers. With established products, retailers are demanding a minimal level of turns per year. With new products, retailers are demanding slotting fees and ever-increasing promotional support. Product managers are forced to achieve current product revenue and market share goals while stimulating demand for new products. Many industry experts feel that there will be consolidation of brands within many of the market segments in which Unilever USA competes and, as a result of this brand consolidation, that Unilever USA will lose critical sales mass and become a major casualty. In the last two years the allocation of marketing dollars has changed dramatically; trade promotion has risen to 40% of total marketing spending, consumer promotion has climbed slightly and advertising has declined. Industry analysts have pointed to Unilever USA’s trade promotion strategy as being the catalyst for the growth in trade promotion in the industry as competitors have been forced to respond. Unilever USA is widely regarded as a retail-oriented company. With a sales force that is twice the size of anyone else’s in the industry, Unilever USA has forged great retail relationships over the years. Unilever USA traditionally had the best order fill rate in the business; however, recently some of the efforts to reduce inventory has caused shortages in key promoted products. Unilever USA Organization Structure There are six Executive Vice Presidents (EVPs) in Unilever USA responsible for functional areas. All the EVPs report to the President, who is also CEO. The Executive Vice Presidents represent Marketing, Sales, Finance, Manufacturing, Engineering and Human Resources. The EVP of Finance has responsibility for financial reporting and analyses as well as managing Procurement, Deployment, Scheduling and Logistics. All the market segment managers report directly to the Executive Vice President of Marketing. Much of the blame for the performance of the company over the last ten years fell on the shoulders of the former president. It was whispered that he was from the “old school” and could not change his ways. The new president of Unilever USA, an American, joined the company six months ago. He was the Executive Vice President of an important European division of a sister company. The conglomerate has always prided itself on being able to leverage its multi-national resources. Unilever USA Current Situation

Venn Teldren, the Executive Vice President of Finance, is considered to be a brilliant man by many in the industry. Born and raised in Europe, Mr. Teldren rose quickly through the organization. However, because of his outspoken nature, he angered enough senior level executives (“showed up” as Mr. Teldren would say) that he has never received a position of president, even though his name is mentioned every time an opening appears. Recently the vice chairperson of the conglomerate responsible for the group in which Unilever USA is a member, sat down with the Unilever USA President and EVPs. The vice chairperson stated that the company needed to improve performance within one year. He offered a couple of scenarios of what the conglomerate was considering in the event that the management failed to improve profitability. Scenario 1: Drop unprofitable brands and reduce the size of the company. Scenario 2: Merge the company with a sister company that has similar distribution requirements and have proven profitability. Recent Initiatives Unilever USA has recently taken part in an industry-wide study called Efficient Customer Response or ECR. The study found that an industry-wide effort to develop more efficient trade practices and delivery systems could save an aggregated $30 billion dollars a year. PwC assisted Unilever USA in this study. All the EVPs agree that there are huge dollar savings that can be achieved with efficiency improvements. Venn believes that the supply chain (i.e., Procurement, Manufacturing, Deployment, Scheduling, Logistics, and Warehousing) can become a strategic advantage for the company if it can outperform its competitors. PwC studies have shown that improvements cannot be made without the input or the support of all the functional areas of the company, especially Marketing and Sales. The EVPs from Marketing and Sales do not always see the Supply Chain as key players; in fact, the EVPs of Marketing and Sales see the Supply Chain as only a vendor to them. Venn knows that the results of the ECR initiative may not be enough to rally support among the EVPs. Venn knows that whatever strategy is accepted needs to define the roles of each of the EVPs and to provide an outlet for each EVP to demonstrate his and her skills. He is also aware that the other EVPs are very conscious of the growth of Venn’s power. Each EVP will initiate a project with the assumption that the architect of the solution to Unilever USA’s current situation will be in position for the next presidency. The EVP of Human Resource has championed the need to implement a whole new way of envisioning the company working together. She has envisioned a flatter organization and has spent years developing studies with another leading consultancy to support her vision. She has a strong supporter in the EVP of Sales. The present EVP of Sales was originally from Human Resources. Her vision has always entailed an extensive re-structuring and re-training effort. The EVP of Engineering feels that the company needs to invest in its new product capability. The strategy is to acquire smaller, regional companies that are producing differentiated products. “We can absorb them

into us and stimulate our new product pipeline,” he stated. “With these new, regionally proven products, we can fill capacity and leverage our distribution and sales strength. I can also energize my area with fresh ideas. It’s win-win, no doubt about it.” The EVP of Manufacturing is sick and tired of hearing that manufacturing is the problem. He points to the fact that they re producing and shipping three times the product they were five years ago with the same number of people they had eight years ago. If things don’t change in other areas, then things won’t change in Manufacturing, other than the inability to support the orders coming in. The EVP of Marketing believes that a combination of re-structuring and acquisition is needed. He wants to reduce the salesperson’s role with the retailer and focus on consumer spending behind a “high quality” message grounded in tangible product benefits across all product segments. He wants to broaden the product mix with new products from acquisition. The Engagement Venn has mentioned to Gary Forstman, the PwC engagement partner, that he is willing to devote the necessary resources in his functional areas to prove out the right strategy to the other EVPs. Venn has also indicated that the company is willing to devote significant resources and capabilities to the right effort. “All the EVPs know,” he said, “that there will be whole-scale changes if the company doesn’t turn itself about.” Mr. Forstman has called Grady Means, PwC Strategic Business Unit (SBU) leader and partner, and said, “This is PricewaterhouseCoopers’ first major engagement with Unilever USA after several years of smaller engagements where we were able to demonstrate our ability to implement solutions. Now we have an opportunity to really shine. The company is re-evaluating its strategic position and has asked several consulting firms to talk to them.” Grady discussed the situation with PwC partner, Michael Hanley, and they agreed that you would be a great person to work on this project. You receive a call from Grady. After exchanging pleasantries, Grady explains the situation to you: “We need some dynamic thinking on this one. I know Venn Teldren from years ago. Venn is going to be all over us if we don’t get this right. What’s important is that we show Venn that we have a vision of where the company needs to go, how the parts fit together and how they are going to get there. What is important is that our analysis is fact-based. We need to be ready to say to Venn, ‘This is the situation, this is the problem, this is the solution and this is step one, step two, step three on what you need to do tomorrow.’ This is a big opportunity for us and I’m counting on you. See what you can come up with by this time next week. Feel free to call Michael or myself with any questions. Okay, talk to you soon.” Specific Questions :  What is your assessment of the present situation?  What are the key areas for change? Why do you believe so?  What do you envision your product to be in a week?  What type of additional information would you want?  What type of analysis do you believe needs to be performed?  Do you have an idea concerning the analytical structure?

What type of framework might you envision for this strategy? What are the key elements you would include in designing a strategy? What are the key elements you would include in implementing a strategy? What is your assessment concerning Unilever USA’s ability to implement a strategy? What are some key performance indicators that you would suggest? What are the key issues between the Supply Chain and other areas of the company (such as Marketing and Sales) that must be addressed?  What are some ways that improvements in the Supply Chain will impact the other areas of the company, especially Marketing and Sales?  How does Supply Chain effect the value of the company’s brands?  What are the risks that the PwC team faces in this engagement? Possible Answers:      

No answer is provided yet. Feel free to share your own answer/solution or any thought to this case by leaving a comment below.

PwC to Develop Business & IT Strategy for Telekinesis Case Type: operations strategy; organizational behavior. Consulting Firm: PricewaterhouseCoopers (PwC) final round job interview. Industry Coverage: telecommunications & network. Case Interview Question #00279: PricewaterhouseCoopers (PwC) has recently proposed on, and appears to have won, a major engagement to create an information technology (IT) strategy for a big telecommunications company called Telekinesis Inc. Telekinesis is modestly profitable, with $131 million

in sales last year and approximately  200,000 customers spread out over four adjacent, mostly rural geographies. PricwaterhouseCoopers has worked for Telekinesis Inc. in the past, but has not done any significant work for over a year and a half. This is PricwaterhouseCoopers’ first substantial engagement with the company. As a consultant assigned to this project by a PwC partner, how would you go about developing an Business/IT strategy for the client? Additional Information: The following additional information will be provided to you during the case interview. Client Company Background The client Telekinesis Inc. was formed in 1992 by executives from four former Regional Bell Operating Companies (RBOCs) and two principals in Silicon Valley technology start-ups. One of the principals is from a start-up company that pioneered a new kind of wireless propagation technology.

Telekinesis was founded on the principle that the current telecommunications industry is populated with companies who are almost congenitally incapable of optimizing their form of organization and culture to meet the competitive challenges of the 1990s. The founders believe that local loop technology, which relies on communication devices which are peers in a large technology community, where every device has a permanent and unchangeable identification, is the silver bullet of the telecommunications industry, and that the RBOCs are not ready or willing to exploit it. Local loop technology (LLT) is considered by RBOC management to be radical, unproven and unreliable. Telekinesis Inc. is modestly profitable, with $131 million in sales and approximately 200,000 customers spread out over four adjacent, mostly rural geographies. Approximately 90% of its sales come from four small local telephone companies. The company’s strategy is to use the operating experience and customer positioning of the four local telephone companies to develop and implement local loop wireless service or LLWS (often pronounced “laws”). The concept behind this service is based on the fact that the current phone companies control wiring to and from a central office facility. This facility is in effect a big switching box. The central office acts like a big hub with many spokes radiating from it. LLWS eliminates the central office and substitutes simple, unobtrusive, premises wireless relay equipment. There is at least one local loop server facility that is somewhat analogous to a central office but not needed to maintain service. The server facility is used to monitor quality and provide a trap for billing. Local loop wireless services are fully integrated. They include telephonic communication as well as cellular, pager, on-demand video, and “highway” services. Highway services permit companies within the local loop to communicate with each other as if they were on a large universal local area network. Computers located in both home and office are immediately interconnected by the local loop. Importantly, there are literally no wires involved in any of these services (except of course for plugging into the wall to get electricity). Physical customer hook-ups are non-existent. Customers are granted access, and services and information are secured through software interfaces in LLWS devices, such as television sets, laptop computers, pagers, etc. Telekinesis has a number of arrangements with software and hardware vendors to create LLWS devices. Understandably, the industry discounts LLWS as another “high tech California fantasy.” Bell Core engineers, while acknowledging the future potential of local loop technology, dispute Telekinesis’s claims that the bandwidth and quality is actually present in production, commercially available products to be installed in the real world. Telekinesis’ doctrine is to completely convert all 200,000 current subscribers of the four local phone companies at once, with no phase in. Each of the four local companies will be converted separately. Telekinesis bought the four local phone companies in order to have large scale pilot sites for local loop wireless services. Telekinesis’ fundamental business proposition is that the changing regulatory landscape will allow it to compete with local Bell telephone companies, providing a higher performance, lower cost alternative to the existing local phone companies for local and long-distance telephone service, paging, cable t.v., and cellular phones.

Telecommunications Industry Trends The early 1980s were a time of turmoil for the telecommunications industry. For the first time in history, AT&T was deregulated and lost its monopoly status. This meant competition for AT&T where none had existed before. Long-distance was the arena of competition. “Telecommunications” includes much more than simply making a phone-call. It encompasses cable television service and network connectivity which brings interactive television, shopping forums, education and information services into the home. The phone lines that the telecommunications companies control enable computers to communicate from remote locations, and can gather information from databases and news services around the world within seconds. The possibilities for profits in this arena are practically limitless, and the sphere of competition is expanding. Up to 1994, only long-distance carriers were in competition, but local calling areas are going to be opened up for competition in the late 1990s. Telekinesis Organization Structure There are currently three business units: 1) residential, which is divided into the “plain vanilla” customers that have only one phone line into the house and no add-ons such as cellular phones, pagers, additional lines, etc. and 2) residential customers who have add-on services and are good candidates for taking advantage of the new technology; and 3) small business. Each of Telekinesis’s business units has a President who reports to the CEO. In addition, R&D and Technology Assurance, essentially a quality management program, also report directly to the CEO. Telekinesis is tightly controlled by the principals who founded the company and all the senior positions just described are held by the founders. There are really no Corporate functional areas such as Finance, Purchasing, Distribution, and Human Resources. These functions exist in the original phone companies as they did before the companies were acquired. An outsider with the title of Chief Financial Officer runs the Corporate functional area. She had a brief tenure as the CFO of an RBOC. The Technology Assurance Group helps support the existing communications and networking infrastructure. Telekinesis Current Situation PricewaterhouseCoopers was retained because of their knowledge of the RBOCs and an audit relationship with the four local phone companies. They were retained by Telekinesis for special start-up services, legal and regulatory counsel and assistance in dealing with obtaining additional venture capital financing. Because of the technology nature of Telekinesis, the PricwaterhouseCoopers Financial Advisory Services partner contacted IT Strategic Services. The Firm has now been asked to deal with the operational dimensions of Telekinesis as it commences detailed tactical planning for LLWS activation. Another management consulting firm is providing some business strategy consulting to Telekinesis. PricewaterhouseCoopers has been asked to propose on three major stages of work: 1) process vision; 2) tactical doctrine; 3) infrastructure and value. These are meant to give Telekinesis “process efficacy.” This is their language.

The current company is, in effect, the combination of the four small southern telephone companies that were acquired and are now operated by Telekinesis. However, except for top management, the vast majority of employees of the telephone companies were retained, as were the administrative and operational support systems. Some of those employees are very excited to be able to participate in this opportunity, but a lot of the old timers are dubious and apprehensive. All telephone company processes and functions are essentially the same as before the acquisition by Telekinesis. Telekinesis concentrated on establishing a simple, “no frills” system for collecting financial and operating information on the telephone companies but did virtually nothing to change the actual operations of the companies. Marley and Cratchet (the two silicon valley entrepreneurs) expect that the consultant selected will be able to bring fresh creative ideas to the process of what they term is “…creating a 21st Century company for a 21st Century business.” Included in their definition of process efficacy is the notion of “enterprise extensibility,” or put more simply put the capability to seamless team with external suppliers in a variety of value-adding, integrative relationships that can be episodic or persistent. Particularly important is the aspect of Telekinesis strategy in which vendors will provide LLWS compatible devices to customers who will pay a one-time $15 fee for the equipment. All four Telekinesis executives expect that the process efficacy initiatives will include information systems and technology strategy and planning. They want the consultant to provide a guaranteed “operational profile” that states that the recommended configuration of hardware and software, costing $xx and operational by 19yy will be able to support the local loop wireless service business. There are four distinct flavours of legacy systems across the four companies. Hardware and software is different, with three of the companies having an IBM mainframe in addition to other computers. Telekinesis installed IMRS on a high end x486 computer to provide financial consolidation and reporting of the four companies. Spreadsheet disks prepared at month end are FedExed to Telekinesis home office in Bernardsville, New Jersey and loaded into IMRS. Specific Questions:  Do you believe you have enough information to develop an Business/IT strategy for this client? If no, what additional information would you require?  What skills would the consulting team need to successfully complete this engagement?  How would you structure the work for this engagement?  What are the risks that Telekinesis faces?  Should PwC guarantee an “operational profile”? If so, should there be any caveats included in the guarantee?  What types of business processes will be needed?  How would you integrate the processes of the four existing local phone companies and Telekinesis?  What kinds of information systems will the company require?

Where can PwC add the most value in the engagement? (i.e., of all the items that Telekinesis requested assistance with, where should we focus?) Possible Answers: 

No answer is provided yet. Feel free to share your own answer/solution or any thought to this case by leaving a comment below.

Century 21 Real Estate to Increase Earnings by 200% Case Type: reduce costs; operations strategy. Consulting Firm: Bain & Company first round job interview. Industry Coverage: property & real estate. Case Interview Question #00266: Our client Century 21 Real Estate LLC is a real estate agency based out of Los Angeles, California. It focuses on managing and renting apartments for short-term contracts and temporary leases: our client rents the entire apartment building for 5 to 10 years, furnishes the apartments and rent each of them to its clients for a short period of time (usually shorter than 1 year).

 Its own clients are usually corporations (that rent apartments for expatriates or temporary transfers) and people from the entertainment industry (movie stars, pop singers, etc). The company currently has $750M in revenues but only $12M in EBITDA (Earnings Before Interest, Taxes, Depreciation and Amortization). Bain was hired to help them increase the EBITDA to $34M. What are the possible recommendations? Additional Information:  Fully-owned maintenance and cleaning services are charging 20% premium over market.  Client does not own properties, they are all long-term leasing contracts.  Decentralized procurement.  There is no price discipline. Possible Answers: Interviewee: (Summarize the case and work on a profitability framework, profits = revenues – costs). I want to understand all sources of Revenue as well as Costs, and figure out room for cost reduction or revenue optimization. I will analyze opportunities for expanding the business as well. I will start with Revenues. I am assuming the biggest revenue source is the rental revenues, right? What do we know about the customer demand? Interviewer: I can tell you that our client has 95% of occupancy rate.

Interviewee: This is very impressive; I do not believe there is room to increase this rate because of the maintenance and turnover lead time. The client seems to have a very efficient sales force, thus volume cannot be increased. What about pricing? Interviewer: This is something I actually looked at. Regardless who the client is, there is no price differentiation at all. Also, at different times throughout the day, a client might be given different prices. Interviewee: Here we already see something to work on. The client should most likely implement a pricing schema which would allow price differentiation. Certainty some of the customers are less price sensitive. Also, our client does not seem to have a robust CRM or standardized processes. The client should perhaps implement systems and review processes which would improve its operation and offer pricing discipline. Interviewer: Right. Anything else to consider? Interviewee: Do we have market share data? I want to analyze the competitive landscape and see room for additional revenue streams. Interviewer: Yes we do. Our client’s 20 major competitors have all together the same market share of our client. Interviewee: Which means our client is the largest player in this industry. Interviewer: Yes it is, and for now it is not considering diversifying the business. Thus let’s focus on costs. What are the major cost elements? Interviewee: The major fixed costs would be rent expenses and real estate taxes. Overhead and marketing expenses are also fixed. Operating staffing is primarily fixed, and so is furnishing. Other than that, there are only variable costs, such as: Maintenance services, cleaning services, utilities, sales. Let me start analyzing the rental costs, which is most likely the major cost element. What do you know about this cost category? Interviewer: Well, I was also very interested to know more about rental costs. This is what I found out. Rental contracts are managed locally by each of the client’s 45 regional offices. They argue that the close relationship with landlords offers them more flexibility with contracts, eg. not paying rent during the summer months (low season). Interviewee: Although it could indeed be true, I would imagine economies of scale are huge in the real estate industry. There are very big real estate owners in this country. The client should think of centralizing rent negotiation for the major markets and perhaps keep some of the local markets with some autonomy. By doing so, our client could also consolidate and reduce overhead expenses.

Interviewer: By doing so we estimated a $5-10M savings per annum. Interviewee: What do we know about the maintenance and cleaning costs? Is it outsourced or internal resources? Interviewer: This is also interesting. Our client has fully-owned maintenance company and cleaning services company. They are both high-end and offer great services. Interviewee: Do we know costs for equivalent services in the market? I would assume there is limited scale in the internal business and the company could save by contracting external services. Interviewer: We do. As I said, the current services are high-end, so our client could save 20% of a $60M budget by outsourcing these services. Interviewee: Perhaps the client should spin off this business and then require lower costs. There is room for $12M savings. Interviewer: Anything else? Interviewee: Unless the client wants to expand into other businesses, we have already accomplished enough. We have already identified opportunities to generate the additional $24M in EBITDA. Interviewer: What are your recommendations? Interviewee: I would recommend a set of actions: Spinning off the maintenance and cleaning service businesses will most likely generate an additional $12M in EBITDA;  Designing a pricing differentiation schema would allow our client to extract a higher margin from some of its customers;  Implementing CRM and new systems will generate cost savings;  Centralizing procurement with major real estate owners will generate cost savings in renting expenses. Interviewer: Excellent! Let’s stop here. Do you have any questions for me? 

Interviewee’s Notes: This is an interesting case, and the interviewer said that she was at that time working with this client. Take the time to structure your thoughts and discuss the case. Besides the framework, the interviewer was expecting a different set of recommendations. Some important elements to be discussed were:  Opportunities for outsourcing  Economies of scale  Customer segmentation  Cost structure (variable vs. fixed)

Emerged from Bankruptcy, Chemtura Seeks Strategic Alternatives Case Type: operations strategy; business turnaround Consulting Firm: Deloitte Consulting final round job interview. Industry Coverage: chemical. Case Interview Question #00255: Our client Chemtura Corporation (NYSE: CHMT) is a manufacturer of basic commodity chemicals, specialty chemicals, polymer products and processing equipment for a variety of industries, with a single-digit market share. The company was formed in 2005 from the merger of Great Lakes Chemical and Crompton Corporation. Headquartered in

Philadelphia,  Chemtura employs about 6,000 people internationally and had sales of USD $3.7 billion in 2007. Currently Chemtura possesses no other competencies or capabilities other than as a chemicals manufacturer. It recently emerged from bankruptcy reorganization and financial restructuring. Having limited access to capital, the client has asked for our help to evaluate the sustainability of their business and recommend some strategic alternatives to their current business model. How would you go about the case? Additional Information:  Prices for client’s chemical products are cyclical and run in seven-year cycles (peak to peak). The client is currently at the top of the pricing cycle.  The primary chemical produced is Chemical X, of which the client produces 100,000 tons annually.  A by-product of the production of X is the production of Chemical Y, which is produced at a ratio of 1.5 Y : 1.0 X.  The current price of X is $150/ton. This will fall to $100/ton in 2008.  The current price of Y is $175/ton. This will fall to $100/ton in 2008.  Variable costs are $50 for every combined ton of X & Y. This will rise to $95 in 2008.  Fixed costs are $20 MM. Question #1: Keeping in mind that the client will be unable to endure any net losses, can the client survive 2008 (which will be the bottom of the pricing cycle)? Possible Answer:  Chemical X Revenue: 100,000 ton * $100/ton = $10 MM.  Chemical Y Revenue: 100,000 ton * 1.5 = 150,000 ton; 150,000 ton * $100/ton = $15 MM.  Total Revenue: $10 MM + $15 MM = $25 MM  Variable Costs: 100,000 ton * $95/ton = $9.5 MM.  Fixed Costs: $20 MM.  Total Costs: $20 + $9.5 = $29.5 MM.

 2008 Net Income = $25 MM – $29.5 MM = – $4.5 MM. Answer: No, the client has a net loss in 2008 and cannot survive the bottom of the pricing cycle. Note: Candidates will often be asked to perform some basic calculations during a case interview. In real client situations, consultants are frequently required to process data and make estimates in real time, without the aid of a calculator or spreadsheet. It is important not only to correctly perform the calculation (e.g., net loss of $4.5 MM), but also to understand the business implication (e.g., the client cannot survive the bottom of the pricing cycle). Question #2: What could be the underlying reasons for the client’s inability (for the first time) to survive the trough of the pricing cycle? Possible Answer: It is important to consider what has changed during this cycle that may not have changed (or changed as dramatically) in previous cycles. One way to approach the question is to evaluate the sides of the revenue / cost equation. Revenue Factors: The trough of the pricing cycle is deeper (lower) than ever before. Production volume is lower than before, thereby not providing enough income to cover the fixed costs. Cost Factors:  

Variable costs have increased to a greater percentage of revenue than ever before. Fixed costs have increased – perhaps b/c of outside issues (i.e. environmental issues) requiring more expensive equipment to produce the same volumes. Note: Questions like this help the interviewer assess the candidate’s understanding of profitability. The interviewer can evaluate the candidate’s approach to breaking down the components of profitability and identifying the underlying factors affecting the client. Having a structured approach to thinking through these factors is critical as it demonstrates logical thinking and ensures the candidate has completely thought through the problem.  

Question #3: Discuss the opportunity and risks associated with each of the following strategic alternatives: a. Acquire / move into a counter-cyclical chemicals business. b. Provide additional products / services relevant to one or more of its key customer segments (i.e. in addition to selling X and Y, sell Products A and B, and perhaps Services C and D). Possible Answer: a. Acquire / move into a counter-cyclical chemicals business: 

Potential Opportunity: To balance the existing cyclical nature of the client’s core products, resulting in a potential scenario where one core product is always near the top of its pricing cycle while the other is near the bottom of its own unique cycle.

Potential Risks: Unlikely to find a chemicals business that is purely countercyclical to the client’s existing business. Industries such as these tend to move in tandem – there may be some lag between various products, but a complete inverse of cycles is unlikely.  To enter such a business fast enough to avoid bankruptcy in 2008 would require an acquisition in addition to any efforts to enter new markets organically. This may stretch current resource capabilities.  Such an acquisition would require capital, which would be difficult to obtain given the dire financial situation of the client b. Provide additional products / services relevant to one or more of its key customer segments. 

Potential Opportunity: Leverage an existing set of relationships to provide a greater share of their chemicals or chemicals-related needs. This would provide the catalyst to enter new businesses that would potentially prop up the client’s income statement while its core business is suffering through the bottom of its price cycle.  Potential Risks: The organization has no skills or capabilities to leverage beyond its core manufacturing competencies. It is unlikely that its leadership would successfully evolve the business away from a volume focus to a relationship / service focus.  Once again, the need to move quickly and get this strategy producing results would require a quick acquisition. This is unlikely given the client’s recent emergence from bankruptcy, poor prospects, and limited financial position. Note: The above-listed question allows the candidate to demonstrate an understanding of strategic alternatives and formulate an approach to identifying the opportunities and risks associated with each. Consulting firms are frequently hired by clients to provide an unbiased assessment of their strategic options – thus the ability to create a logical, clear approach to such issues is a skill we look for in a candidate. 

Question #4: If neither of these alternatives is viable, what strategic options would you recommend to the client? Possible Answer: Although there could be a number of options, a strong answer would include the following: Sell the company. Liquidate plants and other fixed assets – essentially, perform a managed bankruptcy. Reduced debt, improved cost structure and resolve a considerable amount of environmental and other liabilities.  Perform some type of profitability analysis and identify plants / assets that could be shut down so that the anticipated net loss in 2008 does not occur. Note: This question tests a candidate’s ability to develop strategic options with limited information. Consultants are often asked to identify viable options for clients with minimal direction and limited data. A strong candidate must be comfortable offering recommendations under these conditions.   

Romance Publisher Harlequin Rejects Bookstore’s Deal Case Type: operations strategy; new business; math problem. Consulting Firm: IBM Global Business Services 2nd round job interview. Industry Coverage: mass media & communications. Case Interview Question #00233: Your client Harlequin Enterprises Limited is a Toronto, Ontario-based publishing company owned by the Torstar Corporation (TSX: TS.B), the largest newspaper publisher in Canada. Harlequin Enterprises is the world’s leading publisher of series romance and women’s fiction with approximately 120 new titles published each month in 29 different languages in 107 international

markets on six continents. Harlequin Enterprises sell its romance novels directly to bookstores. Typically, the company reimburses its customers at the end of the year for any unsold inventory. Now, one of Harlequin’s customers, a large retail bookstore, has come to it with an offer for a deal. In return for a 10% discount on wholesale prices, the bookstore will no longer send back any books at the end of the year. Should Harlequin do the deal or not? Why? Additional Information: (to be provided to you if asked)  Romance novel sales have been flat for a decade, and sales are expected to remain so in the coming years.  Harlequin’s clients have to sell books at a price they dictate, no cheaper.  In 2008, Harlequin sold 10,000 books to this retail bookstore. At the end of the year, the bookstore sent back 20% of its books to Harlequin.  It costs Harlequin $5 on average to make a single book, and Harlequin previously sold books to the bookstore at $10.  Harlequin’s Salvage Value for unsold books = 0.  Note to interviewer: Have the interviewee explain how he/she will estimate/guess at a figure of how many books the bookstore will order this year, then give the projected number of 7500.  MSRP of books is not relevant in this case. Possible Answers: Key elements of analysis to solve the case: 1. Math: Margin calculations. Don’t forget salvage value. Ask the candidate to explore rationale for the projected number (7500) of books that will be ordered this year (directionally up, down or flat?). 2. Opportunities:  

 Reduction in operational complexity.  Good will meeting customer/channel request.  Open up new business model – may lead to additional distribution channels. 3. Threats: Cash Flow (with new model, Harlequin will get less cash flow upfront – a big problem in this kind of economy).  Market Share (assuming fewer units sold).  May affect other clients’ choices as well (i.e. first of many, not isolated case).  Relationship with client (supply them with less books, they will view you as less important). Note to interviewer: When mentioning cash flow, have candidate do the calculations. Previously, get $100,000 upfront, now only get $67,000 (see part 4 Calculations below). 

4. Calculations:  Pre-change: profit = 8,000 * ($10-$5) – 2000 * $5 = $30,000  Post-change: profit = 7,500 * ($9-$5) = $30,000  Pre-change: cash-flow: 10,000 * $10 = $100,000 upfront  Post-change: cash-flow: 7,500 * $9 = $67,500 upfront  Pre-change: market share at 8,000 books  Post-change: market share at 7,500 books Conclusion: The expected profit is the same, but initial cash flow is lower and market share is lower. There does not seem to be a direct financial incentive to take the deal. Final Recommendation: The deal does not seem attractive on a financial or strategic basis. The expected profitability is the same but cash flow and market share will decline. Such problems are complicated by the fact that others in the industry may follow the lead of the first bookstore. Recommendation is not to accept the deal.  Possible Risk: By not accepting the request of your client, you risk alienating the bookstore if other publishers shift business models.  Possible alternative: Consider the differences in “blockbuster” books vs. lower-volume books. Perhaps new business model (at different pricing) may make sense for some titles. Note to Interviewer: This case is primarily a quantitative math problem with margin calculations. Make sure that the candidate explore the calculation part in the case first and foremost; discussion of business and strategic alternatives can come later.

Ryder to Grow Business & Offer Maintenance Only Product Case Type: add capacity, growth; operations strategy; new product. Consulting Firm: A.T. Kearney final round job interview. Industry Coverage: transportation. Case Interview Question #00226: The client Ryder System, Inc. or Ryder (NYSE: R), is an Americanbased Fortune 500 provider of leading edge transportation, logistics and supply chain management solutions worldwide. Headquartered in suburban Miami, Florida, Ryder’s global operation has total revenue of more than $6.5 billion in 2007.

Ryder’s product offerings include: LM, which provides leasing and programmed maintenance of trucks, tractors and trailers to commercial customers; SC, which manages the movement of materials and related information from the acquisition of raw materials to the delivery of finished products to end-users; and DCC, which provides a turn-key transportation service that includes vehicles, drivers, routing and scheduling. The focus of this case is on the Leasing & Maintenance (LM) group. The growth in the overall number of truck registrations has slowed, 2.2% CAGR (Compound Annual Growth Rate). The LM market is declining. However, the client’s revenues within the LM market have been flat. The client has asked your help to put together a growth strategy. More specifically, Ryder is looking at achieving significant growth over the next 2 years and is looking for some major improvements. Note: This case is a classic growth strategy case which involves penetrating a new segment by offering new products. The candidate will be tested on his thoughts for achieving organic growth and on some light quantitative analysis. Question #1: In general, what are the different ways to achieve organic growth? Possible Answer:  Achieving growth through current products offered to current customers (reducing prices).  Achieving growth through new products offered to current customers (cross selling).  Achieving growth through current products offered to new customers (Expansion).  Achieving growth through new products offered to new customers (Penetration). Questions #2: What are your thoughts on the current market dynamics facing the client? Taking this into account, please recommend an organic growth strategy. Additional Information: (to be provided during the course of the interview) 1. Market and growth: 4.7M truck registrations annually (slow growth 2.2% CAGR). 2. Market and customer is segmented as follows: For Hire: 21% (customers who use trucks for hire). Private: 60% (customers who own and maintain their own trucks. Lease and Manufacturing: 11% (customer who buy trucks on lease with packaged maintenance program).  Financing: 8% (customers who finance purchase of their trucks through banks and other groups). 3. Competition: Client and one other competitor U-Haul own 24% each of LM market. The remaining 50% is divided up by smaller players.   

4. Product/Offering: The offerings in the LM market are highly undifferentiated. Each offering has an asset (truck, tractor, and trailer) and a maintenance program for the asset. The asset is owned by the provider and the customer pays a fixed monthly price for leasing the truck (based on the brand, age and financing term) and subscribing to the maintenance program. Subscribing to the maintenance program is no optional as the truck is not owned by the customer. The customers are requited to sign contracts for the specified term with the provider. Possible Answer: The candidate should be able to figure out that the client has grown primarily through acquisitions. Here’s why. The LM market is declining but the client has maintained revenues. Increasing prices is not an option since the product offering is highly undifferentiated. Enough said. The client needs to look elsewhere. This is basically a case on penetrating a new segment. Another key point: If LM market is declining and truck registrations are increasing (albeit slowly), there must be another segment that is increasing share within the overall market. That would be the private segment. What does the private segment need? There are big customers (grocery chains, retail outlets etc.) who are not really interested in leasing trucks and freeing up capital. They may however be interested in maintaining their trucks since that is by no means their core competency. This means that the current LM product (asset + maintenance) no longer appeals to them. Hence we need to offer a new maintenance only product. This product will likely be lower priced since it involves only the maintenance component. Question #3: What are some of the key challenges with introducing an additional new product (especially one that is lower priced)? Possible Answer:  Cannibalization: Existing customers may want this product as well.  Service differentiation: the client may be unable to provide differentiated service based on the type of customer and may end up over serving customers. Question #4 (ask only if time permits, otherwise skip to Question #5): How would you go about formulating what product to offer to the private customers? What process or steps would you follow? Possible Answer: 1. Qualitative Assessment: Interview prospective customers, conduct focus groups and brainstorming sessions with subject matter experts to find out what the private customers needs are. Formulate several hypotheses and list attributes that the new product should have (like different service levels for maintenance program in terms of priority servicing, regional or national coverage of shops that customers can take their trucks to). 2. Quantitative Analysis: Conduct a survey to measure the interest and preference for the attributes outlined in the qualitative assessment. The sample size should be representative of the truck industry

across private customers and other segments and the different types of customer industries (e.g. construction, groceries, utilities etc.) 3. Capability Assessment: Can the client do this? What would it cost them? What upfront investments need to be made? Can the existing infrastructure support the new products and new customers? Can the customer service differentiate? Are there any cultural challenges? 4. Business Case: Use the findings to build a business case quantifying the revenue opportunity using estimated pricing and estimated customer volumes from the survey. Question #5: If the client Ryder captures 1% of the private market by introducing this new product, at what rate do his overall revenues increase? Possible Answer: Client has 24% of 11% of 4.7M trucks. = ~2.6% of 4.7M trucks. If Ryder captures 1% of the private segment, its market share will be 3.6%. This represents an increase of 1/2.6 = 38%.

Microsoft Pursues Strategy for Penetrating Large Organizations Case Type: math problem; organizational behavior; operations strategy. Consulting Firm: PricewaterhouseCoopers (PwC) final round job interview. Industry Coverage: software, information technology (IT). Case Interview Question #00200: The client Microsoft Corporation (NASDAQ: MSFT) is a multinational corporation headquartered in Redmond, Washington, USA. Microsoft develops, manufactures, licenses, and supports a wide range of software products and IT services predominantly related to computing through its various product divisions.

One particular division of the client makes a software product called Microsoft System Manager (MSM) that manages desktop applications across different machines. There are 2 broad categories of customers – large organizations (with 1000 + computers) and small organizations (with 100 – 1000 computers) that use the MSM software. Microsoft has a strong presence in the small customer segment but has not been able to penetrate the large segment. Question: 1. Why is Microsoft not able to have a footprint in the large segment? 2. What recommendations would you have – stay or exit the large segment? 3. What tactics would you use if the client decides to stay/exit? Additional Information: Note to case giver/interviewer: This is a quantitative numbers case – so guide the candidate appropriately.

The Microsoft System Manager (MSM) desktop application software reduces system crashes and calls to system administrator. It is a background application so the end-user does not even know that it exists on his / her computer.  Microsoft sells the software through computer retail stores that sell software products. There are over 2,000 such retail stores across the country.  Large segments are more profitable than small segments, and currently the small segment market is almost saturated. So from the client’s perspective, they clearly need to sell more to large segment. The question is: How do we sell more?  No other player operates in this niche market – however an open source system application manager is in the cards.  Microsoft’s MSM software is extremely stable and has clear benefits because of reduced system outage. These benefits are more for large companies than for small companies.  Client sells the software, retail stores sell support services at time of sale. Retail stores sell using a direct sales force that visits the large / small companies.  The average license price for small firm is $2,000. Stores sell 100 licenses a year and get a 10% margin. In addition, retail stores sell support services 100% of the time for each license, each of which produces another $2,000 in revenue with a total margin of 10%.  The average license price for large firm is $20,000. Stores sell 12 licenses to get a 25% margin. In addition, retail stores sell support services 20% of the time for each license, each of which produces another $20,000 in revenue with a total margin of 25%.  The actual sale is made by sales representatives who get 1% of every $1 in revenue they bring in to the store. A sales representative can either make 8 small company sales or 2 large company sales in a given month. Possible Answer: 

While a 3-C approach works, a faster approach is to contrast the large versus small company and look at the client’s product, competition and features in these 2 segments. The candidate should quickly determine that the problem is in the channel, the open source application is just a distraction. Candidate should realize that large companies will not go in for open source product until the product matures. To compare small company v.s. large company: 1. Small company: Revenue = 2000 * 100 = 200,000 Profit to store = 200,000 * 0.1 = 20,000 Profit from support services = 200,000 * 0.1 * 100% = 20,000 Net profit to store / company = 40,000 2. Large company: Revenue = 20000 * 12 = 240,000 Profit to store = 240,000 * 0.25 = 60,000 Profit from support services = 240,000 * 0.25 * 20% = 12,000 Net profit to store / company = 72,000 Therefore, retail stores get more profits if they are able to sell to large companies.

3. Sales representative incentive: Small firm = 8 * 40,000 * 0.01 = $3,200 per month Large firm = 2 * 72,000 * 0.01 = $1,440 per month Sales reps therefore tend to favor searching for small firms – which is okay if they can continue to make 8 sales a month, but this market is maturing, so they are potentially going on fruitless searches. The sales rep incentives should reflect the margin per sale and not just the net margin. Outstanding Answer After the candidate gets the numbers, an outstanding answer will just have to be creative:   

Examine merits / demerits of moving away from channel partner towards having a dedicated direct sales force. Piggyback on another software vendor’s direct sales force. Partner with computer vendors (Dell, HP, etc) to have software installed in OEM (Original Equipment Manufacturer) fashion, etc.

US Airways to Re-paint Its Fleet of 350 Aircrafts Case Type: math problem; operations strategy. Consulting Firm: Simon-Kucher & Partners first round job interview. Industry Coverage: Airlines. Case Interview Question #00189: Your client US Airways, Inc. is a major airline based in Tempe, Arizona, United States. The airline is an operating unit of US Airways Group (NYSE: LCC) and is the 6th largest airline by traffic and 8th by market value in the country. US Airways operates major hubs in Charlotte, Phoenix and Philadelphia and maintains focus city operations at Ronald Reagan Washington National Airport.

The U.S. domestic airline industry is extremely competitive and airline companies frequently sustain large losses. Consequently, decisions that involve even small outlays of cash are carefully considered. Question #1: The client US Airways has ugly planes. Should it re-paint them? Additional Information:  Most airlines paint their planes to match their corporate logo or as part of a brand image campaign.  US Airways has updated its brand image/marketing materials, but has not yet painted its planes.  The client US Airways’ planes are currently painted with a logo/image dating from the mid-1980’s in colors currently considered out-of-date.

The airline industry is a fixed-cost business. Adding passengers does not increase costs very much. Thus, the client should do whatever is required to fill the planes.  If customers think the planes are ugly, they will fly an airline with better-looking planes.  The customer segment that cares about plane appearance is the vacation traveler who travels out of non-hub cities.  Re-painting planes costs about $500,000 per plane, the expected revenue increase due to painting planes is 1%.  US Airways’ fleet currently has 350 mainline jet airplanes and total Revenues of $10 billion. Possible Answer: 

The first part of the case requires a cost-benefit analysis. Essentially it is simple math problem. The candidate should attempt to discover the costs and benefits of the action before recommending a solution. Increase revenue by 1% will lead to a ($10 billion x 1% = $100 million) increase in Revenue, compared to the ($500,000 x 350 = $175 million) cost, the painting expenditure will be recovered in less then two years. Therefore, paint the planes! Question #2: Since the client US Airways is going to paint its planes, how can the job be accomplished without idling aircraft? Additional Information:  Grounding an airplane reduces its utilization, thereby reducing its potential revenue.  Airlines don’t like grounding planes even if few flights are scheduled, since they are more likely to incur uncompensated costs if problems occur. Possible Answer: The candidate ought to point out that planes routinely are grounded for two weeks per year for D-check major maintenance. It so happens that it would take 12-14 days to paint one plane: 2 days to paint and 10-12 days for the drying/curing cycle. Therefore, the planes ought to be painted during the regular maintenance cycle.

What Would Southwest Do If Oil Price Dropped to Zero? Case Type: industry analysis; operations strategy. Consulting Firm: Gallup Consulting final round job interview. Industry Coverage: Airlines; Oil, Gas & Petroleum Industry. Case Interview Question #00185: Imagine you are the CEO of Southwest Airlines (NYSE: LUV), the world’s largest low-cost carrier. Headquartered in Dallas, Texas, Southwest is the largest airline in the world by number of passengers carried per year as of 2009. It maintains the fourth-largest passenger fleet of aircraft among all of the world’s commercial airlines. As of December 31, 2009, Southwest operates more than 3,200 flights daily.

As the CEO, you have just learned that starting from tomorrow the price of oil will drop to zero. Obviously, this has significant ramifications for your business. Now the question to you is: what three people would you want to talk to regarding this development and why? Additional Information:  Price of oil, including transportation and refining costs, is zero.  Cost of jet fuel is zero.  Assume that oil companies make oil at zero profits.  Ignore geo-political issues.  No other specific information – Candidate must decide on the critical issues. Possible Answer: The interviewer should seek to conduct a conversation, where the interviewee creates a structure and works through questions such as the following:  How does this change impact the business – revenue drivers?  What is the cost structure of the airline? Fuels costs are a significant portion of the operating costs of an airplane. Thus, it is likely that prices could be reduced. This will result in an increase of primary demand. People who might have taken the bus, train, or car might now be willing to fly at the lower cost.  What effects will increased demand have on utilization rates?  Do you need more planes? Is there going to be a first mover advantage?  Are Boeing and Airbus going to be able to make enough planes to meet the new demand? Will the prices of planes go up? Can airports handle more planes?  Will planes travel farther now since they have no fuel costs?  How will competitors react to this news? So, who might you talk to (not an exhaustive list by any means; there are many, many others):      

CFO or Accounting Officer of Southwest who knows about the internal cost structure will be able to tell you what new items will be driving cost and constraining operations. Market Researcher – how does one sell the new airline to the public? Labor Unions – labor will be a bigger factor in the cost structure Airplane Manufacturer Airbus/Boeing – can you get more planes if demand increases? Can you get planes that fly longer (more fuel capacity) Airport Authority – can you reserve more landing slots before others? Competitors like United, American, Delta, etc – without discussing prices and avoiding collusion, are there other things you can learn from your competitors?

Once again, there are no right answers here – the key is did you discuss the strange issue intelligently? Did you pick three reasonable people and does your logic support the three you picked?

Bank of East Asia Improves Cost-to-Revenue Ratio Case Type: reduce costs; operations strategy. Consulting Firm: Roland Berger Strategy Consultants final round job interview. Industry Coverage: banking. Case Interview Question #00179: Your client is the processing division of the Bank of East Asia (BEA), a large regional Asian bank based in Hong Kong. You have been hired because the processing division wants to improve its cost-to-revenue ratio (CRR). Present calculations reveal that the current CRR is 0.60. The client wants to improve this to the industry standard of 0.40. What strategies can the client pursue to lower its CRR to 0.40?

Additional Information: (to be given to you if asked) The processing division of BEA provides back-office services for the bank’s three other divisions: Retail Banking, Corporate Banking, and Regional Banking. These services include: credit processing, payments processing, customer service processing, call center services, and credit card processing. The client receives $100M in annual revenues. In the past, revenues have always grown or diminished in accordance with industry trends; costs increase and decrease in proportion to revenue. This trend is expected to continue in the future. An analysis of the revenue/cost distribution will show that revenue and costs are distributed evenly among the three customers. [The candidate can calculate that costs are approximately $60M.] The client is organized into three highly independent groups, with each group servicing one customer. Analyzing the individual groups will reveal that none of them stand out as being particularly inefficient, and they all have CRR’s of about 0.60. However, there are many similarities and overlaps in the processes they perform (all processes are handled internally). [There is no need for the candidate to explore the specific processes or overlaps.] A cost analysis will reveal two types of costs: labor and systems. Labor costs stem from the organization’s workforce of 1,000 employees, with an average annual salary of $40,000 to $45,000—salaries are in accordance with industry standards. [Salary cuts are not advisable.] Systems costs are primarily IT related, and comprise the remainder of costs. A study of the client’s IT systems will reveal that they are below industry level in sophistication.

A competitive analysis will reveal that the average industry cost distribution is 60% labor, 40% systems. No other information is available about competition. The client has no control over the volume of revenue it receives from its three internal customers. Furthermore, the client believes that strategies to boost revenue by fine-tuning the operations of its customers are beyond the scope of the engagement. The client’s pricing for its internal customers is consistent with the industry standard. The client is aware that it can increase revenues by increasing internal pricing, but would like to hear other recommendations. Possible Answer: This case can be approached by looking at ways to reduce costs and increase revenues in order to hit the target.

The client’s labor/systems cost mix is 70/30%, compared with an industry standard of 60/40%. Upgrading IT systems may allow it to decrease its workforce, and reduce overall costs. The client can integrate its three groups (servicing Retail Banking, Corporate Banking and Regional Banking), and look for synergies. Possible synergies include integrating common processes, and transferring best practices. This can result in reduced manpower requirements, and further cost savings. (The candidate should list possible synergies that can occur from integration.)

Reducing costs may not be enough to achieve the target CRR. There are no options to increase revenues from the internal customers; however, the client can explore insourcing, where they can offer processing services to external banks for services that they have high competencies in. This can increase revenues substantially, while bringing economies of scale. The client can combine the insourcing strategy with outsourcing the processes that it does not have competencies in, and are not highly sensitive. The client can determine its strategy to maintain, develop, insource, or outsource the different processes as the above matrix.

DeVere Group Assesses Health Club Industry Trend Case Type: industry analysis; operations strategy. Consulting Firm: OC&C Strategy Consultants final round job interview. Industry Coverage: sports, leisure & recreation; tourism, hospitality & lodging. Case Interview Question #00177: Our client DeVere Group runs a major chain of health clubs in the UK (Greens Health & Fitness), alongside hotels, resorts, venues, and casinos (which are not the focus of this case). The Greens Health & Fitness clubs are large out-of-town sports centers offering state-of-the-art gym, fitness classes, Jacuzzi, swimming pool, etc. They are relatively expensive, about £30-40 per month for an individual membership.

As part of a broader strategy review, the client DeVere Group wants to know what they should do with their leisure clubs division – should they sell it, rapidly build more clubs (if so, what sort), or maybe acquire another player? Your specific task on this case is to look at the market trends and assess competition in the leisure clubs industry. Demand for leisure clubs Question #1: What factors might you analyse to determine what is going to happen to demand for leisure clubs? (In consulting terminology, the job candidate should look at the ‘drivers’ of demand in the industry) Possible Answers: The Good Answer will name some of the following factors, with some (prompted) discussion of the associated issues:  Trends in society towards more or less participation in sport – Whilst more participation in sport generally may be positive for demand, increases in popularity of sports not offered at health clubs may have a negative effect (e.g. people may play football instead of going to the gym).  Trends in obesity – if the population is getting more obese, there are two possible implications of this. One is that people are getting more obese because they are not exercising (i.e. declining demand for leisure clubs). The alternative interpretation is that an increasingly obese population will create demand for facilities to exercise more.  Trends in available leisure time and money. If people have more spare time, they are likely to use health clubs more. National income / state of the economy – leisure clubs are likely to be a luxury good, for which demand will decline if there is a recession.  Demographics – Younger people are more likely to be members of gyms. Therefore, if the population as a whole is getting older, demand for leisure clubs is likely to decline. The Excellent Answer will name most of the above factors, with more explanation of why they are important, and may include other sensible suggestions e.g. sales of Slim Fast (a brand of dietary supplement food products promoting diets and weight loss plans) would be a good indicator of what has

been happening to demand for leisure clubs, because people who buy “Slim Fast” are the type of people who would use leisure clubs. An excellent candidate will also be able to defend sensible answers when questioned or pushed on why a particular factor is important; often it is at this point in the case study where excellent candidates differentiate themselves.

Interpretation of data – demand factors The interviewer would then present five charts (including the two charts below; the other three charts would not provide useful information but serve as ‘distraction’) for interpretation by the interviewee, who would be asked to explain what the data showed, and what that meant for the demand for leisure clubs. Question #2: What is likely to happen to demand for leisure clubs? Possible Answers: The good answer–the charts clearly show that obesity has increased rapidly over the past two decades, and this trend is forecast to continue into the future. This means that demand for leisure clubs will increase, as people will have an increasing need to exercise. The excellent answer–the first chart shows that obesity has increased historically, and this trend is forecast to continue. However, from the first chart it is not clear whether people are becoming more obese because they are not exercising enough (which would indicate declining demand for leisure clubs), or whether an increasingly obese population will exercise more and hence create increased demand. If I combine the two charts, however, you can see that most people are likely to exercise more if they wished to lose weight, and this is particularly true of wealthy people (who are likely to be our target customer anyway). On the assumption that most obese people want to lose weight, we can say that an increasingly obese population will create increased demand for leisure clubs.

Interpretation of data – market outlook Having seen the data, the candidate should come to the conclusion that demand for leisure clubs is likely to be strong for the next few years. He/she will then be shown a chart showing the competitors’ plans for building clubs, demonstrating that the number of leisure clubs is forecasted to rise from 150 in 2007 to 500 in 2010. Question #3: What is the critical issue for our client DeVere Group? Possible Answers: The good candidate would indicate that their market may become saturated, as supply grows faster than demand. The excellent candidate would say: “The important question is whether supply in 2010 is greater than demand (i.e. there are more spaces available in leisure clubs than can be filled by people wanting to visit clubs). In this situation, there is likely to be price competition, which may make the business unprofitable. However, not all of the 500 clubs will compete with our client – we need to differentiate between different club types.” Market Segmentation Question #4: In order to better understand the competition, we need to understand what differentiates leisure clubs? (This may be phrased in consulting jargon as “How might we think about segmenting the leisure club market?”) Possible Answers: The good candidate will point to some of the following: Price – the type of customer who uses a premium club (£50 per month) is unlikely to switch to a budget club (e.g. Council-run gyms costing £2 per visit).  Location  Geographic – customers will only travel a limited distance to go to a club, so clubs in London do not compete with clubs in Sheffield, for example  In town / out of town – out of town clubs may be ‘destinations’ (primarily aimed at families, for example), whereas in town clubs are likely to appeal because they are ‘convenient’ (e.g. to office workers).  Customer Group – different clubs are designed to attract different types of customer e.g. prefamily (~20-30 year olds), fitness fanatics, female-only, family-oriented, etc. The excellent candidate will name all the above factors, with maybe some others (e.g. sports offered), and will elaborate more on each. 

Conclusion: This phase will probably only be reached by a few excellent candidates. So we know that demand for leisure clubs is growing, but so is supply. We know that not all of the 350 new clubs will compete with our clients’ business, since clubs differ based upon price, target customer and location. Applying our segmentation to the 350 clubs, we found that most of them do fall broadly into our segment (i.e. premium price, similar target customers). So we started to think about the geographical dimension, and considered where we should build our new clubs. The competition was planning to build in the large cities; however, we recommended that our client build its clubs in smaller towns, which only had populations large enough to sustain one leisure club. Why do you think this was? The excellent candidate: “By placing your club in the smaller towns, you will insulate yourself from competition. Because you have built your club first, you will attract members who will then be loyal to your club. When the competition considers where to build their next club, they will not build in that town, because they will see that there are not enough ‘available’ customers for their club to be profitable, and they know it would be difficult to attract customers away from your club. Hence, even though in later years there may be excess supply at a national level, you will have local monopolies, and thus be protected from potential pressure to lower prices.” Note: This case interview question is based on a real case study that OC&C Strategy Consultants worked on in 2007, assessing the market for Leisure Clubs as part of a strategy review for a major UK hotel chain. This is typical of a ‘data interpretation’ case study that you may be asked to attempt – requiring analysis of data presented to you, alongside some creative and logical thinking.

Should Cab Driver Stay in O’Hare or Go Back to City? Case Type: operations strategy; math problem. Consulting Firm: IBM Global Business Services final round job interview. Industry Coverage: transportation. Case Interview Question #00174: You have a friend who is a taxi cab operator (not the cab owner) in Chicago and he is asking for your help in assessing a situation: After dropping off customers at Chicago O’Hare International Airport (ORD), the taxi driver has 2 choices: 1. Stay at O’Hare and wait in the queue for 2 hours to take a customer back into the city. 2. Go directly back to the city and conduct business.

Your taxi driver friend wants to determine which one is the better option. What would you recommend and why? Note to Interviewer: At this stage, ask the interviewee to brain storm how he/she would approach the problem. Seek a broad level understanding of the different issues that should be addressed. This case is meant to be very interactive and you need to ask the candidate very often for his/her thoughts. After a short brainstorming session and an introduction of the framework, the candidate should ask for the

following facts specifically. If he/she does not think of one, help them reach it by asking questions that will lead to it. Additional Information: (to be given to you if asked)  Distance: The distance of the trip between O’Hare International Airport and Downtown Chicago is approximately 20 miles, and the trip is all highway  Operation Revenues: $4 for the first mile, and $2 for every subsequent mile, tips = 15% of total meter.  Operation Costs:  Current gas price is $3/gallon, the cab’s gasoline consumption is 25 miles/gallon when driving on the highway and 20 miles/gallon in the city.  50% meter revenue goes to the cab owner.  When waiting in the queue, assume no gas is consumed.  The toll for a one-way trip is $3. Possible Answer: The variable cost for a one way trip (Gas) is: ($3/gallon) x (20 miles) / (25 miles/gallon) = $2.4. Assume all other costs i.e. maintenance etc., are negligible. Option #1 Cost/Benefit Analysis: Waiting in O’Hare Airport  Revenues: meter $4 + $2×19 = $42, tips $42 x 15% = $6.3, $48.3 in total  Costs: – $21 (half the meter to cab owner) – $2.4 (Gas for trip from O’Hare to city) = -$23.6  Profit: $48.3 – $23.6 = $24.9 Option #2 Cost/Benefit Analysis: Going back to the City There is no right answer here – any reasonable assumption would do. Try to gauge the interviewee on what he/she thinks is appropriate. Assume 40 minutes ride from O’Hare to Downtown Chicago (lost). In the remaining 80 minutes we assume the cab driver to have 6 rides. The candidate can set up a table like this to structure the calculations:

Number of rides

Number of miles driven Value per ride Total

2

6 miles

4+10=14

28

2

3 miles

4+4=8

16

2

1 mile

4

8

Total

8+4+2=14 miles

52

We also assume the cab will be driving a total of 10 miles around Chicago downtwon looking for customers. 

Revenues: meter $52 + tips $7.8 = $59.8

Costs: – $2.4 (Gas for trip from O’Hare to city) – $3 (Tollway) – $26 (half the meter) – $3.6 ($1.5 gas for driving around for 10 miles looking for customers + $2.1 driving customers for 14 miles) = $25  Profit: $59.8 – $25 = $24.8 Conclusion: In both scenarios, the taxi driver will earn about $25. In recommending one approach, we believe it is better to wait in the queue so that the taxi driver can use the 2 hours to do other things (i.e. study, relax). Also, there is less risk involved with less miles driven, and extended life of cab, reduced maintenance costs etc. Note: An especially excellent answer would also address the a microeconomic stand point of this equilibrium, especially if the numbers of the two options don’t match in the interviewee’s calculation. 

Burton Snowboards Develops Profit Growth Strategy Case Type: improve profits/bottom line; operations strategy. Consulting Firm: OC&C Strategy Consultants second round job interview. Industry Coverage: sports, leisure & recreation. Case Interview Question #00170 Our client Burton Snowboards is a manufacturer of snowboarding equipment and accessories. The company specializes in a product line aimed at snowboarders: snowboards, bindings, boots, outerwear, and accessories, with two distinct major product divisions: boards and clothes. Burton Snowboards has experienced a fall in profitability in recent years. You have been hired to identify the causes for falling profitability. Once that has been completed, the objective is to develop a range of strategic options for the company, which will lead to a return to profit growth.

Additional Information: At the top level, financial performance has changed as follows:

Total business in year, £ million 2005

2006

2007

Revenue

80

90

120

Profit

10

10

5

Profit margin (%)

12.5 %

11.1% 4.2%

Question #1: What information would you require to help explain the change in profitability?

Possible Answer: The Good Answer: Revenue has increased, but profit is going down, clearly increased costs are causing the problem. You need to understand what has been driving the cost increases in the business, perhaps by looking at volumes vs. price levels, or by splitting fixed and variable costs. Also, you would need to consider the market and competitive environment for snowboarding equipment – in particular whether competitors were also suffering price pressure. The Excellent Answer: all of the above, but considering the performance of the board division and the clothing division separately. Solution: the key to a good understanding in this situation is to break down the problem into manageable chunks. If we consider the two divisions separately a much clearer picture emerges:

Total business in year, £ million 2005 2006 2007 Boards Revenue

60

60

60

Profit

15

15

15

Profit margin (%)

25% 25% 25%

Clothing Revenue

20

30

60

Profit

-5

-5

-10

Profit margin (%)

-25% -17% -17%

We can now see that the board division is relatively flat in revenue terms, but brings in constant profit. However, the clothes division, despite growing rapidly, is losing money. We will now turn our attentions to the boards division. Question #2: What are the strategic options available for the boards division? The context given for this part of case is as follows: The market for snowboards is growing slightly, at around 5% per annum, in volume terms, but prices are rising much more quickly. Our client Burton Snowboards is clearly not sharing in this growth. Their volumes and price are static. What additional information do you need and what sort of things could Burton Snowboards do to improve their performance? Possible Answer:

The good answer will consider some or all of the following points: We need to know who the competitors are, and how their price levels and quality compare to our client’s. We also need to understand the level of marketing activity in the industry, and how much marketing the client does compared to the competition. Then we can investigate if our client could reduce costs and prices to increase their sales. The client would also consider doing more marketing, and extending their distribution to more shops. The Excellent answer would also consider the need to understand how the market works in practice. What are the different types of board available – e.g. top of the range versus standard? What makes customers choose a particular make of snowboard? Once we understand these issues, we can devise more appropriate strategies for our client, such as sponsoring professionals riders and tour events such as National Snowboarding Championships, creating beginner learning to ride program. Question #3: What can the client do with the clothes division? Possible Answer: The good answer will consider some or all of the following points – Why is the clothes division not making money currently? What options does the client have to manufacture more cheaply? Should they pull out of clothing manufacture? The excellent answer would also include that the business is growing quickly and that some of the costs may be temporary. Also, that the two businesses, although separate, are intertwined, and that the clothing business will benefit if the profile of the boards business is increased through funding of tour events etc. Question #4: what else could our client consider doing with the business? Possible Answer: It is unlikely that there is time to consider many of the following, but excellent candidates often discuss one or two:     

Potential for opening some owned shops, dedicated to retailing our client’s own products. Online E-business opportunities around snowboarding destination site – holidays, merchandise, reviews, music etc. Investigation of potential mergers and take-overs to improve competitive position. Expansion into other product areas – e.g. surfing merchandise. Outsource clothing production to another company and licence use of brand name.

Lockheed Martin Reduces Costs of Cyclical Swings Case Type: reduce cost; operations strategy. Consulting Firm: A.T. Kearney second round job interview. Industry Coverage: Aerospace & Defense; Manufacturing. Case Interview Question #00169: Our client Lockheed Martin (NYSE: LMT) is a United States aerospace, defense, security, and advanced technology company formed by the merger of Lockheed Corporation with Martin Marietta Corporation in 1995. In the early 1970’s, Lockheed Corporation

manufactured L-1011 TriStar, a three-engined wide-body aircraft seating up to 400 passengers for

commercial airlines. The industry was very cyclical with swings in demand occurring as frequently as every 6 months (see attached figure below). During the down months, Lockheed would have to layoff employees and shutter the plants, which created turmoil for the company and the local community. Jet aircraft were normally built to the order specification of the purchasing airline. To alleviate the costs of cyclical swings, Lockheed considered building aircraft to a predetermined schedule based on average expected aircraft sales over the next five years (see attached figure). Do you think this is a good idea and why? What are the pros and cons of pursuing such a plan? Possible Answer: Unlike some other costs cutting cases, this case doesn’t really have a yes/no solution. The important thing in a case like this is to identify the major issues and state your approach for arriving at a solution. The interviewer wants to see if you have a basic understanding of manufacturing business and the costs inherent in running such an operation. On a real problem like this, you would need to model the costs (with a spreadsheet) of the current (build to order) approach and the new approach (build to schedule) and then test the new approach under a range of sensitivities, both positive and negative. Revenues are unlikely to be impacted by this decision, unless having aircraft in inventory would facilitate greater sales. A good case solution would identify the cost drivers and risks and arrives at an educated guess of the right answer. Let’s look at some of the costs and how they would be impacted under a new “build to schedule” plan: 1. Inventory or Working Capital Holding Costs – This could be huge under a “build to schedule” production plan. If demand is not as predicted or the market heads into a cyclical dip, Lockheed could end up holding a lot of very expensive (tens of millions of dollars) inventory that just sits on the books. At a 68% risk free rate of return, the inventory holding costs of such expensive assets would add up rapidly. 2. Labor Costs – Labor would probably be cheaper under a build to schedule. The company could avoid costly retraining and rehiring of its workforce after layoff. Additionally, the company would have to pay less severance costs due to fewer layoffs. With a more guaranteed production schedule, the company may be able to extract wage concessions from its unions. But, such savings would evaporate if the company were to busily build new aircraft that the market doesn’t want.

3.Technological Obsolescence – With build to schedule, you run the risk of building aircraft that aren’t demanded in the marketplace because they are obsolete. Thus, if a competitor introduces a much better model at the existing price points or a technology change renders current models as inefficient, Lockheed would have to liquidate any existing inventory at fire sale prices. 4. Forecasting – A build to schedule plan for such costly goods requires a very accurate forecast of future demand. Can demand really be forecasted with sufficient accuracy? Average expected sales based on linear regression of past sales, like the one shown in the attached figure, is clearly not good enough. 5. Rework Costs – Airlines request specialized configurations of the aircraft to meet their particular needs. If the company pursues build to schedule, they would need to budget rework costs to change pre-built models to the specifications of the purchasing airline. Or, if they only build aircraft partially, how will they handle the production backlog of moving these partially completed aircraft through the remaining production steps? 6. Fixed Production Equipment and Facilities – In general, the fixed costs of production equipment and facilities should not change with a change in production approach. Now, if the company amortizes equipment on a per unit basis, then net income could be affected under the new plan. But, cash flows, the important thing to look at when making business decisions, should not be affected. 7. Variable Materials Costs – These include materials and components required for building the aircraft. Under a build to schedule plan, Lockheed could probably negotiate lower costs from their suppliers since they would be able to guarantee a steady stream of purchases. 8. Unused capacity – Under “build to schedule” plan, Lockheed are likely to have unused production capacity since they currently carry sufficient capacity to meet cyclical demand surges. The analysis should carefully examine the existing production assets to determine if savings could be realized through capacity reduction. Conclusion: After identifying the variables, the interviewer would expect you to take a guess on the right answer based on your assumptions. It turns out that “build to schedule” approach is not viable for this company because they cannot predict demand with sufficient accuracy and the capital holding costs are simply too expensive.

How to Analyze Spanish Conquest of Inca Empire? Case Type: HR/organizational behavior; operations strategy. Consulting Firm: PricewaterhouseCoopers (PwC) final round job interview. Industry Coverage: Consulting, HR, Business Services. Case Interview Question #00162: In the 16th century, Spanish explorer and conqueror Francisco Pizarro, together with an army of 106 foot-soldiers and 62 horsemen, defeated the Inca Empire of 80,000 soldiers South America. Characteristics of the Inca people are:  they were rich, and they had mastered the art of making gold and handicrafts.

they had agriculture, but no way of storing food for long. they regarded their king as a god, the child of the Sun god. there had recently been civil war among the Incas and the brother of the king had led an uprising against the king. How would you draw parallels between this historical incident and modern corporate operations in terms of strategy, IT, HR, marketing and finance? Possible Answers:   

Financing: Getting cash from the King and Queen of Spain. Queen Isabel, in the absence of her husband King Charles I, signed a grant of license which authorized Francisco Pizarro to proceed with the conquest of Peru. Human Resources: Recruiting and motivating the soldiers and sailors. One of the conditions of the grant was that within six months Pizarro should raise a sufficiently equipped force of 250 men. When Pizarro’s expedition was ready, it numbered three ships, 180 men (including several of his relatives), and 27 horses. Technology Advantage: Leveraging diverse technologies from gun-power to navigation. Contemporary accounts by members of Pizarro’s force explain how the Spanish forces used a cavalry charge against the Inca forces, who had never seen horses, in combination with gunfire from cover (the Inca forces had never encountered guns before). Other factors in the Spaniard’s favor were their steel swords, helmets and armor, against the Inca forces which only had leather armor and were unarmed. The Spanish also had 4 small cannons commanded by a Greek artillery captain which were used to great effect in the crowded Inca town square. IT/Knowledge Transfer: Pizarro learned from Hernan Cortes and other explorers. The Incas did not have a written language, so knowledge transfer was difficult. Marketing: Playing one Incan faction off against the other faction. Pizarro and his men were greatly aided in their enterprise by the fact that they arrived when the Inca Empire was in the midst of a war of succession between two princes.

Operations Strategy: The first target of the Spanish attack was the Inca Emperor and his top commanders. Once these had been killed or captured the Inca forces were disorganized as the commanding structure of the army had been effectively decapitated.

Marriott Hotels to Develop E-commerce Strategy Case Type: operations strategy. Consulting Firm: Diamond Management & Technology Consultants (now PwC Advisory) 2nd round summer associate interview. Industry Coverage: Tourism, Hospitality & Lodging; E-commerce, Online Business. Case Interview Question #00130: You client Marriott Hotels (NYSE: MAR) is a large lodging company and franchisor of a broad portfolio of hotels and related lodging facilities. Their strategy has been to maintain and develop their brands. They have about 10% market share and this makes them one of the

largest players  in the hotel and lodging industry. The year is 1999. Your client is concerned with the impact of the Internet. They currently have a web site that allows customers to make reservations at their hotels. They expect to do somewhere between $50 and $90 million worth of business through their website next year. They want you to develop an ecommerce strategy that will prevent them from losing business to their competition and help them maintain and develop their brands. How would you go about it? Possible Answers: Interviewee: The case has at least three levels. The first level can be attacked using the basic three C’s framework. Competition: Think about who the competitors are in this situation. There are two groups, the traditional competitors – other large lodging companies (Hilton Hotels, Hyatt Hotels) and new competitors providing travel services on the web. (By identifying these two groups, you can ask some pointed questions: What are the traditional competitors doing on the web?) Interviewer: Most of them are in wait and see mode. Everyone has a web page. Some can make reservations on-line , some cannot. Interviewee: Are there new competitors such as startups that are providing travel-related services? Interviewer: Yes, they are still young, but growing fast. They provide many services such as rate comparisons and availability searches across competing providers of travel services. Interviewee: (This answer should prompt deeper investigation) What travel offerings are start-ups selling?

Interviewer: Some are doing hotels, some transportation, some are attempting to combine many services. It is not clear what will happen. These new companies are aggressive and are trying many innovative things that may or may not work. Interviewee: (At this point you have some good competitor information that will be useful in levels 2 and 3 of the case, but time is short so it is probably a good idea to move on.) Customers: It is always good to find out how the clients customers are segmented. This is a basic and very fair 3Cs question to ask. In this case the client is large and has customers across the spectrum. The high level segments are business travelers and vacationers. Business travelers are generally less price sensitive than vacationers. In this case it is a good idea to dive a little deeper by asking more questions based on the information you uncovered so far: Which customer segments are currently using the clients web site? Interviewer: The web site is currently used primarily by business travelers. Interviewee: What types of customers are using the competitor’s web sites? Interviewer: For traditional competitors the client thinks that they are similar to their own. For startups, they don’t know, they are looking to you for these answers. We do know that some new sites are targeted to vacationers, others to business travelers, some to both. Interviewee: Company/Capabilities: You already have much of the basic information about the company so you probably don’t need to spend much time here gathering more basic information. A good question here would be to better understand the project: What are the clients objectives for their e-commerce strategy? Interviewer: They aren’t sure, they are looking to you to help understand what is possible. They are worried about competition and want to build their brands. Interviewee: It may also be good to understand the financial situation of the company: How does the balance sheet look, do they have the money to do anything? Interviewer: The finances are sound. The client has the capability to invest but the investment must be justified. Interviewee: At this point you have done a good 3C’s analysis. Now comes the hard part, what next? This is level 2 of this case. It is not going to be possible to develop a brilliant and comprehensive e-commerce strategy in 20 minutes so a good tactic for attacking a broad strategy question is to develop a short list of options: The Client has at least 3 options:

Option 1. Stay the course. Continue to enhance the current web site to make it user friendly, provide some incentives for reserving on line, track customer information to learn more about customers. Hope that strength of brands and customer loyalty will maintain market share and revenues. This option will have the lowest impact but makes our client vulnerable to losing business to innovative startups. Option 2. Focus on cooperating with winning startups and make sure that their brands get good positioning. This would work best if there are clear winners in web travel services. However, if the client is concerned about brand building, this option may not provide the control they would want. They risk being “commoditized” by sites that focus only on price. Option 3. Develop a comprehensive travel services website that competes with the startups and meets all of the travel the needs of our client’s customers. The client is a leader in the lodging and can leverage experience and brand equity to build a successful Internet business. This strategy is customer focussed and provides the control that the client can use for brand building. At this point you can further evaluate each option using a cost/benefit or risk/reward framework and follow up with a plan to do additional research and analysis that would allow you to select the right choice for the client. However, it is likely that the interviewer will interrupt and challenge you with questions about an area that they want to examine further. In this case, the interviewer might say: Interviewer: OK, those are good options, the client is very excited about option 3, they want to be aggressive and build their own Internet business. However, senior management is concerned about the impact that starting a new business will have on their EPS (earnings per share). What should they do to deal with this problem? Interviewee: Again, listing some options might be a good approach. They could mitigate the risk by partnering. Potential partners could be airlines or existing startup travel services websites with weak lodging offerings. They might also look to obtain venture financing to fund part of the business. If they are not concerned with losing control of the new business they could spin it off completely and make a small investment that would not require them to consolidate the financials of the new companies. Depending on how much time you have, you could continue this line of discussion by asking what options the client might prefer and then further developing those options. However, if time is short it is always a good idea to summarize.

US Steel Concerned about Vulnerability to Market Cycles Case Type: operations strategy. Consulting Firm: Towers Watson 2nd round job interview. Industry Coverage: Mining & Metals Production.

Case Interview Questions #00104: Your client United States Steel Corporation (NYSE: X, more commonly known as U.S. Steel), is an integrated steel producer with major production operations in the United States, Canada, and Central Europe. US Steel operates a steel mill in Chicago, Illinois and is

concerned about vulnerability to  market cycles. What should it do? Additional Information: (to be given to you if asked for) 1. Fixed costs are 50% of total costs. 2. Demand for steel is highly cyclical. Demand in the trough of a recession can be as low as 70% of the demand at the peak of the business cycle. 3. The market pressures the company to pay out excess cash in the form of dividends during upturns in the economy. The dividend as of 2008 was $0.30 per share. As of April, 2009, it was reduced to $0.05 per share. 4. Labor unions are inflexible with regard to work rule changes. 5. There is increased competition from mini-mills and foreign competitors. 6. The company has been implicated in generating water pollution and toxic waste recently. 7. US Steel had sought to induce the federal government to take action to counteract dumping of steel by foreign producers at below-market prices. Possible Answers: No answer is provided yet. Feel free to share your own answer/solution or any thought to this case by leaving a comment below.

Airbus Replaces CAD-CAM with Virtual Reality Technology Case Type: business operations strategy. Consulting Firm: Roland Berger Strategy Consultants 1st round job interview. Industry Coverage: Software & Information Technology; Airlines. Case Interview Questions #00086: Your client Airbus is a major commercial airline manufacturer. It is the aircraft manufacturing subsidiary of EADS (FWB: EAD, Euronext: EAD), a European aerospace

company. Based in Blagnac, France, near Toulouse, and with significant activity across Europe, Airbus

produces around  half of the world’s jet airliners. Recently, the head of the IT department of Airbus suggests that a new virtual reality technology can replace the current CAD-CAM (computer aided design and computer aided manufacturing) system for designing airplanes. How would you frame your analysis in trying to come up with a recommendation for Airbus? Possible Answers: Me: I would like to structure this strategy case using the 3C’s framework – Customer, Company, and Competition: Customer – Our client Airbus’ clients are people who buy commercial aircraft. They are likely concerned with price and with quality. They would also likely be concerned about how long it takes us to respond to design change requests and new designs for airplanes as well. I would ask the following questions. Will this new technology save us in production costs, allowing us to sell planes for less money while making more money? Will the new technology allow us to bring planes to market with fewer bugs/defects, improving quality in the eyes of our clients? Will this technology enable us to shorten our production cycle and bring a product to market faster? Company – Is this Virtual Reality technology viable today or in the near future? Are there competing technologies? Will the companies that we will be buying the new technology from be around next year or in five years? How reliable is the technology? Can we produce the new system in-house? Can we produce a small scale, less expensive model first to see if we can really make this work? Will this technology be able to be copied rather easily by our competitors? Can we make this a proprietary technology and hold on to this competitive advantage longer? Is the market for commercial aircraft stable or unstable? If it’s stable then there may be no need for new aircraft designs in the near future. Our capital might best be used elsewhere if this is the case. Competition – How will our competitors respond? Will they hire their own consulting company and produce a competing system? How long will this take them to do? Are they focusing on different issues such as fuel consumption or weight and strength of the metal alloys used to manufacturing? Are these more appropriate places for us to focus our attention? After answering and quantifying these questions, I would look at the cash flows resulting from this project. I would come up with a best-case scenario, a worst-case scenario, and a likely scenario. Interviewer: How would you incorporate risk into the equation?

Me: In the interest rate that I would use to discount the future cash flows. Interviewer: Could you be a little more specific about how you would go about determining the proper discount rate? Me: Sure. If Airbus had alternative uses for the capital, I would analyze those alternatives. Let’s say the alternatives are investing in treasury bonds at 7% or investing in additional parts and labor to help reduce our production backlog. Let’s say we calculate that this second alternative would return 11% over the same time horizon as the new technology project. I would use 11% as the discount rate. Interviewer: Good. Can you foresee any problems with the company’s suppliers if we decide to go with this project? Me: Thanks for the help. If we are going to dramatically reduce our cycle time in bringing new designs to market, we will require our suppliers to design and produce the necessary parts more quickly as well. Perhaps we could share our technology with our suppliers in an effort to help them do this. If ultimately our suppliers could not respond to the new time frame, then we might be investing in great technology that will yield us no benefit whatsoever. This should certainly be considered when analyzing the options. Interviewer: Great! Let’s stop there. Do you have any questions for me?

Corn Products International Refurbishes Ohio Plant Case Type: operations strategy. Consulting Firm: Hewitt Associates (now Aon Hewitt) 2nd round job interview. Industry Coverage: Food and Beverages. Case Interview Questions #00064: Your client Corn Products International (NYSE: CPO) is an Illinoisbased corn feed company that refines and processes corn-based food additives and sweeteners. Corn Products International has eight manufacturing plants located in the Midwest. These eight plants service

the entire United States. Recently, their plant in Ohio is in need of  and they have hired you to help them with it. The company has four possible options: 1. Refurbish the existing plant 2. Build a larger plant at the current location 3. Build a similar size plant at a new location 4. Build a larger plant at a new location

refurbishing

Which is the best option for your client Corn Products International? Possible Answers: There are two issues to this decision. The plant size and the plant location should be considered separately. 1. Size of Plant First consideration is the demand for the product. Corn feed is a commodity product. Pricing on the product is dependent on current corn prices as opposed to the manufacturing process. There are four main players - our client Corn Products International is the second largest. All four players have similar manufacturing processes and similar cost structure. The proposed larger plant will not have economies of scales not currently present at the existing plant. The capacity utilization of client’s existing plant is 65% which is industry standard. The current customers buy from all four manufacturers in order to guarantee supply. Currently demand is being met and there are no alternative use for corn feed. 2. Location of Plant Transportation cost and perishability are the main issues with location. The transportation cost for the corn stock (raw material) is much higher than the cost of transporting the actual corn feed. The corn is grown in the Ohio area and the feed is sold to the East Coast. The raw material is perishable where as the corn feed can be stored for any length of time and easier to transport. Cost analysis of the transportation cost of feed versus raw materials should be completed. Included in this analysis would be the % of spoilage for longer transportation of corn stock Conclusion: The current plant is located close to the corn fields and this is the best location for the plant from the cost/benefit analysis. Therefore, option #1 “Refurbish the existing plant” is the best choice for the client.

How to Save Bank One’s Retail Lockbox Services? Case Type: operations strategy; business turnaround. Consulting Firm: PricewaterhouseCoopers (PwC) 2nd round job interview. Industry Coverage: Banking. Case Interview Questions #00050: Mr. Check is the Director of Retail Lock Box Services for Bank One, a medium sized Midwestern bank. The Retail Lock Box Department consists of 100 clerks and 8 managers and supervisors. Each year, in addition to their handling of retail lock box transactions, the Department generated $1.5 million of fee revenue processing retail credit card and mortgage payments

(“items”) for 75 commercial accounts.  The bank has many other commercial accounts that use other companies of’ their item processing. In fact, the Bank recently lost the item processing business for one of its largest accounts to Visa Inc., the largest item processor in the US. The item processing industry has undergone dramatic changes in recent years. Types of items processed include credit card, mortgage, and utility payments (checks), airline tickets, and coupons. In the past, these items were usually processed by the issuing company (e.g., airlines would process their own tickets) or by bank item processing departments like Bank One’s. At banks, the processing of payment items was done more as a service to bank customers rather than as a profit making endeavor. Hence, it received little focus from management. Historically, processing was accomplished by verifying the correctness of incoming paperwork and manually sorting, filing, and totaling the items: only the largest banks were highly automated. Companies specializing in item processing have emerged in the past ten years. Visa, Inc., the largest such company, is a subsidiary of a small bank in Georgia. Each year Visa processes millions of airline tickets and retail payments for hundreds of companies, most of whom are not customers of its hundreds of competitors most of whom are not, customers or its parent bank. Visa uses high speed processing equipment and is highly automated. Processing time is rapid and processing costs are low. In fact, because of this speed advantage, the parent bank is beginning to profit from the float of checks processed. Although industry wide a majority of items are still processed by the issuing company or by small processors, it is expected that large processors. Within five years, it is expected that most of the business will continue to migrate to Visa and other large processors, such as MasterCard, American Express and Discover. Within five years, it is expected that Visa and other large processors will dominate this market. Visa had a significant cost advantage over smaller operations, such as Bank One, because of the great economies of scale they gain from processing such volumes of items. In addition, Visa benefits from a more constant workload by processing both airline tickets and retail lock box receipts: airline tickets have few peaks and valleys, whereas mortgage payments always peak early in the month with very low volumes the rest of the month. Mr. Check believes that Visa quotes prices of 20 cents per item to large prospective customers while Bank One processes items for 40 cents per item. The President of Bank One Mr. McCoy, has asked Mr. Check to evaluate how the retail lock box service can be made profitable; the service lost $100,000 last year. Mr. Check believes that the bank must offer retail lock box services, and it must price the service to be competitive with companies such as Visa. Recognizing that outside expertise will be needed, the President has given Mr. Check a budget to be used to hire a consulting firm. Mr. Check has asked you to visit his office to discuss the proposed engagement. While walking to his office, you observe that Bank One’s retail lock box operations remains

primarily a manual system, with limited use of modern, high speed equipment and methods. Once in Mr. Check’s office, you note a picture showing the Department’s staff in 1965; Mr. Check was a supervising clerks at that time. After reviewing some background information with you, Mr. Check asks you the following questions: Question #1: What do you see as your (the consultant’s) role at Bank One? Question #2: What steps would you take and what information would you gather to diagnose the problems facing the Retail Lock Box Department and to develop solutions to those problems? Question #3: From what you now know, what are the problems facing the item processing service and what recommendations would have the greatest impact on the performance of Bank One and the item processing service? Possible Answers: In this case, we want to test the candidate’s ability to handle a case in which the events appear hopeless until the end, when an apparently easy solution (automation) is made available. The candidates should challenge the general premise of the case, and not simply believe that the business is necessary just because Mr. Check says so. We also want to test creativity with this case. We purposely leave the case rather vague, not suggesting any particular actions and offering little data. The candidate should be given time to think about this case and propose solutions which are not readily apparent:  Why not sell the business of these customers?  Why not offer increased services to justify higher fees?  What is the strategic plan for the bank, and how does this unit fit into it?  What does Mr. Check feel his unit should be generating? (after all, $15,000 per employee is pretty low!)  Has he considered acquiring other banks’ customers to increase the economies of scale in his own operation? This case can also be used to discuss cost cutting. Again, creativity and sensitivity to the real issues should be the goals of your probe; cutting 25% of the staff is too obvious and too easy.

Meijer Considers Establishing Centralized Warehouse Case Type: operations strategy. Consulting Firm: Censeo Consulting Group 2nd round job interview. Industry Coverage: Retail; General Merchandisers. Case Interview Questions #00030: Your client Meijer is a regional chain of grocery stores based in Grand Rapids, Michigan. About half of the company’s 196 stores are located in the state of Michigan, with

additional locations in Illinois, Indiana, Ohio, and Kentucky. Meijer currently receives its stock on a

decentralized basis, i.e. each store deals directly with the various suppliers. The CEO of the chain is wondering whether it would be better if they established a centralized warehouse through which all supplies would be delivered and then distributed to individual store by company trucks. The CEO of Meijer has hired you to determine whether he should make the move or not. What are the key issues to consider? Possible Answer: The following are some of the key issues that should be addressed at minimum: 1. First and foremost, would the savings from bulk purchasing more than compensate for the cost of:  Building and maintaining the central warehouse;  Employing additional personnel and trucks;  Opportunity cost of capital tied up in inventory for additional periods? 2. Do all the Meijer stores buy similar products? i.e. do purchasing synergies actually exist? 3. Will delivery frequency to the stores by better or worse? Consider the costs of stockout and the need for fresh produce. 4. Will the stores prefer delivery direct from the supplier or from the central warehouse? Consider the time tied up in order processing, the flexibility of delivery times and quantities. Possible Solution: The proposed solution would depend upon your interpretation of the trade-offs both financially and organizationally for the two methods of delivery. For you to propose going with the new distribution method, you need to establish not only that it will cost less, but also that all the affected players can be persuaded to buy into it.

Alcoa Improves Manufacturing Process and Reduces Cost Case Type: operations strategy; reduce costs. Consulting Firm: Marakon Associates 1st round job interview. Industry Coverage: Mining & Metals Production.. Case Interview Questions #00025: Your client is a subsidiary company of Alcoa Inc. (NYSE: AA, name from ALuminum Company Of America) that is mainly engaged in the business of manufacturing aluminum can used for foods and beverages. Alcoa is a world leader in the production and management

of  primary aluminum, fabricated aluminum, and alumina combined, through its active and growing participation in all major aspects of the industry: technology, mining, refining, smelting, fabricating, and recycling. Recently, Alcoa’s research & development department has discovered a way to improve its aluminum manufacturing process. As a result, the client’s manufacturing cost of aluminum can has been reduced from $0.89 to $0.79 cents per unit. You have been hired to figure out how the company can best exploit this cost advantage. How would you go about the case? What recommendation would you give to the client? Possible Answers: 1. Suggested Frameworks: Remember basic economics. The company can either use a penetration strategy or price skimming strategy. Consider the impact of either strategy on the company and its competitors. Also, don’t forget to think about any substitutes for aluminum cans. 2. Possible Solution: Clearly, the client should either drop price or reap additional profits. It turns out that the client is the leader in its market with a 40% share and supplies directly to major beverage manufacturerslike Coca-Cola, PepsiCo, etc. The number two player in the market has about 30% of the market and the rest is shared by many small competitors. Aluminum cans have a lower priced substitute, steel cans, which have inferior printing and stamping characteristics. Steel cans are used by customers who do not want to pay the premium for aluminum cans. If the client drops prices, other competitors will have to follow since this is a commodity market and not following would mean a quick demise. The lowering of prices might increase the client’s market share marginally, but some smaller competitors will have to start exiting the industry and larger competitors will have to start investing to discover the client’s cost advantage. At the same time, steel can users will start switching to aluminum cans, thus hurting manufacturers in that market. The resulting growth in the aluminum can market will attract steel can manufacturers to enter it. Since some steel can manufacturers have deep pockets and a strong backing, these new entrants could pose a future threat to our client. 3. Conclusion:

In conclusion, it is best to retain prices and generate extra profits for now. The cost advantage may help another day during a price war.

AIG Assess Corporate Compensation Structure Case Type: operations strategy; organizational behavior. Consulting Firm: Mercer Consulting 1st round job interview. Industry Coverage: Insurance: Life & Health. Case Interview Questions #00022: Your client American International Group Inc. (AIG) is a large multinational insurance company with its corporate headquarters located in New York City, New York, United States. According to the 2011 Forbes Global 2000 list, AIG was the 29th largest public company in

the world.  It was listed on the Dow Jones Industrial Average from April 8, 2004 to September 22, 2008. Currently, AIG pays its insurance sales people a base salary of monthly wages and commission of 25% of new policy sales (2% of renewal). The CEO of AIG feels something is not quite right with the current compensation structure and he has hired you to assess what might be the right way to pay its sales agents. How would you go about the case? Possible Answer: This, in case you have not already surmised, is an organizational behavior and HR case. Again, you must first define what the “right way” is. Assume some generic definition like “the manner by which agents are both motivated and equipped to accomplish there tasks in the interests of the organization…” is applicable. Having set up by definition, the results achieved by the above mentioned composed system are examined. The only factor determining how much the agents paid is their sales revenue. In essence, they are motivated to issue a policy to anyone at as high a price as possible. They are not motivated to give consideration to the riskiness of the insured party. The absence of such a consideration (for example) would be detrimental to the company in the long run. A more efficient compensation structure might be paying the agent on a sliding scale, depending on how risky (costly) an insured party proves to be.

Naked Juice Considers Selling Chilled Juices Business Case Type: operations strategy; business turnaround. Consulting Firm: LEK Consulting 2nd round job interview. Industry Coverage: Food & Beverages. Case Interview Questions #00009: You are consulting for the manager of a division of Naked Juice Company, a wholly owned subsidiary of food and beverage giant PepsiCo (NYSE: PEP). Her division produces fruit juices in three forms, all marketed under the same “Naked Juice” brand name: chilled

(found in the milk section  of the supermarket, usually), juice boxes, and frozen concentrate. This division has sales of $600 million per year. The entire company has sales of over $20 billion. The chilled segment represents $120 million in sales per year. While juice boxes and frozen concentrate are profitable, chilled juices are only breaking even in good quarters and losing money in bad quarters. The division manager has just received a proposal from upper management to sell the entire chilled juices business, but she is unsure whether this is the right decision. What would you advise her to do? Should Naked Juice sell the chilled juices business? Additional Information: (to be given to you only if asked) 1. Market Share: Chilled beverages is a $5 billion dollar industry nationwide. There are two large players that have 40% and 25% of the market share, respectively. Your client’s market share, 12%, makes her third in the industry. 2. Competition: The best available information indicates that the two market leaders are profitable. Also, the two market leaders are able to fund more advertising and more promotion, trade and couponing than your client. 3. Products: The two market leaders produce pure orange juice and blends that are based on citrus juices. Your client’s product uses more elaborate blends of juices, usually with a base of pear or peach juice (95% of the inputs) and flavored with cranberries, bananas, mangoes, etc. (the other 5% of the inputs). Pear and peach juice are about the same price as orange juice, but the other flavorings cost about twice as much. 4. Customers: The market for chilled juices is essentially mothers with school age children. This is a highly price sensitive market that loves coupons, promotions, etc. 5. Price: Brand name is important in this market, as in juice boxes and frozen concentrate, as mothers tend to prefer highly reliable products for their children. However, the brand premium must be in line with other branded products. Therefore, all branded juices tend to sell in the same price range. 6. Company: The client has one plant in California that produces all of the product: chilled, juice boxes and frozen. It would be difficult to find another use for the plant without a major conversion. Possible Solutions: Basically, there are three options for your client Naked Juice: 1. Sell the entire chilled juice business. This would, however, affect the juice boxes and frozen concentrate businesses, as there are both advertising and manufacturing synergies. 2. Sell all of the juice business. This may be more feasible, as the buyer could capture the synergies, but would not be too likely to turn the business around. The selling price is likely to be low.

3. Keep the chilled juice business and rework the ingredients and costs. This turns out to be the most feasible option, as evidenced by the success of the competitors.

P&G Pet Food Approached by Wal-Mart Case Type: new product; operations strategy. Consulting Firm: Deloitte Consulting internship interview. Industry Coverage: Household Goods, Consumer Products; General Merchandisers. Case Interview Questions #00004: Your client Procter & Gamble Co. (P&G, NYSE: PG), a leading pet food manufacturer, has experienced a loss of market share and a downturn in top line revenue over the past 5 years. P&G’s primary channel of distribution is veterinarians who recommend the brand to their

customers  and sell directly from their offices. In fact, veterinarians are largely responsible for the launch of the company and the identity of the brand, as they contributed freely to the development of the quality formula that distinguishes P&G from competitors. P&G now must make a decision critical to its future. The company has just been approached by retail chains Wal-Mart (NYSE: WMT) and asked to sell its most popular products through Wal-Mart’s chain of stores. There is one problem: Wal-Mart would sell the products at significantly lower prices than could the veterinarians. You have been asked to help P&G decide whether to pursue this alternative channel of distribution, and if so, how? Additional Information: (to be given to you if asked):  The number of veterinarians distributing P&G’s products has remained constant over the 5 year period in question.  There are a number of customers who have been purchasing P&G products from veterinarians that continue to do so; however, an increasing number of customers are more resistant to paying the premium prices attached to P&G products and instead purchase pet food from discount retailers like Wal-Mart.  After covering fixed costs the company enjoys a substantial profit off its premium priced goods.  P&G products are only sold at veterinarians’ offices and consumers generally associate high quality and nutritious value with the pet food. Consumers that are willing to pay premium prices for P&G products believe that the food is superior to any close substitute on the market. Since these consumers trust that their veterinarians would only recommend the best, they subsequently choose only P&G products. The brand name is recognized widely and is always aligned with quality.  While P&G’s current customers value perceived quality, Wal-Mart customers value price. As a result, Wal-Mart customers, who make purchasing decisions based on price, will not place high enough value on quality to pay a premium to obtain it. To cater to this potentially highly profitable

customer segment, P&G must instead compete with other pet food companies in the Wal-Mart domain on price. Possible Answer: Suggested Frameworks: In establishing a framework for this case, be sure to look at these three factors:  The reasons for the downturn in revenue and loss of market share  The impact of increased sales volume  The importance of P&G’s unique positioning Possible Solution: 1. Create a new line of products under the P&G name with cheaper/lower quality inputs – while this may initially draw customers at Wal-Mart, I would not recommend this strategy. I think P&G would lose nearly its entire veterinarian business as it may damage its brand image through the new offering. P&G would no longer be associated with its current superior standing in the industry. 2. Create the new product line and sell Wal-Mart the rights to sell the goods generically, under the WalMart name – while I think this strategy would work, I think P&G is not leveraging one of its core strengths: its brand. So while it will sell more volume with this strategy, I do not think it will reach its potential earnings if it employed a different strategy. 3. Create a new line of products under a new name that is just endorsed by P&G – this may be the most feasible solution. While the new product line would enjoy benefits being associated with the major brand, it still stands on its own with a new, unique name. Thus, the two offerings will be differentiated and neither should cannibalize the other’s sales. Got a better solution? Leave a comment below!

Kimberly-Clark to Increase Profitability in Paper Stationery Case Type: business turnaround; operations strategy. Consulting Firm: ZS Associates 2nd round job interview. Industry Coverage: Household Goods & Consumer Products. Case Interview Questions #00002: You are a management consultant hired by Ms. Pepper, the head of a division of Kimberly-Clark Corporation (NYSE: KMB, BMV: Kimber), a large consumer products company that produces mostly paper-based products. Her division produces paper products in

three  forms: toilet tissue, paper plates/cups, and stationery (writing paper, note pads, decorative envelopes, blank cards, etc). While toilet tissue and paper plates/cups are both profitable, stationery products are barely breaking even in good quarters and losing money in bad quarters. Ms. Pepper has just received a proposal from upper management to sell the stationery business and she wants us to help her determine if she should sell it. Or if not, what she should do to turn around the stationery business to make it profitable. What would you suggest she do? Additional Information: (to be given to you only if asked) 1. Market Share: Stationery is a $15 billion dollar industry nationwide in the US. There are two major players that have 30% and 25% of the market share, respectively. Your client’s market share, 12%, makes her third in the industry. 2. Competitions: The best available information indicates that the two market leaders are profitable. The two market leaders are able to fund more advertising and promotions, such as coupons, than your client. 3. Company: Ms. Pepper’s division has sales of $825 million per year. The entire company has sales of over $35 billion. Stationery products represent $118 million in sales per year. One plant in California produces all of the paper products: toilet tissue, paper plates/cups and stationery. It would be difficult to find another use for the plant without a major conversion. 4. Products: The two market leaders produce pure stationery products using only cellulose, which is derived from plant fibers. The cellulose is made by cutting down trees and then grinding up trees and dumping the wood pulp in acid. Your client’s product uses a more elaborate process to produce recyclable stationery paper. In addition to the wood pulp in acid, the client’s product uses special fiber pellets. The wood pulp in acid is the same cost as the competitors, but the other items necessary for creating recyclable materials cost about twice as much. Actually, the company does not have a real preference for which type of paper is produced. The recyclable goods niche was at one time perceived to be an attractive market, but the high costs of production have limited our profitability in that space. 5. Customers: The market for stationery products is essentially mothers with school age children, young adults, and college and graduate students. This is a highly price sensitive market that loves coupons, promotions, etc. 6. Price: Brand name is very important in this market, as in paper towels and paper plates/paper cups. However, this market is also very price sensitive; as you may note earlier, it is very responsive to coupons and other price promotions. Additionally, because this market is so responsive to price all branded

products tend to sell in the same price range. Kimberly-Clark is no exception, and their stationery products are priced much like their competitors’ products. Possible Answers: Suggested Frameworks: To solve this profitability case, the job candidate could explore the following in more details:  The product mix and production process.  The industry environment and competitive landscape.  The customer base and consumer profile. Possible Recommendations: 1. Sell the stationery business. This would, however, affect the toilet tissue and paper plates/cups businesses, as there are both manufacturing and advertising synergies. Also, it would be difficult to get a good price for the stationery business as it is currently not very profitable. 2. Sell the entire paper division. This may be more feasible, as the buyer could capture the synergies, however the large consumer company would lose valuable revenue from the toilet tissue and paper cups/plates divisions. 3. Keep the stationery paper business and rework the ingredients and recycling process. This could potentially entail eliminating or replacing the unique fiber pellets which are driving higher costs.